PNC 1 Final Practice Questions

¡Supera tus tareas y exámenes ahora con Quizwiz!

A construction worker admitted to the unit with a chest injury and broken ribs from a fall from a ladder has nursing diagnoses of "Disturbed Sleep Pattern," "Ineffective Breathing Pattern," and "Risk for Infection." The client states, "I've never been sick a day in my life and am really worried about how I can support my family while I'm out of work." When evaluating the client's plan of care during the shift, the nurse adds the nursing diagnosis "Anxiety" to the plan of care. Which diagnosis would be the priority for nursing interventions? A) Risk for Infection B) Ineffective Breathing Pattern C) Disturbed Sleep Pattern D) Anxiety

B) Nursing diagnoses of "Ineffective Breathing Pattern," "Disturbed Sleep Pattern," and "Risk for Infection" are diagnoses that need to be considered because of the client's inability to fully expand his lungs because of his rib injury. Of these, "Ineffective Breathing Pattern" would receive the highest priority in the nurse's care planning. Addressing the client's disruption in sleep patterns in order to promote quicker healing and taking measures to minimize his risk for infection are also important, but are lower in priority than addressing his current ineffective breathing. In addition, addressing the client's psychosocial needs-his worry about being unable to support his family while he recuperates-is also an important aspect of holistic nursing care, but again of lesser priority than his ability to breathe effectively.

Which of the following terms is used to describe osteoarthritis (OA) that is caused by an underlying condition, such as injury, congenital malformation, or metabolic disease? A) Idiopathic B) Secondary C) Localized D) Generalized

B) OA can be classified as either idiopathic or secondary. Idiopathic OA has no identifiable cause. Idiopathic OA can be further divided into localized or generalized, with localized OA affecting one or two joints and generalized OA affecting three or more joints. Secondary OA is caused by an underlying condition, such as injury; congenital malformation; metabolic, endocrine, or neuropathic disease; or other medical cause.

The nurse is providing care to several clients in an outpatient clinic. Which client is at high risk of developing gastroesophageal reflux disorder (GERD)? A) A client who is 6 weeks pregnant B) A client who is morbidly obese C) A client who follows a strict vegetarian diet D) A client who drinks one glass of wine monthly

B) Obesity is a risk factor for GERD. Pregnancy is an increasing risk factor in the later stages due to pressure on the stomach. A vegetarian diet is not a risk factor for GERD. Rare alcohol consumption is not as strong a risk factor for GERD as morbid obesity.

A young adolescent client is in the hospital preparing for major surgery for the removal of a tumor on the kidney. The client's mother tells the nurse that she doesn't want her child to receive narcotics for postoperative pain. What is the nurse's best response? A) "Okay, I'll tell the healthcare provider not to order any. Are you sure you want to do this?" B) "The pain will be severe. Why don't we ask your child about this?" C) "Your child's pain will be severe after the surgery. Can you tell me why you feel this way?" D) "You do not have a choice of medication. Decisions involving pain relief are up to the healthcare providers."

A young adolescent client is in the hospital preparing for major surgery for the removal of a tumor on the kidney. The client's mother tells the nurse that she doesn't want her child to receive narcotics for postoperative pain. What is the nurse's best response? A) "Okay, I'll tell the healthcare provider not to order any. Are you sure you want to do this?" B) "The pain will be severe. Why don't we ask your child about this?" C) "Your child's pain will be severe after the surgery. Can you tell me why you feel this way?" D) "You do not have a choice of medication. Decisions involving pain relief are up to the healthcare providers."

The nurse is assessing a client for a sleep-rest disorder. Which client behavior is most indicative of such a disorder? A) The client answers history questions in detail. B) The client expresses impatience after two or three questions and insists that the nurse hurry up and finish. C) The client expresses a concern about the privacy of the information he reveals. D) The client cannot answer definitely whether he sleeps in strange positions.

B) Observation can indicate the presence of fatigue, decreased cognitive functioning and coordination, dark circles under the eyes, and irritability. Expressing impatience after just two or three questions is indicative of irritability, which could be a sign of sleep deprivation from a sleep-rest disorder. Answering history questions in detail does not demonstrate decreased cognitive functioning, a concern about privacy is a normal concern, and the client may not be able to answer about strange sleep positions because he may simply not know.

A nurse is reviewing the medical record for a school-age client prior to a scheduled health maintenance visit. Which data from the record indicates that the client is overweight? A) Body mass index (BMI) >85th percentile B) BMI >95th percentile C) 25% increase in weight in a 6-month period D) 35% increase in weight in a 6-month period

A) A child with a BMI greater than the 85th percentile is considered overweight. A child with a BMI greater than the 95th percentile is considered obese. Percentage of weight gain in a 6-month period (regardless of baseline) does not determine whether a client is overweight or obese.

A client presents with delayed wound healing. During the physical assessment, which nutrient deficiency does the nurse anticipate based on the data? A) Protein B) Digestive enzymes C) Insulin D) Carbohydrates

A) A deficiency of protein may delay wound healing. Digestive enzymes aid in the digestion of nutrients. Insulin allows glucose to be used by the cells. Carbohydrates are a source of energy.

A client reports morning headaches that extend into the neck and go away as the day wears on. Based on this initial data, which assessment finding does the nurse anticipate? A) Elevated blood pressure B) Tachycardia C) Otitis media D) Swollen lymph nodes

A) A headache, generally in the back of the head and neck, that is present on awakening and subsides during the day is an early sign of hypertension. The nurse would expect that the client's blood pressure would be elevated. This type of headache is not directly associated with tachycardia, otitis media, or swollen lymph nodes

A charge nurse is making assignments for the shift and notes that a client from a different culture was recently admitted and will require a thorough admission assessment during the upcoming shift. Which generation of nurse is likely to be the most culturally sensitive and thus the best choice for this client assignment? A) The Millennial nurse B) The Generation X nurse C) The veteran nurse D) The baby boomer nurse

A) A nurse from the Millennial generation would most likely exhibit the highest level of cultural sensitivity and would thus be the best choice for this assignment. Broadly speaking, Millennial generation nurses have received the most education regarding culturally sensitive care and can be a good resource for the older nurses on the unit.

An individual's work ethic reflects his or her A) belief in the importance and moral worth of work. B) desire to advance in a chosen profession. C) level of affective commitment to a profession. D) degree of optimism with regard to the work environment.

A) A person's work ethic reflects his or her belief in the importance and moral worth of work. An individual does not need to be part of a specific profession in order to have a strong work ethic. Although an optimistic attitude is often associated with a strong work ethic, optimism itself does not determine an individual's work ethic.

The nurse is teaching a school-aged client how to use a peak flow meter to monitor asthma. Which approach by the nurse is most likely to result in the desired outcome? A) Providing positive reinforcement after every attempt B) Explaining that asthma can be fatal if not monitored and treated C) Telling the client he cannot play until he learns to use the meter correctly D) Using colloquialisms and slang when describing how to use the meter

A) A sense of achievement is very important to school-aged children, and rewarding and reinforcing behavior is most effective. Scare tactics and negative reinforcement are not appropriate. Nurses should use clear, concrete language to enhance understanding.

How does accountability differ from responsibility? A) Responsibility involves specific tasks that must be completed in order to fulfill a role, whereas accountability involves being answerable for the outcomes of those tasks. B) Accountability involves specific tasks that must be completed in order to fulfill a role, whereas responsibility involves being answerable for the outcomes of those tasks. C) Responsibility involves the professional standards used to determine what a nurse should or should not do, whereas accountability involves taking ownership of the actions of others. D) Accountability involves the professional standards used to determine what a nurse should or should not do, whereas responsibility involves taking ownership of the actions of others.

A) Accountability involves being answerable for the outcomes of a task or assignment. Nurses are accountable for their own actions and behaviors, but they may also be accountable for the actions of others, such as subordinates or trainees. Responsibility is the specific obligation associated with the performance of duties of a particular role, and it belongs to the individual performing the duties. Nurses are responsible for performing their assigned tasks reliably, dependably, and to the best of their abilities.

A nurse is working with an adolescent client who is attempting to lose weight. The client admits having difficulty being compliant with the diet prescribed by the healthcare provider. Which suggestion by the nurse might assist the client in being compliant with the prescribed diet in a way that is sensitive to the client's age? A) "It can be difficult to avoid unhealthy foods if that's what your friends are eating, but try to choose healthier options when you can." B) "Write down the exact foods you eat so that you can see what and how much you are eating." C) "Watch the nutrient content and number of calories in everything you eat." D) "Eat at the kitchen table so that you eat along with the rest of the family."

A) Adolescence is a time of identity formation, and adolescents align with peers in regard to food selection. Keeping food diaries and monitoring the nutrient content and caloric values of food intake are helpful behavior modification strategies, but these don't take into account the age of the client, and studies on food consumption show that caloric information or nutrient content is not a major consideration in choice among adolescents. Parental food choices can have a strong impact on adolescents, but some adolescents rebel against these food choices, positively or negatively, and eating with the rest of the family is only beneficial if the family's habits are healthy.

The nurse is conducting a clinic visit with a mother and an adolescent client. Both the mother and the adolescent report that the adolescent is not able to sleep until late at night and then wakes up too late in the morning. This has caused the adolescent to be late for school several times. The mother states, "I don't know what to do." Which response by the nurse is the most appropriate? A) Inform her that several techniques can potentially help correct this problem. B) Recommend a polysomnography (PSG). C) Indicate that this is simply a normal change of the body's internal clock associated with puberty, and it will work itself out. D) Ask about other medical conditions because the adolescent's sleep patterns indicate the onset of a chronic illness.

A) Adolescents experience changes in the body's internal clock associated with puberty that in some adolescents cause a delay in melatonin release each night; the result is delayed sleep phase syndrome. Signs of delayed sleep phase syndrome include an inability to fall asleep and wake up at the desired time. The use of sleep hygiene, sleep restriction therapy, and relaxation techniques can be beneficial in the treatment of this syndrome. A sleep study would only be indicated if the adolescent were experiencing other sleep disturbance symptoms. This isn't a sign of another illness but shouldn't be ignored either.

The nurse is conducting an assessment on a client who is 36 hours postoperative following an appendectomy. During the assessment, the nurse is unable to hear any bowel sounds. The client denies passing flatus. Given this information, which action is most appropriate by the nurse? A) Withholding food and oral fluids until intestinal motility has returned B) Encouraging the client to increase oral fluid intake to promote peristalsis C) Encouraging the client to increase solid food intake to promote peristalsis D) Encouraging the client to decrease the amount of oral food and fluid intake

A) After abdominal surgery, the risk of a paralytic ileus exists. An ileus results when the bowel is not experiencing peristalsis. Oral intake of both food and fluids must be withheld during this time.

The nurse at a health fair is educating clients on risk factors associated with urinary incontinence. Which risk factor does the nurse include as a nonmodifiable risk factor for urinary incontinence? A) Age B) Obesity C) Smoking D) Diabetes

A) Age is a nonmodifiable risk factor and is a primary risk factor for the development of urinary incontinence; older individuals experience more frequent incontinence than younger individuals. Obesity, smoking, diabetes, inactivity, pregnancy, and depression are all modifiable risk factors for urinary incontinence

A patient with an allergy to latex develops contact dermatitis following an examination during which the nurse wore latex gloves. Which best describes the associated pathophysiology? A) An immune response that leads to issues with tissue integrity B) Impaired tissue integrity that leads to an immune response C) Impaired tissue integrity that leads to an infection D) Decreased perfusion that leads to issues with tissue integrity

A) Allergic reactions are an example of an immune response that leads to issues with tissue integrity. Impaired tissue integrity, such as a cut, can lead to an immune response, but that is not the case in this scenario. If left untreated or exposed to bacteria or other infectious agents, the dermatitis could lead to an infection, but there is no evidence of that in this scenario. Decreased perfusion can lead to tissue damage or death, but not dermatitis.

The nurse is working with a morbidly obese client who is seeking help to lose weight at a bariatric clinic. When planning this client's care, which nursing diagnosis is the priority? A) Activity Intolerance B) Disturbed Body Image C) Defensive Coping D) Constipation

A) Along with diet, exercise is an important part of a weight loss program. A client with morbid obesity has a sedentary lifestyle and will have activity intolerance. Disturbed Body Image and Constipation may both be legitimate diagnoses, but Activity Intolerance is a greater priority if the client is to lose weight. There is no evidence that this client exhibits defensive coping.

The nurse is providing care to a client with a compromised immune system. Which independent nursing intervention is appropriate for the nurse to include in the client's plan of care? A) Educating the client on the importance of a nutritious diet B) Administering corticosteroids per order C) Prescribing prophylactic antibiotic therapy D) Recommending gene transfer therapy

A) Although all of these interventions may be appropriate for a client with a compromised immune system, the only independent nursing intervention is educating the client on the importance of a nutritious diet. It is outside the scope of nursing practice to prescribe medication and to recommend therapies. However, the nurse can administer antibiotics and educate the client on gene transfer therapy if doing so is prescribed by the healthcare provider.

A client with a history of gastroesophageal reflux disorder (GERD) presents with metabolic alkalosis. Based on the data reviewed in the client's history, which medication does the nurse suspect contributed to the current diagnosis? A) Aluminum hydroxide B) Omeprazole C) Ranitidine D) Metoclopramide

A) Aluminum hydroxide antacids neutralize gastric acid. Overuse of antacids may cause metabolic acidosis. Omeprazole, ranitidine, and metoclopramide are all GERD medications that do not cause metabolic alkalosis

Which physiological changes associated with aging increase the risk of hypertension in older adults? A) Increase in systolic blood pressure B) Increase in diastolic blood pressure C) Increase in the pulse pressure D) Decrease in the diastolic blood pressure

A) An age-related increase in the systolic blood pressure is the primary factor leading to the high incidence of hypertension in older adults. Unlike the diastolic blood pressure, which tends to rise until approximately age 50 and then decline, the systolic blood pressure continues to rise with age. The pulse pressure, which is the difference between the systolic and diastolic blood pressures, does not determine hypertension status.

What drug may be used to treat nausea and vomiting associated with operative procedures? A) Metoclopramide B) Acetaminophen C) Midazolam D) Fentanyl

A) An antiemetic such as metoclopramide may be used to treat nausea and vomiting associated with operative procedures. A nonopioid analgesic such as acetaminophen provides temporary analgesia for mild to moderate pain. An anxiolytic such as midazolam is a relaxant. An opioid analgesic such as fentanyl controls moderate to severe pain but does not alter the pain threshold.

The nurse is planning care for a client who is experiencing an alteration in mobility. Which would the nurse include as an independent nursing intervention? A) Instructing on the importance of proper nutrition and an active lifestyle B) Administering a prescribed nonsteroidal anti-inflammatory drug (NSAID) C) Identifying necessary modifications to the home environment D) Prescribing a skeletal muscle relaxant

A) An appropriate independent nursing intervention for a client who is experiencing an alteration in mobility is providing instruction on the importance of proper nutrition and an active lifestyle. Administering a prescribed NSAID is an example of a collaborative intervention that the nurse can implement. Identifying necessary modifications for the home environment is a collaborative intervention often implemented by the occupational therapist. Although it is appropriate for the nurse to administer a skeletal muscle relaxant, it is outside the scope of nursing practice to prescribe this medication.

The nurse must start an IV for a toddler in the emergency department. The toddler is accompanied by a parent. The parent asks the nurse, "Can I stay with my child and comfort him?" Which response by the nurse is best? A) "Yes, it would be helpful for you to stay and comfort your child." B) "We do this all the time, so don't worry. I will come get you when we are done starting the IV." C) "Be ready to hold your child down when I tell you to." D) "It doesn't take long to start an IV, so there's no need for you to stay for the procedure."

A) An important part of nurse competency is knowing what procedures to follow when performing skills. For example, when initiating an IV on a pediatric client, procedure dictates that the nurse should always seek assistance from the client's parent if the parent is willing and capable of offering assistance. If the parent agrees to offer assistance, the nurse can then provide appropriate teaching. Instructing the parents to hold down the child without giving them a choice is inappropriate; many parents do not want to participate in activities that cause pain to their child. Telling the parent not to worry is both pointless and dismissive of the parent's concerns. It is also inappropriate for the nurse to say that the procedure won't take long, because the nurse does not know how much time the procedure will actually require.

The nurse is caring for a client who was admitted to the emergency department with abdominal pain. The client speaks very little English and requires an emergency appendectomy. The nurse has enlisted the hospital interpreter to explain the procedure and help with informed consent. When the interpreter arrives, which action by the nurse is appropriate? A) Ask the interpreter to translate as closely as possible. B) Ask the client's family to be included in the interpreting process and exchange of information. C) Direct questions to the interpreter and not the client. D) Request that the interpreter use the same dialect as the client to promote understanding.

A) An interpreter is an individual who mediates spoken or signed communication between people who use different languages without adding, omitting, distorting meaning, or editorializing. It is not the interpreter's responsibility to determine the dialect with which the client is most familiar. The nurse should direct all questions to the client, not the interpreter. The nurse should also avoid asking the client's family, especially a child or spouse, to help interpret.

The nurse is providing care to a client who is experiencing urinary retention. Which diagnostic tool does the nurse anticipate will be ordered for this client? A) Ultrasonic bladder scan B) Urinalysis C) Intravenous pyelography (IVP) D) Cystoscopy

A) An ultrasonic bladder scan is the diagnostic test that is used to examine for residual urine. A urinalysis is often used to monitor the urine for infection. An IVP is used to diagnosis a kidney stone. A cystoscopy allows direct visualization of the bladder wall and urethra. It is often used to remove stones.

The nurse is providing care to a client who experiences chronic inflammation due to arthritis. For which collaborative intervention should the nurse plan when providing care to this client? A) Administering anti-inflammatory medications B) Administering diuretics C) Administering frequent doses of opioid medications D) Administering antibiotics

A) Anti-inflammatory medication will reduce the pain and inflammation caused by arthritis. Opioid medication is not usually indicated to treat a chronic inflammatory process. Antibiotics would be ordered for an infection, not for chronic inflammation. Finally, diuretics are not used to treat the inflammatory process.

A nurse failed to complete a competency within the required time frame. When confronted by the manager, the nurse says, "Oh well, it's not like I don't know what I'm doing." This nurse's statement is most reflective of which of the following qualities? A) Arrogance, which prevents the nurse from developing an accurate assessment of personal strengths and weaknesses B) Pessimism, which endangers the nurse's professionalism C) Optimism, which helps the nurse realize that things will turn out for the best D) Sarcasm, which threatens the nurse's job performance

A) Arrogance, or excessive pride and a feeling of superiority, can be an extremely dangerous characteristic in the nurse, as it can lead to a false belief that the nurse is always right and does not need input from others. Accurate self-assessment of strengths and weaknesses and acceptance of feedback from others promote both safety and growth and are therefore essential abilities for the nurse.

The nurse is assessing an adult client in a urology clinic. The client reports that she has been having "accidents" and expresses frustration about this "normal part of aging." Which response by the nurse is the most appropriate? A) "Tell me more about the incontinence you are experiencing." B) "You may need to have surgery to manage this problem." C) "I understand you are frustrated about this occurrence." D) "Unfortunately, aging and incontinence go hand in hand."

A) As the body ages, there are anatomic changes that can increase the risk for urinary incontinence; however, this is not a normal part of aging. It is appropriate for the nurse to gather more information regarding the client's incontinence. It is beyond the nurse's scope of practice to recommend surgery to the client. Telling the client the nurse understands does not help to determine the cause of the client's incontinence

The nurse is providing care for several clients. For which client should the nurse anticipate an order for administering 1000 mg of aspirin? A) A 68-year-old client with rheumatoid arthritis who is experiencing hand pain B) A 5-year-old client who is experiencing ankle pain after a fall from a horse C) A 38-year-old client who is experiencing headache pain after a skiing accident D) A 70-year-old client who is experiencing back pain after laminectomy

A) Aspirin is appropriate for the client with rheumatoid arthritis who is experiencing hand pain, assuming there are no other contraindications. This medication is not appropriate for the other clients, however. Aspirin therapy is not recommended for children because it is associated with an increased risk of Reye syndrome, and it may contribute to bleeding in adult clients who have sustained physical injury.

A nurse educator in a medical-surgical unit is demonstrating the use of new equipment to the rest of the nurses on the unit. After initial efforts at having the class gather closely around the models were met with discomfort and inattention, the nurse educator sets up the models in the front of the classroom. Which level of proxemics would be ideal for this situation? A) 4 to 12 feet B) 1 1/2 to 4 feet C) 12 to 15 feet D) Less than 1 1/2 feet

A) Because the nurse is communicating with a group and all members of the group must be able to see the models clearly, a social distance of 4 to 12 feet would be appropriate. Social distance is characterized by a clear, visual perception of the whole individual, and it is expedient for communicating with several people at the same time. In contrast, an intimate distance of 1 1/2 feet or less would likely make the group members extremely uncomfortable and prevent them from seeing all aspects of the demonstration. A personal distance of 1 1/2 to 4 feet might still cause discomfort among participants and make it difficult for them to observe the models in full. Finally, a public distance of 12 to 15 feet would make it hard for participants to observe the models' smaller features and would also make the communication process less personal for all parties involved.

The nurse is providing care to a client with arterial blood gas analysis as follows: PaO2 of 82, PaCO2 of 49, HCO3 of 26, and pH of 7.31. Which assessment by the nurse is correct? A) Respiratory acidosis B) Respiratory alkalosis C) Metabolic alkalosis D) Metabolic acidosis

A) Both the pH and the carbon dioxide levels represent acidosis. The PaO2 of 82 is on the low end of normal and the bicarbonate level is normal, indicating that this is respiratory acidosis rather than metabolic acidosis.

Reducing the risk of functional decline in older adults can help prevent which complication? A) Pressure ulcers B) Macular degeneration C) Hyperglycemia D) Hearing loss

A) By reducing the risk of functional decline, nurses and independent older adults can help prevent complications such as pressure ulcers, delirium and depression, decreased mobility, loss of independence, and incontinence. Macular degeneration, hearing loss, and hyperglycemia are not complications that occur as a result of functional decline.

How can a healthcare facility's management best take advantage of the multigenerational nursing staff? A) By emphasizing the unique strengths each generation brings to the workplace B) By encouraging the hardworking, loyal veterans to serve as mentors for the younger nurses C) By highlighting the adaptability of the techno-savvy Generation X nurses D) By encouraging the older nurses to adopt the optimistic viewpoint of the Millennial nurses

A) Celebrating the unique strengths of each generation of nurse can decrease interpersonal tension and facilitate personal growth. Nurses who learn to acknowledge and appreciate their colleagues from different backgrounds, including generational backgrounds, have a distinct advantage. The best teams use the contributions of each generation's skill set and strengths. The hardworking, loyal veterans; the idealist, passionate baby boomers; the technoliterate, adaptable Generation Xers; and the young, optimistic Millennials can come together in a powerful network with a remarkable ability to support each other and maximize client care.

The nurse is presenting a program at a senior center on how to survive a fall. Which statement by a program participant indicates that this person needs clarification about what emergency actions to take after a fall? A) "I should crawl to a phone on the affected side to keep it stable against a hard surface." B) "I should try to cover myself with a blanket while I wait for help to arrive." C) "To call for help, I can scoot on my bottom to a low wall-mounted phone." D) "If possible, I can crawl to a stairway and use the stairs to lift up to a standing position."

A) Clients at risk for falls and hip fractures should be taught how to notify emergency services in the event of a fall and injury. These clients should be instructed to turn onto the stomach and crawl to the phone, or to scoot to the phone using the buttocks on the uninjured side. And another option is to crawl to a stairway and use the stairs to gradually lift the body to a standing position. While waiting for help to arrive, clients should cover themselves with a blanket if possible to help prevent shock.

A student nurse says, "I'm nervous about taking the NCLEX, and I'm a little worried about all the responsibility that comes with being a registered nurse. At the same time, I'm eager to pass the test so I can start working. I think the rewards of nursing are well worth the hard work and sacrifice." This statement suggests that the student nurse is in which stage of commitment? A) The integrated stage B) The testing stage C) The passionate stage D) The exploratory stage

A) Commitment to a profession develops in five stages: exploratory, testing, passionate, quiet and bored, and integrated. The integrated stage is the final stage of commitment development. Individuals who reach this stage have integrated both positive and negative elements of the profession into a more flexible, complex, and enduring form of commitment. They act out their commitment as a matter of habit. These students are in the final stages of their nursing program and are beginning to see themselves as nurses, eager to take the NCLEX-RN and begin employment.

A client seeking treatment for severe knee pain has worked in a factory for 30 years in a position requiring repetitive lifting and carrying of 20- to 40-pound boxes. Based on the client's history, the nurse should anticipate which initial recommendation from the multidisciplinary healthcare team? A) Joint replacement surgery B) Pharmacologic therapy C) Referral for a disability application D) Intermittent use of a cane

B) Of these options, pharmacologic therapy would be the most likely initial intervention. Acetaminophen, nonsteroidal anti-inflammatory drugs (NSAIDs), and joint injections are all possible options. Joint replacement should be delayed as long as possible because artificial joints often require replacement within 15-20 years. There is not enough information to determine whether applying for disability is appropriate at this time. A cane is not indicated at this time

The nurse is caring for a critically ill infant in the neonatal intensive care unit (NICU). The infant's parents ask whether the child's 3-year-old sibling can visit. Small children are not permitted to visit the NICU because of the risk of infection to the infants on the unit. Which action by the nurse best demonstrates a compassionate response? A) Seek permission from unit management and the neonatologist to allow the sibling to visit B) Offer to make counseling available to the sibling C) Tell the parents that visiting is not permitted and offer to take pictures of the client D) Tell the parents to bring the sibling in to visit in the middle of the night

A) Compassion, or awareness of and concern about other individuals' suffering, is an important component of professionalism. Nurses demonstrate compassion when they recognize a client's need and respond appropriately to meet that need. In this situation, although the nurse would realize that certain rules are in place for the protection of clients, the nurse would also recognize the need for compassion. Of the options provided, the most appropriate way to demonstrate compassion would be to seek permission to allow the sibling to visit and see that the infant is alive and being cared for. The nurse would not make the decision without the permission of the management team. Telling the parents what the rules of the unit are will only increase the family's frustration when they are looking to the nurse for assistance. Offering counseling may or may not be an effective means of assisting the family.

A client is experiencing sudden-onset severe pain in the left lower quadrant of the abdomen that is rated as a 10 on a pain scale of 0-10. The client is also experiencing nausea, vomiting, and restlessness. Based on this data, the nurse concludes that the client is experiencing which phenomenon? A) Acute pain B) Chronic pain C) End-of-life pain D) Fibromyalgia pain

A) Duration establishes the difference between acute and chronic pain. Acute pain is defined as pain that lasts only through the expected recovery period, which is usually 30 days to 6 months. Acute pain typically has a sudden onset related to injury, surgery, or illness. Chronic pain outlasts the illness and extends beyond the recovery period. End-of-life and fibromyalgia would most likely involve chronic pain.

A goal of care for a client with congestive heart failure (CHF) is for serum sodium levels to be within normal limits. Which information documented in the medical record would indicate that the client is not meeting this goal? A) The client is experiencing dependent edema. B) The client experiences joint pain. C) The client is constipated. D) The client is experiencing wheezing respirations.

A) Edema can be a result of a high sodium level, so the nurse who assessed that the client has dependent edema would determine that the goal for normal serum sodium levels is not being met. Joint pain would be relevant for a different goal. Constipation would not be related to a high sodium level. Wheezing would be addressed by a different goal for the client.

Which of the following statements best characterizes vitamin use? A) Clients should be careful not to exceed recommended allowances for daily vitamin intake. B) Vitamin D is dangerous if taken in large quantities, but there is no upper limit to Vitamin C intake. C) Generally, two multivitamin pills a day is recommended for all clients regardless of diet. D) Fat-soluble vitamins in general present the least risk of toxicity to clients who take them in excess.

A) Excess consumption of some vitamins, especially the fat-soluble vitamins, can lead to significant toxicity. The disorder is referred to as hypervitaminosis. When taken in excess, vitamin D can cause bone destruction, rather than contributing to bone formation. Excess intake of vitamin C can lead to diarrhea, nausea, and stomach cramps. Two multivitamin pills a day is probably excessive because one such vitamin typically contains the recommended intakes of most vitamins and minerals needed on a daily basis, and determinations of vitamin intake should always consider diet.

Which of the following statements best characterizes risk for injury as it relates to pain? A) Risk for injury is an external risk factor for pain. B) It is difficult to predict what might pose a risk for injury. C) Risk for injury can be decreased by living a healthy lifestyle. D) Precautions against risk for injury are rarely successful in preventing injury

A) External risk factors for pain, such as the risk for injury, can be decreased by safety precautions such as wearing a seat belt or helmet. Similarly, internal risk factors for pain, such as the risk for developing a chronic disease, can be decreased by living a healthy lifestyle. It is not difficult to anticipate what might pose a risk for injury.

A nurse is always on time, neat in appearance, and caring toward clients. Which statement best characterizes this nurse's level of professional commitment? A) The nurse is demonstrating a pattern of behaviors congruent with nursing's professional code of ethics. B) The nurse is demonstrating a strong belief in and acceptance of the unit's goals, values, and morals. C) The nurse is demonstrating a willingness to exert control over personal behaviors. D) The nurse is demonstrating a strong desire to be part of a group.

A) Factors associated with professional commitment include a strong belief in and acceptance of the profession's code, role, goals, values, and morals; a willingness to exert considerable personal effort on behalf of the profession; a strong desire to maintain membership in the profession; and a pattern of behaviors congruent with the nurses' professional code of ethics.

A pregnant client experiencing fatigue describes her typical diet as consisting of fast food such as burgers and fries for lunch, takeout from Chinese or Mexican restaurants for dinner, and orange juice or a fruit smoothie in the morning. What is the problem with this diet? A) The fast food because it's likely high in fat. B) The takeout food might be saucy and heavy. C) The burgers contain red meat. D) The fruit smoothies are processed.

A) Fatigue in expectant mothers is often caused by iron-deficiency anemia. Expectant mothers should be encouraged to eat more protein, including lean meats and beans, and to take an iron supplement. Vitamin C increases iron absorption, so pregnant women should be encouraged to eat fruits and vegetables that contain high amounts of vitamin C. The fast food would be a problem because it's fatty, not because of the red meat content. The Chinese or Mexican food might be fatty as well, but that depends on what it is, which isn't clear from this information. The fruit smoothies would likely be high in vitamin C and therefore good for this client.

The nurse provides an in-service to peers regarding situations that can affect the comfort level of the clients on the unit. Which client statement indicates that the client's sense of well-being is negatively impacted? A) "I feel like I have no energy today." B) "I don't feel any physical pain today." C) "I was able to sleep uninterrupted last night." D) "I am so glad that playing cards takes my mind off my worries."

A) Fatigue is a lack of energy and motivation. A fatigued client is unable to focus on healing and lacks the ability to cope in stressful situations. Restful sleep, physical well-being without pain, and appropriate diversion all promote a sense of comfort for the client.

Which is a characteristic of fibromyalgia? A) Difficult to treat B) Definite as to cause C) Easy to prevent D) Brief in its effects

A) Fibromyalgia is difficult to treat. a combination of pharmacologic and nonpharmacologic therapies is needed to control pain and fatigue. Individuals with fibromyalgia are often referred to a rheumatologist for specialty care. No exact cause for fibromyalgia is known, and it may present without precipitating factors. No means of preventing fibromyalgia is known, and it may only be diagnosed if the pain has lasted at least 3 months.

The nurse is discussing goals to relieve pain and fatigue with a client newly diagnosed with fibromyalgia. Which goal statement would be realistic for this client to achieve within 30 days? A) Cook dinner five nights a week. B) Join an exercise group to meet five nights a week. C) Walk her son to school daily. D) Get a job outside the home.

A) Fibromyalgia reduces the client's energy. The client might set as an initial goal to be able to perform daily tasks for the family such as cooking and doing the laundry. Walking her son to school daily is a bit ambitious to start, as are joining an exercise group and getting a job outside the home.

The nurse is preparing to teach a class on the prevention of constipation. Which food choice will the nurse include as an example of a high-fiber food? A) Raw fruits B) Cooked vegetables C) White bread D) Cooked fruits

A) Foods high in fiber include raw fruits, bran products, and whole grain products. Low-fiber foods would include cooked fruits, cooked vegetables, and white bread.

Why is laparotomy typically the surgery of choice for a perforated appendix? A) Surgeons are better able to remove contaminants from the peritoneal cavity via laparotomy than via laparoscopy. B) Laparotomy requires a smaller incision than laparoscopy and thus involves less blood loss. C) Laparotomy involves a shorter period of postoperative hospitalization than laparoscopy. D) Laparotomy allows for direct visualization of the damaged appendix, whereas laparoscopy does not.

A) Generally speaking, laparoscopy offers several benefits over laparotomy for removal of the appendix. Laparoscopy allows for direct visualization of the appendix without the need for open abdominal surgery, requires a shorter postoperative hospital stay, carries a lower risk of postoperative complications, and allows for more rapid recovery and resumption of normal activities. However, when a client's appendix has burst, laparotomy is usually the procedure of choice, because it allows for removal of contaminants from the peritoneal cavity by irrigation with sterile normal saline

During a home visit with new parents, the nurse learns that a new mother is fatigued because the baby is not sleeping well. Which suggestion could the nurse make that would most help decrease this client's fatigue? A) Advise the client to alternate night feedings with the baby's father to allow each parent to rest. B) Suggest that the client ask the neighbors to babysit one night a week. C) Ask the physician for medication to restore energy. D) Increase exercise time each week to promote energy.

A) Getting up with the newborn causes fatigue over time. If the parents take turns getting up, each parent will get a full night of rest every other day, which should help with fatigue. Medications to restore energy are not appropriate in this case. Asking someone to babysit once a week will not significantly increase rest time and depends on the sustained commitment of neighbors to giving up a night out of every week. These parents need sleep, and increasing exercise is not a long-term solution for sleep deprivation.

A nurse is providing care to a client who is scheduled for a colonoscopy. The client requires a bowel prep prior to the diagnostic test. Which approach should the nurse use to facilitate the client's understanding of the procedure? A) Use layman's terms to explain the procedure, then ask the client to describe the procedure in her own words B) Use medical terminology when explaining the procedure to the client to ensure maximum accuracy and clarity C) Focus on intonation when describing the procedure to the client D) Speak slowly and loudly when providing client teaching about the procedure

A) Good verbal communication incorporates simplicity, brevity, and completeness. Simplicity involves the use of commonly understood words rather than medical terminology. For example, the term "bowel prep" may be completely meaningless to the client, so telling the client that she needs to drink a gallon of laxative-like medication will get the point across better. Asking the client to repeat the information back in her own words gives the nurse a chance to evaluate whether the teaching has been successful. While intonation can modify the feeling and impact of a message, focusing on intonation is less important than using easily understood terminology. Speaking too slowly or too loudly could be interpreted by the client as patronizing or aggressive.

Lab results are back for a client who has limiting joint pain. Synovial fluid analysis shows no uric acid crystals or bacteria. The client asks what the test results mean. How should the nurse respond? A) "These test results mean that your joint pain is likely not caused by gout or septic arthritis." B) "These test results mean that your joint pain is likely not related to any form of arthritis." C) "These test results mean that your joint pain is likely caused by either rheumatoid arthritis or septic arthritis." D) "These test results mean that your joint pain is likely caused by either cancer of the joint or gout."

A) Gout is caused by the collection of uric acid crystals in a joint. The absence of uric acid crystals in the synovial fluid (joint fluid) makes gout unlikely. Septic arthritis is caused by infection, so the absence of bacteria makes sepsis unlikely. However, this test does not rule out osteoarthritis or rheumatoid arthritis, so these are still possible diagnoses. Although the nurse may provide information related to which conditions have been ruled out, providing a medical diagnosis is outside the nurse's scope of practice.

The staff nurses are discussing interventions to reduce the risk of infection for the client population. Which intervention is the most important to decrease client infection? A) Practice appropriate hand hygiene. B) Assess vital signs once daily. C) Raise the temperature in the client's room. D) Wear a mask for all client care.

A) Hand hygiene is always the first and best way to stop the spread of microorganisms, which cause infections. Assessing vital signs is important but should be done more frequently than once daily. Raising the temperature in a client's room would contribute to the growth of microorganisms. Wearing a mask for all clients is not practical and is unnecessary unless a microorganism is airborne and the client is in isolation.

Which of the following lab values is indicative of hypokalemia? A) Serum potassium of 3.25 mEq/L B) Serum potassium of 5.45 mEq/L C) Serum sodium of 125 mEq/L D) Serum sodium of 155 mEq/L

A) Hypokalemia is a deficit in potassium. Normal serum potassium levels range from 3.5 to 5.3 mEq/L, so of the choices listed, only a serum potassium of 3.25 mEq/L would be considered indicative of hypokalemia

A nurse is working in a skilled nursing facility and is performing an assessment on an older adult client. The nurse notes that the client has hypopigmentation of the skin on both hands. The nurse should recognize that this condition is related to which age-related skin change? A) Hyperplasia of melanocytes B) Decreased perfusion of the dermis C) Increased permeability of the epidermal layer D) Hyperplasia of capillaries

A) Hypopigmentation, also known as age spots, is a common finding on the back of the hands of an older adult. Hypopigmentation is caused by hyperplasia of melanocytes. The other findings are incorrect.

A client presents with decreased blood volume, hypotension, tachycardia, and tachypnea during surgery. Which of the following intraoperative complications is most likely? A) Hypovolemia B) Hypervolemia C) Hypokalemia D) Hypernatremia

A) Hypovolemia is an intraoperative complication that presents with decreased fluid (blood) volume, decreased blood pressure, decreased urine output, increased heart rate, and increased respiratory rate. This client is presenting with decreased blood volume, low blood pressure, and fast heart and respiratory rates, and so is probably hypovolemic. Hypervolemia would present with increased fluid volume and blood pressure. This client is not presenting with any of the signs of hypokalemia or hypernatremia.

A client is complaining of pain in the lower-left quadrant of his abdomen. The nurse prepares to auscultate the lower abdomen and notes that the client has a great deal of hair there. Which action by the nurse is appropriate prior to auscultating the client's abdomen? A) Moistening the abdominal hair B) Documenting that the client has hirsutism C) Cutting the client's hair over the entire abdomen D) Discontinuing the use of auscultation and palpating the abdomen only

A) If the client has excess body hair, the nurse should dampen the hair so that it lies flat against the abdomen to enhance sound transmission. The nurse would not shave the client's hair for auscultation. The client complains of abdominal pain, so auscultation would be a necessary part of a thorough examination because the nurse would need to listen to bowel sounds. Hirsutism includes excess hair all over the entire body, not just the abdomen.

A nurse working in the emergency department (ED) notes that a healthcare provider smells strongly of alcohol and appears confused. Which action by the nurse is appropriate? A) Contact the charge nurse to report the problem. B) Tell the healthcare provider to seek alcohol rehabilitation. C) Report the healthcare provider to the hospital CEO. D) Report the healthcare provider to the state licensing board.

A) In a hospital, problems like this one are usually first reported to the charge nurse, then to the unit manager. If the problem is still not resolved, the nurse may approach someone in middle or upper management. Making suggestions to the healthcare provider about rehabilitation does not address the nurse's responsibility to clients and the organization. The nurse should address the problem through the chain of command within the hospital.

The urgent care clinic nurse is treating a client who is experiencing abdominal pain. The client states, "I think I ate tainted food last night." What should the nurse do after the client states that the food was tainted? A) Ask the client open-ended questions to further assess the situation. B) Tell the client the healthcare provider does not need to assess the client. C) Call an ambulance before assessing the client any further. D) Advise the client to take an antacid.

A) In problem solving, the nurse obtains information that clarifies the nature of the problem. This is the first step. The nurse cannot make decisions such as calling an ambulance or telling the client not to see a healthcare provider before knowing all the essential information. The nurse should not give advice about taking a medication.

The home health nurse is talking with the parents outside the bathroom door while their 1-year-old twins are playing in the tub. Which client statement would require further safety teaching? A) "Why don't we talk in the living room?" B) "Let me get the children out of the tub so we can talk." C) "I do not like to leave the children alone in the bathroom." D) "I often bathe the children together."

A) Infants and toddlers are at risk for drowning, even in small amounts of water. The nurse would want to teach the parent that it is never appropriate to leave young children unsupervised in the tub. Taking the children out of the tub and not wanting to leave toddlers alone in the bathroom demonstrates an awareness of risk. There is no risk with bathing the children together.

Because of the way the influenza virus is transmitted from person to person, nurses who are working with clients with influenza should implement A) droplet precautions. B) isolation precautions. C) airborne precautions. D) contact precautions.

A) Influenza is spread through droplets when the client sneezes or coughs. Therefore, droplet precautions should be used. Isolation precautions limit the number of people who come in contact with the client and are used only for severe or life-threatening infections. Airborne precautions are used for tuberculosis and other infections that are airborne. Contact precautions are used when the nurse is at risk of contacting infected body fluids such as stool or wound drainage.

The nurse is planning care for a client with osteoarthritis of the hip. Which intervention would be appropriate for this client? A) Provide moist heat packs to the affected joint 3 times each day. B) Instruct the client on the importance of strict bedrest. C) Provide nonsteroidal anti-inflammatory drugs (NSAIDs) when pain becomes severe. D) Provide opioid pain medication as prescribed.

A) Interventions appropriate for a client with osteoarthritis (OA) include NSAIDs, moist heat, active range-of-motion exercises, proper posture and body mechanics, and assistive devices to safely maintain independence with activities of daily living. Opioid medication is not typically prescribed for the treatment of OA. NSAIDs are most effective if taken before the pain is severe. The client should be encouraged to be mobile, not on strict bedrest.

The nurse is providing discharge teaching for a client diagnosed with a urinary tract infection (UTI). The client is prescribed a 3-day course of oral trimethoprim-sulfamethoxazole (TMP-SMZ). Which client statement indicates that teaching has been effective? A) "I will return within 10 days for a follow-up urine culture." B) "I will practice Kegel exercises on a daily basis." C) "I will increase my intake of fluids, especially citrus juice." D) "I will only wear 100% cotton underwear."

A) It is essential to validate eradication of infection with a follow-up culture that is negative. Doing Kegel exercises and wearing cotton underwear are both useful in prevention of future urinary tract infections, but they are not the best evaluation of effectiveness of teaching. Citrus juices will not increase acidity of urine and therefore are not recommended when a client has a UTI.

The nurse is providing care to a client who is experiencing constipation. The healthcare provider prescribes Metamucil, a bulk-forming laxative. Which is a nursing consideration when administering this medication to the client? A) Offering sufficient water B) Administering rectally C) Using to treat acute constipation D) Assessing for tardive dyskinesia

A) It is imperative that the client take Metamucil with a sufficient amount of water for the medication to be effective. Metamucil is an oral medication, and it is not typically for use in the treatment of acute constipation, as results from the medication are not immediate. Prokinetic drugs such as Reglan may cause tardive dyskinesia. Metamucil is not associated with the cause of tardive dyskinesia

A 13-year-old female client is diagnosed with juvenile primary fibromyalgia syndrome. What should the nurse expect regarding this client? A) The nurse will treat the client in much the same way as an adult with fibromyalgia. B) Chronic fatigue can be ruled out as a likely clinical manifestation. C) The client's pain will likely have lasted no more than 3 months. D) The client's WPI score will be no greater than 7.

A) JPFS is most commonly diagnosed in girls between the ages of 13 and 15. Triggers and symptoms of JPFS are similar to those of adult fibromyalgia, and so the nurse's interventions with this client are likely to pertain to pain management, fatigue reduction, and increased activity tolerance, just as they would for an adult patient. Because this client's symptoms are likely to be the same as those of an adult with fibromyalgia, the client is likely to experience chronic fatigue, pain lasting more than 3 months, and a WPI score of at least 7.

An older adult client is admitted to the medical-surgical unit for a hip fracture. During postoperative recovery, the nurse notices a stage 1 pressure injury forming on the client's sacrum. Which action by the nurse is appropriate to reduce the progression of this injury? A) Maintain the head of the bed at a 30-degree angle, with the client positioned on the right or left side. B) Apply a heat lamp to the area to increase circulation. C) Apply a dry dressing to the pressure injury. D) Maintain the head of the bed at a 45-degree angle

A) Keeping the head of the bed at an angle of 30 degrees or less decreases pressure on the sacrum. An angle of 45 degrees would be too severe and could exacerbate pressure injury formation on the sacrum. Dry dressings are not indicated with this stage of pressure injury. Heat lamps are no longer used in the treatment of pressure injuries because they do not provide therapeutic benefit.

The nurse is caring for a preadolescent male client who is accompanied by his mother. Which statement by the mother would be consistent with the client experiencing growing pains? A) "My son often complains that his arms and legs feel sore." B) "My son seems to get injured very easily, especially broken bones." C) "My son often doesn't want to walk because his knees hurt." D) "My son occasionally complains of pain in his lower back."

A) Long bones of children contain an epiphyseal plate that serves as a location for bone growth. Rapid bone growth in these long bones may produce growing pains as the lengthening bones pull on the muscles. Because this only occurs in the long bones, growing pains are most likely to be felt in the arms and legs. Growing pains would not cause joint pain or lower back pain. Growing pains are also not associated specifically with fractured bones.

Softening of the skin as a result of prolonged wetting or soaking is also referred to as A) maceration. B) debridement. C) excoriation. D) shearing.

A) Maceration involves softening of the skin due to prolonged wetting or soaking. Excoriation is loss of the superficial layers of the skin. Debridement is the removal of necrotic material from a wound. Shearing occurs when one layer of tissue slides over another.

During a health history, a client becomes upset because the nurse is asking many questions. Which response by the nurse is the most appropriate in this situation? A) "I use the answers to determine your current health needs." B) "I am sorry the questions disturb you." C) "I will skip the questions that bother you." D) "I cannot help you if you do not answer me."

A) Many clients may see the health history as an intrusion. The nurse would explain the reason for the questions so the client understands why the questions are being asked. Apologizing to the client is not really appropriate, as the nurse needs information about the client. Skipping questions could cause the nurse to miss areas of client need. Telling the client that the nurse cannot help without answers to the questions is true, but can be seen as a threat and would not be considered a therapeutic response.

The nurse is conducting a prenatal assessment on a client. Which finding indicates a risk of sensory impairment in the unborn child? A) Lack of immunity to rubella B) History of otitis media C) Immunity to varicella D) Brief case of moderate conjunctivitis

A) Maternal infection with rubella during pregnancy can cause vision impairment in newborns. Thus, lack of immunity to rubella indicates an increased risk of sensory impairment in the newborn. Immunity to varicella is a desirable trait that reduces a mother's chance of illness during pregnancy, thus lessening the likelihood of harm to the newborn. Sensory deficits resulting from maternal otitis media would not be hereditary. Conjunctivitis, or pink eye, is an infection that usually clears up on its own and is not congenital.

The nurse educator in a gastrostomy clinic is teaching a group of clients about factors that play a role in the formation of gallstones. Which client would the nurse identify as having the highest risk for gallstone formation due to genetic factors?" A) A Native American client B) An African American client C) An Asian client D) A Norwegian client

A) Native Americans have a higher incidence of gallstones than Asians, Caucasians, and African Americans. This higher incidence is thought to result from a genetic predisposition to secrete high levels of cholesterol in the bile

Which of the following total serum calcium levels would be considered normal in an adult client? A) 9.88 mg/dL B) 2.21 mg/dL C) 4.87 mg/dL D) 7.03 mg/dL

A) Normal total serum calcium levels, which represent both bound and unbound calcium, range from 9 to 11 mg/dL.

The nurse is providing care to newborns in the nursery. When assessing the newborns' urinary output, which does the nurse anticipate as normal daily urinary output? A) 15-60 mL B) 100-300 mL C) 250-450 mL D) 400-500 mL

A) Normal urinary output for the newborn at 1-2 days of age is 15-60 mL per day. Normal urinary output for a newborn 3 to 10 days of age is 100-300 mL per day. The normal output for the newborn at 10 days of age to the infant at 2 months of age is 250-400 mL per day. Normal output for an infant at 2 months of age through 1 year is 400-500 mL per day.

A client is admitted to the emergency department for vomiting and diarrhea that has lasted 4 days. The client's current weight is 154 pounds. The healthcare provider has diagnosed the client with a viral infection. The nurse has been monitoring intravenous fluids and urine output. Which urinary output would indicate that efforts to rehydrate this client have been successful? A) 40 mL per hour B) 20 mL per hour C) 25 mL per hour D) 30 mL per hour

A) Normal urine output for an adult client is at least 0.5 mL/kg per hour. This client weighs 70 kg, so adequate urine output would be at least 35 mL per hour. Thus, the only option that indicates adequate urine output is 40 mL per hour.

When a nurse performs or observes nursing practices that are not safe, the nurse has a responsibility to report those actions. This principle ties the concept of safety to what other nursing concept? A) Accountability B) Advocacy C) Assessment D) Clinical Decision Making

A) Nurses are accountable for their actions, so all unsafe nursing practices should be reported and addressed. This principle does not reflect advocacy, assessment, or clinical decision making.

A nurse has just received the shift report for a 12-hour shift. As the nurse is preparing to enter a client's room, the nurse overhears a coworker telling an offensive joke with a sexual undertone to the client. Which action by the nurse is appropriate? A) Tell the coworker, in private, that such conduct is offensive and not professional. B) Ignore the coworker and walk away. C) Report the incident to the unit manager. D) Ask to be scheduled on a different shift than this coworker.

A) Nurses must develop assertiveness skills to deter sexual harassment in the workplace. Privately telling the coworker to stop, and explaining why, is the first step in putting an end to the situation. Ignoring the coworker's behavior or asking to be scheduled opposite this individual does not address the situation in an assertive manner. Reporting the incident to the nurse manager would be an appropriate second step if the behavior doesn't stop after the nurse's initial discussion with the coworker.

A complaint about unsafe working conditions should be reported to which agency? A) Occupational Safety and Health Administration B) National Institute for Occupational Safety and Health C) State board of nursing D) American Nurses Association

A) OSHA enforces the rights of workers to have a safe work environment. Therefore, complaints about unsafe work conditions should be filed with OSHA. NIOSH conducts research to provide advances in safety. State boards of nursing ensure that nurses are prepared and competent to provide safe nursing care. The American Nurses Association is a professional organization for nurses.

The nurse is caring for a client with osteoarthritis. Which factor in the client's history and physical assessment would the nurse recognize as a risk factor for developing this condition? A) Body mass index of 36.5 B) History of esophageal reflux disease C) Client plays tennis three times each week D) Blood pressure of 136/78 mmHg

A) Obesity increases the risk of developing osteoarthritis (OA), because the added weight increases stress on weight-bearing joints, causing the joints to wear down more quickly. This client has a body mass index of 36.5, which is considered obese. Moderate recreational exercise (such as tennis three times per week) has been shown to decrease the chance of developing OA and slow the progression of manifestations when OA is present. Esophageal reflux is not associated with OA. Blood pressure is not a known risk factor for the development of OA.

The nurse instructs an older adult client with arthritis on the side effects of nonsteroidal anti-inflammatory drug (NSAID) therapy. Which client statement would indicate that this teaching has been effective? A) "I will report any abnormal bruising." B) "Caffeine decreases the effectiveness of the medication." C) "I cannot take other medications while using NSAIDs." D) "If I notice a change in my mood after starting NSAID therapy, I will call the prescriber."

A) Older adult clients are at risk for increased bleeding with NSAID therapy. Thus, the client should be taught to report any abnormal bruising, which may indicate bleeding. Older adult clients often take several medications, and refraining from taking them with NSAIDs is an unrealistic outcome. Mood changes are not a side effect of NSAID therapy. Also, there is no reason for avoiding use of caffeine while using an NSAID

A nurse is caring for a 76-year-old client. The nurse suspects that the client may be minimizing her pain. Which should the nurse recognize as a common reason for this behavior in older adults? A) Older adults see pain as a natural progression of aging, causing them to downplay the extent of their pain. B) Older adults fear that admitting the extent of pain will result in administration of potentially addictive pain killers. C) Older adults think that admitting the extent of their pain increases the likelihood they will be sent to a rehabilitation hospital prior to returning home. D) Older adults have usually lost some degree of sensation in their appendages which makes it difficult for them to sense the full extent of their pain.

A) Older adults see pain as a natural progression of aging. As such, many elderly patients tend to downplay the extent of their pain, which leads to undertreatment of pain. The nurse should be aware when assessing for pain that the older adult may not rate his or her pain realistically. Fear of addictive pain killers, admission to a rehabilitation hospital, and loss of sensation are unlikely to contribute to the older adult downplaying pain.

Which of the following medications may be discontinued in a client who is experiencing delayed wound healing? A) Oral prednisone B) Topical antibiotics C) Topical growth factors D) Oral antibiotics

A) Oral prednisone is a steroid. Steroids are known to interfere with healing, so it is likely that use of these drugs may be discontinued. In contrast, topical and oral antibiotics may be appropriate for clients with delayed wound healing, because they can help prevent infection. Topical growth factors may also be applied to a wound in an attempt to "jump start" the healing process.

Where does perception, or the awareness and interpretation of stimuli, take place? A) The brain B) The nerve receptors C) The peripheral nervous system D) The impulses

A) Perception, or awareness and interpretation of stimuli, takes place in the brain. The nerve receptors convert stimuli to impulses that travel along nerve pathways to the spinal cord or directly to the brain.

An older adult client recovering from prostate surgery is waking up frequently during the night. Which client statement supports the nursing diagnosis Disturbed Sleep Pattern? A) "The pain in my hips is unbearable at times." B) "I walk for half an hour after I eat breakfast." C) "I take my Zoloft as soon as I get up in the morning." D) "I have one cup of regular coffee in the morning."

A) Physical discomfort or pain, especially from osteoarthritis and focused in the hips, often disrupts the sleep of older persons. The client is taking his prescribed antidepressant medication Zoloft (sertraline hydrochloride) appropriately because it has a stimulant effect and should be taken in the morning. A short walk in the morning is an appropriate type and timed exercise. Caffeine intake of one morning cup of coffee should have little interference with sleeping during the night.

A nurse is reprimanded for being habitually late. What action by the nurse would best address this performance issue? A) The nurse must take responsibility and accept any corrective action. B) The nurse must provide proof that all instances of tardiness were unavoidable. C) The nurse must have a positive attitude. D) The nurse must trade shifts in order to be on time.

A) Reliability and accountability are key factors in professionalism. From a systems perspective, each nurse is responsible for completing the duties of the job appropriately so that others can complete their work, too. Almost everyone misses work or arrives late on occasion. However, when poor attendance or lack of punctuality becomes a habit, it also becomes a performance issue and possible grounds for corrective action or termination of employment.

A client in the intensive care unit is combative and pulling at the endotracheal tube, which must remain in place. After exhausting all alternatives, the nurse applies soft restraints to protect the client's airway. Which action should the nurse take next? A) Notify the primary healthcare provider. B) Notify the family of the need for restraints. C) Reassess the need for the restraints in 8 hours. D) Document the application of restraints in the chart.

A) Restraints can only be applied under the order of a physician. When there is an urgency to protect the client and others, restraints can be applied and then the physician should be notified immediately to write an order for the restraints. The nurse would notify the family if present, but that is not the legal priority. The nurse would document the use of restraints as soon as possible after notifying the primary healthcare provider. Most agencies require reassessment of need every 1-2 hours.

The nurse is collecting a health history for a 12-month-old child. The child lives in a home where both parents smoke, and the child has had respiratory syncytial virus twice since birth. The child's older sister was recently diagnosed with asthma. The nurse understands that this child's risk of developing asthma later in life is A) above average. B) average. C) below average. D) well below average.

A) Risk factors for asthma include exposure to air pollution, including secondary smoke from cigarettes, and early exposure to respiratory syncytial virus. Genetic factors may also play a role in asthma development. Because of the presence of these risk factors, this child has an above average risk of developing asthma later in life

Sleep disorders mainly do what with respect to the activities of daily living? A) Interfere B) Decrease C) Change D) Increase

A) Sleep disorders can lead to disrupted activities of daily living, and so primarily interfere with such activities. They do not necessarily decrease these activities, change them, or increase them, but they do make them more difficult to accomplish

The nurse is caring for a client with a history of urinary tract infections (UTIs). Which action by the nurse would decrease the risk of the client experiencing future UTIs? A) Instruct the client to avoid delaying urination. B) Tell the client to increase caffeine in the diet. C) Encourage the client to use the pelvic floor muscles to force urine flow. D) Remind the client to wipe from back to front.

A) Suppressing urination increases the risk of urinary tract infections. The pelvic floor muscles should not be used to force urine flow, and doing so is considered a poor toileting habit. The client should wipe from front to back because wiping from back to front would contaminate the urinary meatus. The client should decrease the use of caffeine in the diet because caffeine is a bladder irritant.

The nurse is caring for a client with gangrene of the toe. Which collaborative intervention should the nurse anticipate preparing the client for? A) Surgery B) Debridement C) Myringotomy D) Wound irrigation

A) Surgical intervention is the primary collaborative treatment for gangrene. Wound irrigation and debridement are used to remove dead tissue and debris from a wound. They are not used for gangrenous infections. Myringotomy is used to remove infected inner ear drainage, not for infected toes

The nurse instructs a client about the medication nifedipine (Procardia) for hypertension. Which client statement indicates that additional teaching is needed? A) "This medication will cause my ankles to swell, which is normal." B) "I need to drink 6 to 8 glasses of water each day." C) "I will call my doctor if I gain weight or become short of breath." D) "I need to eat foods high in fiber when taking this medication."

A) Swelling in the feet or ankles when taking this medication should be reported to the healthcare provider. This medication can cause constipation, so drinking 6—8 glasses of water each day and increasing fiber in the diet are appropriate interventions cited by the client. The client should notify the healthcare provider with weight gain or shortness of breath

A client tells the nurse about having increasing difficulty seeing the print while reading a newspaper. Which tool should the nurse select to assess this client? A) Rosenbaum eye chart B) Penlight C) Cover-uncover test D) Snellen eye chart

A) The Rosenbaum eye chart is used to test for near or reading vision. The Snellen chart is used to test far or distant vision. A penlight is used to test extraocular movements and pupillary response. The cover-uncover test is used to assess for eye muscle strength.

The nurse is providing care for a client admitted during an acute exacerbation of asthma. Which medication does the nurse anticipate to relieve the acute symptoms exhibited by the client? A) Inhaled short-acting beta-agonists B) Oral corticosteroids C) Inhaled long-acting beta-agonists D) Oral anticholinergics

A) The client admitted with an acute exacerbation of asthma will require a rescue medication, such as an inhaled short-acting beta-agonist. Oral corticosteroids, inhaled long-acting beta agonists, and oral anticholinergics are maintenance medications used to treat asthma.

An older adult client is experiencing confusion, a temperature of 101.5°F, bruising to the arms and legs, and decreased urine output. The medical diagnosis is a urinary tract infection. Which is the most appropriate nursing diagnosis for this client? A) Risk for Injury B) Ineffective Breathing Pattern C) Activity Intolerance D) Impaired Memory

A) The client has bruising and is confused. A priority nursing diagnosis would focus on client safety. The bruising and confusion place the client at risk for injury related to falls. There is no evidence that the client is having difficulty breathing. Activity intolerance could be a problem; however, the nurse cannot assume that the client has intolerance to activity due to age. Because the client is exhibiting confusion, which is a typical symptom of urinary infection in the elderly, the nurse would choose risk for injury as a priority over memory because it is not possible to assess the client's memory during confusion.

The nurse is assessing an older adult client who is confused. The client is accompanied by his adult son. Who can the nurse employ as a primary source of data when assessing this client? A) The client himself B) The client's adult son C) A nurse who cares for the client at the retirement home D) The client's primary healthcare provider

A) The client is the only person who is considered a primary source of data. Family members, other support people, health professionals, medical records, laboratory and diagnostic reports, and any other information sources beyond the client himself are considered secondary sources of data.

Which client demonstrates a declining immune response that typically occurs with the aging process? A) An 88-year-old client with pneumonia who has a temperature of 99.5°F B) A 70-year-old client who has swelling and redness around an abdominal incision from an open appendectomy C) A 58-year-old client who complains of redness and itching after developing a rash from contact with poison ivy D) A 56-year-old client who has 8 mm induration at the site of a PPD skin test administered 72 hours earlier

A) The client who has only a slight elevation in temperature in response to pneumonia is exhibiting a decline in the expected immune response. The other clients are demonstrating an expected immune response as evidenced by redness, swelling, and induration.

The nurse is preparing to conduct a physical examination of a client's head and neck area. The client is paralyzed from the neck down. Which action by the nurse is appropriate when conducting the physical assessment of this area? A) Supporting the client during the examination B) Placing the client in an armless regular chair C) Placing the client in Sims position D) Placing the client in supine position

A) The client who is paralyzed from the neck down is not able to support his or her body. When considering positioning, the nurse should recognize that the client will require support during the procedure. Placing the client supine will not allow the nurse full view of the client during observation. This client would not be able to sit in an armless chair, which has no support. Sims position would not allow the nurse to observe the client adequately.

The nurse is providing discharge instructions to a client recovering from cellulitis. Which client statement indicates that this teaching has been effective? A) "I will monitor for signs of infection such as fever, chills, malaise, and redness or tenderness at the site." B) "If the lesion looks healed, I will stop taking the antibiotics so that I will not develop resistance to antibiotics." C) "If pustules develop, I will squeeze the lesion to remove the pus." D) "Drainage from the site is an expected finding, and I should not be concerned."

A) The client will need to be taught to monitor for the signs and symptoms of infection. Infection may be manifested by fever, chills, erythema, tenderness, and drainage at the site, especially if it is cloudy or serous. Changes in the color, amount, and odor from drainage are cause for concern and should be monitored. The physician must be notified if these symptoms occur. Pustules are never to be squeezed open, as there is risk of greater infection. Stopping antibiotics before the doses are complete causes resistance.

The nurse is assessing an older adult client in a long-term care facility after a fall. Which finding requires priority action? A) The injured leg is shortened and externally rotated. B) Redness and severe swelling are found at the hip joint. C) Pain is relieved by moving the affected extremity. D) The client is repeatedly flexing the injured leg at the hip.

A) The client with a fractured hip is often in extreme pain and assumes a position with the leg on the affected side shortened and externally rotated because of gravity and the pull of the muscles. Any movement of the leg on the side of the affected hip is likely to cause severe muscle spasms and further pain. Redness and swelling are the classic signs of inflammation not immediately present after hip fracture. Extreme pain associated with hip fracture prevents any voluntary movement in the leg.

A client is admitted to the hospital with an elevated temperature, nausea, and pain and tenderness in the lower right quadrant of the abdomen. After receiving pain medication, the client continues to complain of pain at a level of 8 on a 0-10 pain scale. The client is not scheduled to receive pain medications for at least another 2 hours. Given these circumstances, which statement by the nurse is most appropriate? A) "I will inform the healthcare provider about your continued pain." B) "I do not have any medications ordered for you at this time." C) "Try to rest for a while longer until it is time to receive your medication" D) "Let's try a heating pad or warm blanket to see if that helps with your discomfort."

A) The client's inability to achieve comfort will need to be reported to the physician. The reported manifestations are consistent with appendicitis, so the client is at risk for perforation, which is manifested by increased pain. The use of heat to manage the pain is contraindicated due to the risk of perforation. Advising the client that no medications are available at this time and encouraging rest do not meet the concerns being presented by the client.

A client with appendicitis is highly agitated and states that she is experiencing a great deal of pain. Which intervention will help decrease this client's anxiety? A) Assess pain levels every 2 hours and administer ordered medication. B) Provide reading material to help distract the client. C) Distract the client with ambulation. D) Administer pain medications when the client complains of pain.

A) The extreme pain caused by appendicitis is the source of the client's anxiety. Assessing the client's pain level every 2 hours and administering medications before the pain gets intense is the best intervention to help decrease the client's anxiety. Waiting until the client complains of pain makes pain relief more difficult and increases anxiety. Distraction does not work when clients have severe pain but is often appropriate for those with chronic pain.

What activity should the nurse implement for a 6-month-old client with gross motor delays? A) Pull the child to a sitting position and prop the child in that position. B) Encourage the child to hold a rattle or play patty-cake. C) Talk to the child and play music. D) Encourage the child to pull up to a standing position.

A) The infant at 6 months should have head control and is working on sitting without support. Pulling the child to a sitting position allows the neck muscles to support the head. Propping the child in a sitting position helps to develop self-righting behaviors. It is too early to worry about standing. Talking to the child promotes language development, not gross motor development. Handling a rattle is a fine motor behavior, not a gross motor behavior.

Hemostasis and phagocytosis are characteristic of which stage of the wound healing process? A) Inflammatory phase B) Proliferative phase C) Granulation phase D) Maturation phase

A) The inflammatory phase of wound healing is initiated immediately after injury and lasts 3-6 days. Two major processes occur during this phase: hemostasis and phagocytosis. The inflammatory phase is followed by the proliferative and maturation phases. There is not a granulation phase of wound healing, although formation of granulation tissue occurs during the proliferative phase.

What should the nurse recommend for a pregnant client who is concerned about a recent flu outbreak? A) The nurse should recommend that the client receive the influenza vaccination. B) The nurse should recommend that the client stay home until the influenza outbreak has ended. C) The nurse should recommend that the client take amantadine (Symmetrel) prophylactically. D) The nurse should recommend that the client eat foods that boost the immune system.

A) The influenza vaccine is the client's best method of preventing influenza infection, and it has no indication of harm to the unborn child. Amantadine is a Category C drug and should not be given during pregnancy to prevent influenza. Although eating foods that boost the immune system is a good recommendation, it will not be as effective at preventing influenza in an exposed individual as the influenza vaccine. Recommending the client stay home for several weeks is not a practical method of prevention.

A nurse is caring for a client with a genetic nerve disorder who has difficulty when attempting to move her tongue. The nurse recognizes that this may indicate a deficit in the functioning of which cranial nerve? A) XII B) XI C) VIII D) VI

A) The movement of the tongue for speech and swallowing is controlled by cranial nerve (CN) XII, hypoglossal. XI CN (accessory) controls the movement of head and neck as well as proprioception. CN VIII (acoustic/vestibulocochlear) controls hearing and the sense of balance. CN VI (abducens) control eyeball movement and moves eye laterally.

A novice nurse on a medical-surgical unit is released from the orientation phase of training. The nurse is able to care for a four- to five-client assignment independently and is assigned a coach on the unit who will help with problem solving if needed. According to Benner's stages of nursing expertise, this nurse would belong in which stage? A) Stage II B) Stage V C) Stage III D) Stage IV

A) The new graduate nurse who is fresh out of the orientation phase is considered to be in Stage II. A Stage II nurse is an advanced beginner, meaning he or she demonstrates marginally acceptable performance. Stage III is the competency stage; the nurse in this stage has 2 or 3 years of experience and demonstrates organizational and planning abilities. Stage IV is the proficiency stage. The Stage IV nurse has 3 to 5 years of experience, has a holistic understanding of the client that improves decision making, and focuses on long-term goals. The nurse at Stage V is considered an expert. His or her performance is fluid, flexible, and highly proficient. The expert nurse no longer requires rules, guidelines, or maxims to connect an understanding of the situation to appropriate action. This individual has highly intuitive and analytic abilities in new situations.

A nurse educator is teaching a group of student nurses about newborn skin and factors that relate to this concept. Which statement will the educator include in the teaching session? A) "The newborn's skin is about 40% to 60% thinner than an adult's skin at birth." B) "The newborn's skin contains less water than an adult's and has tightly attached cells." C) "The newborn's thicker skin decreases absorption of harmful chemical substances and topical medications." D) "The newborn's skin has a greater percentage of underlying subcutaneous fat compared to adults."

A) The newborn's skin is about 40% to 60% thinner than an adult's, which makes the newborn's skin more susceptible to absorption of harmful chemical substances and topical medications. The newborn's skin contains more water than an adult's and has loosely attached cells. The newborn's skin has less subcutaneous fat compared to adults.

The home health nurse is visiting a client who is 2 weeks postoperative from a coronary artery bypass surgery. The client has lost 10 pounds, is continuing to experience pain, and is not eating. What should be the nurse's next action? A) Examine the current interventions for pain relief. B) Refer the client to social services. C) Contact Meals on Wheels so that the client will eat. D) Revise the goals in the current plan of care.

A) The nurse evaluates that pain goals for this client have not been met and examines pain relief interventions to determine the problem. The goal of pain management is pain relief, and that goal would not change; what might change is the interventions to meet the goal. Contacting Meals on Wheels may not be appropriate if the problem is pain relief. Pain relief is a medical issue that is addressed by the nurse and physician, not social services.

The nurse is caring for a client who is experiencing chronic fatigue related to medication being taken for seasonal allergies. What should the nurse anticipate would most likely help this client? A) A medication change to treat seasonal allergies B) Physical therapy to promote exercise C) Acupuncture and massage D) Sleep medication to increase rest time

A) The nurse might expect the provider to try different dosages or different medications to try to relieve the symptoms. Because the fatigue is due to medication side effects, other strategies will not be effective while the client is on the medication. Therapies such as acupuncture and massage are not consistently supported by clinical studies as effective interventions for fatigue.

A client is diagnosed with high blood pressure that is not responding to medications. The nurse suspects renal stenosis. When assessing for this condition, which location will the nurse use for auscultation? A) Renal arteries B) Bladder C) Ureters D) Internal urethral sphincter

A) The nurse should auscultate the renal arteries by placing the bell of the stethoscope lightly in the areas of the renal arteries, located in the left and right upper abdominal quadrants. Systolic bruits ("whooshing" sounds) may indicate renal artery stenosis.

Which diagnostic test should the nurse use to assess hearing in an infant? A) Otoacoustic emissions test B) Weber test C) Rinne test D) Whisper test

A) The otoacoustic emissions test uses an earphone and microphone to play sounds into the ear. Failure to detect an echo indicates hearing loss. This test is almost exclusively performed on infants as part of the routine hearing screening. The Rinne and Weber tests assess bone and air conduction with the use of a tuning fork. The whisper test provides a rough estimate of hearing loss. All three of these other tests involve active participation from the client.

The nurse working in a healthcare setting is charged with inappropriate delegation after asking an unlicensed assistive personnel (UAP) to change the IV bag for a client. To which agency should this action be reported? A) Board of nursing B) Occupational Health Safety Network C) Occupational Health and Safety Administration D) Health Hazard Evaluation Program

A) The state board of nursing has established procedures for reporting errors and violations made by licensed nurses and acts to investigate those reports. Complaints can include unsafe nursing practices, such as inappropriate delegation. The other agencies do not investigate nursing errors.

A client asks the student nurse to explain the pathophysiology of diabetes. The student nurse does not know the answer to this question. What should the student respond to the client? A) "I do not know, but I will find out." B) "You'll have to ask the doctor that question." C) "Why do you need to know that?" D) "I do not know."

A) The student nurse best exhibits integrity and awareness of self-limits by admitting not knowing the answer, but offering to find out and get back to the client. Honesty by the student nurse will build trust with the client. Just saying "I don't know" and leaving it at that will make the nurse appear untrustworthy and possibly uncaring. Questioning the client about the need for the information is defensive and might also be construed as a challenge by the client. Referring the question to the doctor is also not good ethics; nurses need to continue learning throughout their practice.

The nurse is reviewing diagnostic and laboratory studies performed for a client with influenza. Which result should the nurse recognize as being consistent with influenza? A) Decreased white blood cell count B) Increased BUN C) Decreased sodium level D) Fluid-filled lungs on chest x-ray

A) The white blood cell count of a client with influenza will typically be decreased. Laboratory tests for BUN and sodium levels are not usually associated with influenza. Unless the client with influenza develops complications, the chest x-ray is clear.

The nurse is caring for a pediatric client with a surgical wound. The wound is red with purulent drainage and is causing discomfort for the client. Which diagnostic test will determine if the discomfort of the wound is caused by an infection? A) White blood cell count B) Hematocrit measurement C) Urine analysis D) X-rays of the site

A) There are a few tests that can help the medical team determine the source of the client's discomfort. In this case, a white blood cell count will determine if the discomfort is being caused by an infection. An x-ray is useful for determining the existence of physical injuries, not the presence of infection. Urine analysis may indicate illness or malnutrition, whereas hematocrit measurement may identify iron deficiency anemia.

A new nurse decides to join a professional organization that represents his practice specialty. Which type of commitment does this action exemplify? A) Affective B) Normative C) Subjective D) Continuance

A) There are three types of professional commitment: affective, normative, and continuance. Affective commitment is an attachment to a profession and includes identification with and involvement in the profession. This type of commitment develops when involvement in a profession produces a satisfying experience. The nurse manager described in this scenario is demonstrating affective commitment. In comparison, normative commitment is a feeling of obligation to continue in the profession, and continuance commitment is awareness of the costs associated with leaving the profession.

A nurse working in a community health center is counseling an adolescent regarding a suspected eating disorder. The adolescent is of normal weight but admits to periods of overeating, especially when his parents fight. This client's eating habits best demonstrate which risk factor for obesity? A) Cultural and environmental factors B) Heredity C) Low socioeconomic status D) Physical inactivity

A) This client most often overeats because of stress, which is an environmental risk factor for obesity. The client's stress comes from the environment, such as problems at school or at home, and not from heredity, low socioeconomic status, or physical inactivity, although these are all risk factors as well.

A staff nurse serves as a valuable source of expertise and advice to other nurses on the unit. However, the nurse does not have the authority to direct their work. Which characteristic does this nurse have? A) Staff authority B) Responsibility C) Line authority D) Organizational authority

A) This nurse possesses staff authority, or the power to provide advice and support to employees or departments but not to assign tasks. In contrast, line authority is the power to direct the activities of subordinates within an organization. Responsibility means being accountable for meeting personal or organizational objectives and performing required tasks. Organizational authority is not a term used to describe this type of situation.

A client has a wound on the left lateral aspect of the thigh. Which action by the nurse would best promote wound healing for this client? A) Positioning the client to keep weight off the wound B) Positioning the client with weight directly on the wound C) Restricting fluids D) Enforcing strict bedrest

A) To promote wound healing, the client should be positioned to keep pressure off the wound, not directly on it. The client should be assisted in early ambulation, and strict bedrest should not be enforced. Fluid restriction does not encourage wound healing.

An 18-month-old toddler scheduled for routine vaccinations begins to cry when placed on the examination table. The parent attempts to comfort the toddler, but nothing is effective. Which action by the nurse is the most appropriate? A) Allow the toddler to sit on the parent's lap and begin the assessment. B) Allow the toddler to stand on the floor until the crying stops. C) Ask another nurse in the office to hold the toddler because the parent is not able to control the toddler's behavior. D) Instruct the parent to hold the toddler down tightly to complete the examination.

A) Toddlers are most comfortable when sitting with the parents. Vaccinations can be administered in this way if the parent is taught proper therapeutic holding techniques to keep everyone safe for the procedure. Allowing the toddler to stand on the floor or holding the toddler down tightly are inappropriate. A nurse can assist if the parent is unable to hold the child during the vaccinations to prevent injury from movement

An older adult client is diagnosed with cardiomyopathy and a cardiac dysrhythmia. What would the nurse expect to be prescribed for this client? A) Beta blocker B) Digoxin C) Nitrate medications D) Fluids

A) Treatment for cardiomyopathy includes calcium channel blockers, beta blockers, and antiarrhythmics. Digoxin should be avoided because it increases the force of contractions. Nitrates should be avoided because they increase blood pressure. The client should be on a sodium and fluid restriction and not be encouraged to drink fluids.

The nurse is caring for a client in an allergy clinic. After completing the client history, the nurse selects the nursing diagnosis of Risk for Shock. Which item in the client's history supports the need for this nursing diagnosis? A) Anaphylactic reaction to shellfish B) A drug reaction to penicillin causing a rash C) Glomerulonephritis D) Dermatitis resulting from a response to laundry detergent

A) Type I hypersensitivities, such as anaphylactic reactions, occur immediately and may be life-threatening. Because the client has a history of this type of reaction, Risk for Shock is an appropriate nursing diagnosis. The other items would not necessitate the need for this nursing diagnosis

16) Vaccinations can help promote healthy oxygenation by A) reducing the transmission of preventable diseases. B) increasing the exchange of oxygen for carbon dioxide in the lungs. C) promoting adequate blood circulation to organs and tissues. D) preventing all respiratory infections.

A) Vaccinations help decrease the transmission of preventable diseases, many of which are spread by respiratory secretions. Many of these diseases also affect the respiratory system and can alter oxygenation. Vaccinations do not directly increase the exchange of oxygen for carbon dioxide in the lungs, nor do they promote adequate blood circulation. Vaccinations can prevent some respiratory infections, but not all respiratory infections, and they can also prevent some nonrespiratory infections.

The nurse on a medical-surgical unit completes the shift assessment for a client diagnosed with a multisystem fluid volume deficit. In the assessment, the nurse documents that the client is experiencing tachycardia, decreased urine output, and pale, cool skin. Based on this information, which should the nurse anticipate as the cause of the client's current symptoms? A) Natural compensatory mechanisms B) Cardiac failure C) Pharmacological effects of a diuretic D) Rapidly infused intravenous fluids

A) Vasoconstrictive compensatory reactions within the body are responsible for the symptoms exhibited. The body naturally attempts to conserve fluid internally to protect the brain and heart. A diuretic would cause further fluid loss and is contraindicated. Rapidly infused intravenous fluids would not cause a decrease in urine output. Also, the manifestations reported are not indicative of cardiac failure in this client

For a 9-month-old infant, which finding is inconsistent with GERD? A) Weight gain B) Vomiting C) Irritability D) Wheezing

A) Weight gain would be normal and not a clinical manifestation of GERD in a client aged 9 months. Clinical manifestations of GERD in children younger than 1 year may include spitting up, vomiting, irritability, poor weight gain, and arching of the back during feedings. Respiratory symptoms such as choking, wheezing, and coughing may also occur.

Inadequate fluid intake slows the passage of chyme along the intestines. This slowed passage increases the absorption of fluid from the chyme. How does this decreased intake and increased passage time affect the feces expelled from the body? A) It is drier and harder than normal. B) It is more watery and more soft than normal. C) It is more watery and harder than normal. D) It is drier and more soft than normal.

A) When fluid intake is inadequate or output is excessive, the passage of chyme slows and the absorption of fluid increases. The end result is feces that is harder and drier than normal. Watery, soft feces is the result of rapid intestinal transit that leads to inadequate fluid absorption.

The nurse is caring for a client who presents with acute malaise, muscle aches, and fever. Which additional assessment findings should the nurse recognize as consistent with influenza? Select all that apply. A) No history of vaccinations within the past 12 months B) Nonproductive cough C) Hypotension D) Difficulty urinating E) Dizziness

A, B Based on the presenting symptoms, the nurse would ask whether the client has had a seasonal flu shot or recently been exposed to the flu. Usually, the cough of a client with influenza is nonproductive. A productive cough may indicate a different diagnosis. Insufficient voiding, hypotension, and dizziness are not routine manifestations of influenza.

The charge nurse, who is a member of Generation X, is training a new nurse, who is a member of the Millennial generation. Which aspects of the two nurses' work ethics may be in conflict? Select all that apply. A) Preference for self-directed activity versus a need for feedback B) Loyalty to profession versus desire for new challenges and opportunities C) Workaholic orientation versus need for work-life balance D) Respect for authority versus questioning of authority E) Preference for personal forms of communication versus preference for communication via cell phone

A, B Nurses from Generation X tend to seek challenges; are self-directed; are comfortable with technology; expect instant access to information; desire employment in which they can balance work and personal life; prefer managers who function as mentors and coaches; have limited motivation to stay with the same employer but are loyal to their profession; desire more control over their own schedule; and have a pragmatic focus on outcomes rather than process. Nurses from the Millennial generation tend to be social, confident, optimistic, talented, well-educated, collaborative, open minded, and achievement oriented; expect daily feedback; be high maintenance; thrive on the adrenaline rush of new challenges and opportunities; and view personal cell phones as a necessity for daily life and interpersonal communication.

When planning care for a client at risk for developing pressure injuries, which intervention(s) should be included? Select all that apply. A) Initiate a frequent toileting schedule. B) Raise the client's heels off the bed. C) Turn the client every 4 hours. D) Use inflatable doughnut-style devices to reduce pressure on the sacrum. E) Massage pressure areas with lotion every 4 hours.

A, B Urine and feces are destructive to skin. A frequent toileting schedule will reduce periods of incontinence and the potential for skin breakdown. The client's heels should be raised off the bed to remove pressure on this area of the body. The client should be turned at least every 2 hours. Massaging pressure areas can cause friction and damage to problem skin areas. Inflatable doughnut-style devices are contraindicated, because they increase pressure and reduce perfusion to affected areas.

An adult client with a BMI of 39 who smokes 1 pack of cigarettes per day is admitted to the hospital with chest pressure. After myocardial infarction is ruled out, the client is diagnosed with erosive esophagitis through upper GI endoscopy. The client now refuses all medications and states, "I'm not getting hooked on any pills." What would the nurse recommend for the multidisciplinary collaborative plan? Select all that apply. A) Assess the client's readiness for change in smoking cessation and weight loss. B) Interview the client and spouse for a 24-hour recall of usual food content, intake, and meal times. C) Enlist the client's son to elevate the foot of the client's bed at home 6 inches. D) Offer the client a surgical consult to reduce the necessity of medication. E) Omit the pharmacist notification of the Multidisciplinary Team meeting about the client

A, B Weight loss and smoking cessation will improve the symptoms of GERD. Determining food types, amounts, and times of consumption can help the client avoid foods that stimulate acid production and avoid eating prior to lying down. The head of the bed should be elevated and the team should recommend this to the client rather than enlisting the client's son. A surgical consult should come from the primary provider and will not necessarily reduce the need for medication. The pharmacist should be included in the Multidisciplinary Team meeting to give input to strategies to improve the client's receptivity to medication therapy.

A participant in a seminar given by a nurse asks for information about lifestyle situations that might contribute to chronic fatigue. Which examples might the nurse identify in response to this request? Select all that apply. A) Thyroid problems B) Chronic back pain C) Marijuana use D) Vigorous exercise three times a week E) Swimming after a meal

A, B, C Fatigue is a symptom that needs investigating. Some risk factors for fatigue are hyper- or hypothyroidism, use of illicit drugs such as marijuana, and chronic pain. Swimming and vigorous exercise will energize, not produce fatigue

Which behaviors by a new nurse are likely to reduce stress and burnout? Select all that apply. A) Meditate or take a long soak in a tub. B) Join a local aerobics class. C) Participate in a professional organization. D) Refuse to acknowledge personal limitations. E) Don't accept failure; try, try, and try again.

A, B, C Nurses can prevent burnout by using healthy techniques to manage stress. Suggestions include planning daily relaxation activities, establishing a regular exercise program, learning to say no, accepting errors and failures as learning experiences, accepting change and limitations, developing collegial support groups, participating in professional organizations, and seeking counseling if needed.

The nurse is providing discharge instructions to an older adult client who is recovering from a fractured hip. The client is planning to stay with an adult child, who is included in the discharge teaching. Which statements on the part of the client indicate appropriate understanding of the information presented by the nurse? Select all that apply. A) "I have signed a contract with Lifeline." B) "We are removing the area rugs in the hallway." C) "I've borrowed a toilet seat riser from the equipment closet." D) "I will be sure to take oxycodone before I go downstairs in the morning." E) "I can help with housework while I'm staying at my child's house."

A, B, C Statements regarding the use of an emergency alert service and a toilet seat riser indicate appropriate understanding of the information presented. Picking up loose area rugs can help decrease the risk of falls. Pain medication should not be taken when there is a risk of a fall, particularly prior to going down a set of stairs. The nurse should assess the housework that the client wants to help with while living with the adult child. Many housework tasks will be inappropriate.

The nurse admits a client to the medical unit for a urinary disorder. Which questions are appropriate for the nurse to include when assessing the client's voiding pattern? Select all that apply. A) How many times do you urinate in a 24-hour period? B) Has your pattern of urination changed recently? C) How often do you get out of bed at night to urinate? D) What color is your urine? E) Does your urine have any type of odor?

A, B, C When assessing the client's voiding pattern it is appropriate for the nurse to ask how many times the client voids in a 24-hour period; if the pattern of urination has change frequently; and how often the client gets out of bed at night to urinate. Questions regarding the color and odor associated with urine are appropriate when assessing urine characteristics

A client is experiencing severe diarrhea. Which data should indicate to the nurse that the client is experiencing fluid volume​ deficit? (Select all that​ apply.) A. Increased heart rate B. Poor skin turgor C. Orthostatic hypotension D. Increased urine output E. Weight gain

A, B, C ​Rationale: Orthostatic​ hypotension, increased heart​ rate, and poor skin turgor are acute manifestations of fluid volume deficit. Increases in urine output and weight gain are not acute manifestations of fluid volume deficit.

The nurse is providing training for the clinical staff of a skilled care facility that primarily treats elderly clients. The nurse wants to include information on functional incontinence. Which risk factors for functional incontinence will the nurse include in the training? Select all that apply. A) Limited mobility B) Impaired vision C) Lack of access to facilities D) Dementia E) Urinary tract infection

A, B, C, D An immobilized client may experience incontinence if a call light is not within reach; a client with Alzheimer disease, along with other forms of dementia, may perceive the urge to void but be unable to interpret its meaning or respond by seeking a bathroom. A client with impaired vision may not be able to find the bathroom. Minimal facilities can create problems in urinary control. Urinary tract infection is not usually related to functional incontinence.

The family members of a recently deceased client wrote a letter to the unit manager, expressing their appreciation for the way the client was treated while dying in the hospital. The family mentioned characteristics that indicate the nurses were caring. What behaviors did the family most likely use to explain the caring actions of the nurses? Select all that apply. A) Delivered care with style B) Treated the client as a human being C) Respected the client D) Maintained client confidentiality E) Established limits with the client

A, B, C, D Caring has been described as encompassing various intentions and actions. There are 10 behaviors within caring, which include appreciating the client as a human being, showing respect for the client, and treating client information confidentially. Delivering care with style describes aesthetic knowing, which includes the concepts of empathy, holistic thinking, compassion, and sensitivity. Establishing or setting limits is an action that a nurse would perform as part of self-care actions.

Which statement about wound care across the lifespan is correct? A) "When applying transparent dressings on older adult clients, do not hold the skin taut, because doing so can cause damage." B) "In young children, staph bacteria and fungi are the most common causes of infection in minor wounds." C) "Pressure injuries and contact irritation are rare among newborns and infants in NICUs." D) "As compared to younger clients, older adults have a heightened inflammatory response, which can contribute to delayed wound healing."

B) Of these options, the only accurate statement is that the two major infectious agents affecting the skin of children are Staphylococcus and fungi. The rest of the statements are not valid. Specifically, the skin of older adult clients should be held taut when applying transparent dressings; pressure injuries and contact irritation are common among hospitalized newborns and infants; and older adults have a delayed inflammatory response as compared to younger clients.

A client receives the yellow fever vaccine before traveling to the Amazon Basin and asks the nurse how the vaccine provides protection. Which responses by the nurse are most appropriate? Select all that apply. A) "Human macrophages engulf the weakened vaccine virus as if it is dangerous, and antigens stimulate the immune system to attack it." B) "In the lymph nodes, which are part of the lymphoid system, the macrophages present yellow fever antigens to T cells and B cells." C) "A response from yellow fever-specific T cells is activated. B cells secrete yellow fever antibodies." D) "The vaccine contains large amounts of protective antibodies that were produced in another host organism, so it provides immediate protection against yellow fever." E) "The initial weak infection is eliminated and the client is left with a supply of memory T and B cells for future protection against yellow fever."

A, B, C, E Administration of the vaccine introduces a small number of weakened yellow fever viruses into the body. Macrophages ingest these viruses while also presenting the viruses' antigens to T cells and B cells. Exposure to the antigens causes the T cells to attack the yellow fever viruses. It also triggers the B cells to begin secreting yellow fever antibodies, which attack and destroy yellow fever antigens. Thus, the initial weak infection is eliminated and the body is left with a supply of memory T and B cells that provide future protection against the yellow fever virus. In this way, vaccination leads to the development of active immunity, unlike administration of large amounts of antibodies that were produced in another host, which is a type of passive immunity.

Which independent nursing interventions are appropriate for a client who is experiencing an alteration in oxygenation? Select all that apply. A) Encouraging deep breathing exercises B) Assisting with positioning C) Providing suctioning D) Prescribing bronchodilators E) Monitoring activity intolerance

A, B, C, E Examples of independent interventions that nurses can provide to clients with alterations in oxygenation include deep breathing exercises, positioning, encouraging smoking cessation, monitoring activity intolerance, promoting secretion clearance, suctioning, and assisting with activities of daily living (ADLs). It is outside the scope of nursing practice to prescribe a bronchodilator to a client. The nurse, however, can administer a prescribed bronchodilator. This is considered a collaborative nursing intervention

The nurse is designing a teaching plan for community members on ways to prevent chronic pain. Which information should the nurse include in this teaching plan? Select all that apply. A) Eating a healthy diet B) Obtaining adequate sleep C) Avoiding illicit drug use D) Limiting smoking to only before bedtime E) Avoiding repetitive movements

A, B, C, E Lifestyle habits that predispose individuals to chronic health alterations increase an individual's risk for experiencing discomfort. Eating a healthy diet and obtaining adequate sleep can prevent the development of chronic diseases that lead to symptoms of discomfort. Using illicit drugs and smoking can cause emotional and physical withdrawal symptoms when the drug is no longer used. It is wise to not engage in smoking or illicit drug use to prevent the onset of discomfort. Repetitive movements can increase the risk for injury and fatigue, leading to discomfort.

A client is receiving IV antibiotics for the treatment of a Staphylococcus aureus infection. Which nursing interventions are appropriate when providing care to this client? Select all that apply. A) Encourage adequate fluid intake. B) Monitor for allergic reaction. C) Assess renal and liver function. D) Assess pain level. E) Monitor vital signs.

A, B, C, E Nursing interventions to support antibiotic therapy include encouraging adequate fluid intake, monitoring for manifestations of an allergic reaction, assessing renal and hepatic function, and assessing vital signs. Although some clients may experience pain related to staph infection, antibiotics do not address pain, so assessment of pain levels is not related to administration of these medications.

The nurse is working in a primary care setting. Which clients should the nurse identify as being at high risk for influenza or its complications? Select all that apply. A) A 25-year-old pregnant woman at 20 weeks' gestation B) A 65-year-old woman C) A 3-year-old with cystic fibrosis D) A 35-year-old man with a severe allergy to eggs E) A 20-year-old healthcare worker

A, B, C, E People at increased risk of influenza or its complications include infants, young children, pregnant women, and anyone age 50 or older. Patients with chronic disorders, especially diabetes and cardiac, renal, or pulmonary diseases, are more susceptible as well. Healthcare workers have increased risk of exposure to influenza. A man with an allergy to eggs is not at increased risk for influenza or its complications.

A client with osteoarthritis (OA) of the knees and hips returns for a 3-month follow-up visit with the provider. The nurse calculates that the client's body mass index (BMI) is now 22. The client reports starting a water aerobics and running program three times per week. The client is also using hot packs for edema for 20 minutes and cold packs for pain for 40 minutes daily. After evaluating the client's actions, which follow-up interventions should the nurse plan? Select all that apply. A) Reinforce the correct use of hot packs. B) Suggest the client replace running with a lower impact exercise. C) Explain the risk of injury associated with use of cold packs. D) Advise the client to continue weight loss. E) Congratulate the client on starting water aerobics.

A, B, C, E The nurse should congratulate the client on starting water aerobics because it is a low-impact exercise mode. The nurse should also congratulate the client on the weight loss. Note, however, that a BMI of 22 is ideal, so continued weight loss should not be encouraged. The client should be informed that using cold packs for more than 30 minutes may cause skin injury. The nurse should also reinforce that hot packs are used to decrease pain and ice packs are used for edema (swelling). Finally, the nurse should suggest that the client replace the high impact exercise of running with a lower impact exercise such as walking or biking.

The nurse is providing care to a client who is exhibiting clinical manifestations of a severe fluid and electrolyte imbalance. Based on this data, which of the following orders should the nurse anticipate from the healthcare provider? Select all that apply. A) Initiate intravenous therapy. B) Initiate hypodermoclysis. C) Administer antibiotics. D) Administer diuretics. E) Administer red blood cells.

A, B, D If the client is experiencing a fluid volume deficit, intravenous fluids may be ordered if replacement oral fluids cannot be taken in sufficient quantity. Hypodermoclysis, or subcutaneous administration of fluid, may also be employed as a delivery method, especially among older adults. Conversely, if the client is experiencing a fluid volume excess, diuretics may be ordered. Antibiotics are not used for fluid and electrolyte imbalance. Blood transfusion is implemented for blood loss, not for a fluid and electrolyte imbalance.

The nurse is providing care to a client diagnosed with celiac disease who experiences frequent diarrhea. Based on this data, the nurse anticipates the client may also experience which associated problems? Select all that apply. A) Skin breakdown B) Fluid and electrolyte imbalance C) Hair loss D) Lifestyle issues E) Sexual dysfunction

A, B, D Clients with diarrhea may have perianal skin irritation and skin breakdown. Diarrhea disturbs the fluid and electrolyte balance and can disrupt normal life activities. There is no known direct connection between diarrhea and hair loss or sexual dysfunction

Which practices support promotion of health safety? Select all that apply. A) Exercise every day B) Avoid driving when sleepy or tired C) Eliminate all foods containing fat D) Wear seat belts E) Only see healthcare providers when sick

A, B, D Health promotion involves many different practices, including staying physically active, following guidelines for motor vehicle safety, eating an appropriate diet, and monitoring personal health status. Eliminating all foods containing fat would eliminate necessary nutrients from the diet, and clients should see a healthcare provider at least annually for a checkup even if not sick.

Which interventions should the nurse anticipate carrying out to promote effective breathing in a client with respiratory manifestations of the flu? Select all that apply. A) Maintain adequate hydration. B) Teach the client coughing and deep breathing. C) Prepare the client for the possibility of a tracheostomy tube. D) Keep the head of the bed elevated. E) Administer antitussives during the day.

A, B, D Keeping the head of the bed elevated improves lung excursion and reduces the work of breathing. Coughing and deep breathing are essential for achieving airway clearance. Hydration thins the mucus and also aids in clearing the airway. Insertion of a tracheostomy and oxygen are not primary treatments for ineffective airway clearance. Antitussives should be administered at night to promote sleep but should not be administered during the day to promote airway clearance through coughing.

An experienced nurse practitioner is always conscious of the need to maintain a high level of competence within professional nursing practice. Which activities support this nurse's goal? Select all that apply. A) Reading professional journals B) Collaborating with peers C) Counseling clients D) Attending professional workshops and seminars E) Administering medications appropriately

A, B, D Lifelong competence can be promoted by attending seminars offered by colleges and professional organizations, reading professional and peer-reviewed journals, and having formal and informal discussions with peers and other members of the healthcare team. Providing counseling to clients and administering medications are ways to implement nursing knowledge, not methods for increasing it.

Which of the following triggers can stimulate an acute asthma attack? Select all that apply. A) Stress B) Animal dander C) Loud noises D) Exercise E) Bright lights

A, B, D Stress, exercise, and animal dander are all known triggers of asthma. Loud noises may trigger hearing loss or headaches, but they will not trigger asthma. Bright lights are also not known to trigger asthma.

What information should the nurse include when teaching parents of pediatric clients about ways to decrease the spread of influenza? Select all that apply. A) "Cover your cough" education B) An explanation of appropriate hand hygiene C) Methods for safe food preparation and storage D) Where to obtain the influenza vaccine E) The importance of withholding immunizations for children with compromised immune systems

A, B, D Teaching children to wash their hands and to use respiratory etiquette helps control the growth and spread of microorganisms. The influenza vaccine can decrease each child's susceptibility to influenza infection. Teaching parents safe food preparation and storage is another tool to prevent the spread of microorganisms, but is not related to the flu virus. Immunizations should not be withheld from immunocompromised children

The nurse is teaching a group of adolescents at a local high school about skin infections. Which students should the nurse identify as being at increased risk for developing cellulitis? Select all that apply. A) A student who plucks her eyebrows B) A student with diabetes C) A student who is a member of the golf team D) A student who squeezes pimples E) A student who applies moisturizer on a daily basis

A, B, D The student who plucks her eyebrows risks an infected hair follicle, which can lead to cellulitis, as can squeezing pimples. The student with diabetes has a weakened immune system and increased risk for nerve damage and vascular changes in the extremities, which can lead to increased potential for skin injury and cellulitis. Noncontact sports like golf do not pose a risk for cellulitis. Keeping the skin well moisturized can protect against skin breakdown and thus reduce the risk for cellulitis.

The nurse provides education and supportive assistance for the family of a preschool-age client diagnosed with encopresis. Which statement indicates parental understanding of appropriate care? Select all that apply. A) "We will establish a limited schedule of activities that has many breaks to provide opportunities to use the toilet regularly." B) "We will schedule an appointment with a play therapist to help our older child adjust to our new baby." C) "We won't change our child's diet because we were afraid it will be stress provoking." D) "We will work on regular elimination after morning and evening meals." E) "We will continue to punish our child for having accidents as the behavior is learned and attention seeking."

A, B, D The underlying constipation that leads to encopresis may be caused by the stress of a full schedule of activities or other environmental changes (e.g., birth of a sibling). Dietary changes including incorporating high-fiber foods and limiting refined and highly processed foods and dairy products may be helpful. It takes several months for the bowel to be retrained to respond to sphincter stimulation. It is inappropriate to punish the child for having the accidents because they cannot be helped due to the underlying constipation

During a home visit, the nurse is assessing an older adult client. Which assessment findings support the nursing diagnosis Imbalanced Nutrition: Less than Body Requirements? Select all that apply. A) Client reports a problem with dentures slipping while chewing. B) Client complains of occasional dry mouth and problems with feelings of nausea. C) Client's adult children arrive to eat dinner together several times a week. D) Client is prescribed 15 medications. E) Client's Social Security payments have gone down over the last year.

A, B, D, E The improperly fitting dentures are causing a problem with chewing, which could lead to decreased protein and fruit and vegetable intake. Polypharmacy could negatively influence taste, and some medications promote nausea and vomiting, cause dry mouth, and suppress appetite, which could explain the dry mouth and nausea the client reports. Low finances could lead to decreased access to food. However, the client's adult children visiting and sharing a meal with the client would likely improve or support the older client's nutritional status.

Which clients are at the highest risk of being admitted to the emergency department with severe nausea and vomiting? Select all that apply. A) A 47-year-old with a 3-hour history of chest pressure B) A 61-year-old reporting sudden onset of vertigo C) A 72-year-old with an asthma exacerbation D) A 23-year-old who sustained a head injury in a fall E) A 19-year-old who is 6 weeks pregnant

A, B, D, E The vomiting center in the medulla of the brain may be affected by the vestibular system of the ear, acute myocardial infarction, pregnancy, and increased intracranial pressure. An asthma exacerbation is least likely to cause severe nausea and vomiting

The nurse is providing care to a pediatric client diagnosed with celiac disease. Which outcomes can be anticipated when the appropriate steps for managing celiac disease have been implemented? Select all that apply. A) The client is free of abdominal discomfort including bloating, gas, indigestion, nausea, and vomiting. B) The client is able to maintain normal or routine bowel habits. C) The client has diarrhea fewer than 3 days weekly. D) The client is able to maintain adequate nutritional status. E) The client is able to make appropriate menu choices prior to discharge

A, B, D, E When the client with celiac disease is placed on a gluten-free diet, treatment generally is successful, as long as the client avoids gluten totally. Symptoms such as diarrhea and abdominal discomfort should be eliminated and nutritional status should improve.

During a staff meeting, the new nurse manager informs the staff that each nurse will be getting an email account that needs to be checked daily for information from the manager. Which responses to this policy would be expected of a "Generation X" nurse? Select all that apply. A) "Can we access the email account from home?" B) "That sounds like a great idea." C) "I would rather get the information directly from you." D) "I would rather receive the information in a unit newsletter." E) "Can we use this account to email one another?"

A, B, E Both Generation X and Generation Y/Millennial nurses would likely support the new email policy. Given their high level of comfort with technology, nurses in these age groups would also likely ask whether the new email accounts could be accessed from home or used to communicate with other staff members. In contrast, older nurses would likely prefer communication that is personal and/or not attached to technology.

The nurse is teaching an older adult client and caregiver about appropriate ways to decrease the client's risk for falls. Which interventions are appropriate for the nurse to include in this teaching session? Select all that apply. A) Start walking for exercise several times per week. B) Wear sensible shoes with good support when shopping. C) Wear socks when walking in the kitchen. D) Encourage the use of throw rugs throughout the home. E) Make sure hallways and stairways have adequate lighting, even at night.

A, B, E Interventions that are appropriate to decrease this client's risk for falls include wearing sensible shoes with good support when shopping and making sure hallways and stairways have adequate lighting, even at night. A mild to moderate exercise program is also beneficial, as it helps improve balance and strength, thus reducing the likelihood of falls. Nonslip footwear should be encouraged. Throw rugs should be discouraged.

The nurse gives discharge instructions to an adult client who sustained a bicycle fall and underwent open reduction and internal fixation of a fractured hip. After the teaching is complete, which statements by the client indicate appropriate understanding of the information presented? Select all that apply. A) "I will use my abduction pillow while sleeping to maintain proper hip alignment." B) "I will use a high toilet seat to prevent excess flexion of my hip." C) "I only need to use my walker during physical therapy appointments." D) "I will take my prescribed ibuprofen to decrease the risk for deep vein thrombosis." E) "I might experience bruising because of the warfarin I've been prescribed."

A, B, E Statements regarding use of an abduction pillow to maintain proper hip alignment; use of a high toilet seat to prevent excess flexion of the hip; and awareness that warfarin presents an increased risk for bruising all indicate client adequate understanding. The nurse should remind the client to use the walker at all times until told otherwise. The nurse should also explain that warfarin, not ibuprofen, is prescribed to decrease the risk for deep vein thrombosis.

The nurse is caring for a client who is diagnosed with fibromyalgia. Based on this diagnosis, which clinical manifestations might the nurse anticipate for this client? Select all that apply. A) Weakness B) Exhaustion C) Pain in the calves of the leg D) Nausea and vomiting E) Anxiety

A, B, E The client with fibromyalgia may experience weakness from fatigue; exhaustion from nonrestorative sleep, waking up too early, or poor sleep quality; and anxiety and other mood disorders. There is no evidence that pain in the calves, nausea, or vomiting are symptoms of fibromyalgia.

The nurse is performing an assessment on a client with fluid volume excess. Which finding should the nurse identify that supports fluid volume​ excess? (Select all that​ apply.) A. Pitting edema B. Weight gain C. Tenting of skin Thirst Crackles on auscultation

A, B, E ​Rationale: Pitting​ edema, weight​ gain, and crackles in the lungs upon auscultation are indicative of fluid volume excess. Tenting of skin and thirst are found in fluid volume deficit.

The postoperative nurse is planning care for a client recovering from major thoracic surgery. Which nursing diagnoses should the nurse select to plan for this client's immediate care needs? Select all that apply. A) Risk for Impaired Gas Exchange B) Risk for Decreased Cardiac Output C) Deficient Knowledge D) Risk for Imbalanced Nutrition: Less than Body Requirements E) Risk for Imbalanced Fluid Volume

A, B, E Nursing diagnoses appropriate for the immediate postoperative phase include the Risk for Impaired Gas Exchange because of anesthesia medications and hypothermia, the Risk for Decreased Cardiac Output because of anesthesia, and the Risk for Imbalanced Fluid Volume because of blood loss and nothing by mouth status. Deficient Knowledge is a diagnosis more appropriate to the preoperative phase before surgery. There is no Risk for Imbalanced Nutrition: Less than Body Requirements during the immediate postoperative phase.

The nurse is counseling an adult client with fibromyalgia. What are some elements of counseling that can help this client develop effective coping skills? Select all that apply. A) Ask the client about specific stressors in the client's life and how the client handles them. B) Inform the client about what to do as the disease worsens. C) Ask the client about sources of support that the client may be able to rely on. D) Advise the client that a dimension of self-efficacy is independently developing a response to the problems of fibromyalgia. E) Suggest to the client that some symptoms may be psychosomatic.

A, C It helps to identify stressors that make pain and fatigue worse, and then develop strategies to avoid those stressors or to minimize symptoms when those stressors occur. Fibromyalgia is not a progressive disease. Getting appropriate help is important in managing fibromyalgia. Clients should be encouraged to see a fibromyalgia specialist, and self-efficacy means engaging in health-promoting behavior to influence the course of the disease, not developing a treatment plan alone. It is important to validate the client's perceptions.

During a performance appraisal, a unit manager praises a staff nurse on the ability to use presencing when caring for terminally ill clients. What did the nurse demonstrate that would exemplify this caring behavior? Select all that apply. A) Face-to-face discussions with clients B) Instructing clients on how they should perform certain functions C) Sitting quietly with clients D) Validating the client's experience through active listening E) Guaranteeing quality and safety in care

A, C Presencing is a nursing concept that involves the interpersonal arts of perception and communication. Presencing is described as face-to-face discussions, silent immersions, and lingering presence. Instructing clients on how they should perform certain functions empowers the client. Guaranteeing quality and safety in care demonstrates competence. Validating the client's experience through active listening demonstrates compassion.

The nurse working on a pediatric unit is caring for a client newly diagnosed with asthma. Which assessment data indicate impending respiratory failure and the need for immediate intervention? Select all that apply. A) Shallow respirations B) Slightly diminished breath sounds C) Decreased wheezing D) Increased crackles E) Increased respiratory rate

A, C Respiratory status can change rapidly during an acute asthma attack. Slowed, shallow respirations with significantly diminished breath sounds and decreased wheezing may indicate exhaustion and impending respiratory failure. Immediate intervention is necessary. Increased crackles are usually associated with heart failure and are not an indication of respiratory failure. An increased respiratory rate indicates respiratory compromise, but not respiratory failure

A client who is 4 days post-cholecystectomy has T-tube drainage totaling 600 mL in 24 hours. Which actions by the nurse are appropriate based on this data? Select all that apply. A) Notify the healthcare provider. B) Place the client in a supine position. C) Assess drainage characteristics. D) Clamp the tube q 2 hours for 30 minutes. E) Encourage increased fluid intake.

A, C The T-tube may drain 500 mL in the first 24 hours and is expected to decrease steadily thereafter. If there is excessive drainage, as in this scenario, the nurse should further assess the drainage to be able to describe it accurately and notify the healthcare provider immediately. Clamping the tube would be contraindicated. Placing the client in a supine position and encouraging fluid intake are of no help.

The nurse is providing care to an adult client with a long history of chronic obstructive pulmonary disease (COPD). The client is admitted to the intensive care unit with a pneumothorax. Which interventions are appropriate for this client? Select all that apply. A) Elevate head of the bed B) Administer a high rate of oxygen by nasal cannula C) Prepare for a chest tube insertion D) Administer prescribed antihypertensive medications E) Administer intravenous caffeine per order

A, C The nurse providing care to a client with COPD and a pneumothorax would elevate the head of the bed because of the client's dyspnea and orthopnea and prepare for a chest tube insertion. Because clients with COPD have a decreased response to hypercarbia, which stimulates breathing, a high rate of oxygen by nasal cannula is inappropriate. There is no indication that the client is experiencing hypertension. IV caffeine is administered to premature infants as a respiratory stimulant. This intervention is not appropriate for an adult client diagnosed with COPD and a pneumothorax

The nurse is planning care for a young adolescent client diagnosed with asthma. Which evidence-based age-appropriate interventions will the nurse include in the plan of care? Select all that apply. A) Referring to a peer-led support group B) Teaching the parents how to administer maintenance medication prior to teaching the client C) Assessing peer support when planning care D) Collaborating with teachers for support in the school setting E) Telling the client to avoid medication while at school

A, C, D Age-appropriate, evidence-based interventions for a young adolescent client diagnosed with asthma include referral to a peer-led support group, assessing peer support of the client, and collaborating with teachers to ensure the client has the necessary support in the school setting. While it is appropriate to include the parents in the educational process, the client should be taught how to administer medications prior to teaching the parents. Avoiding medication administration while in school could lead to an acute asthma attack

The nurse is providing teaching to parents about urinary tract infections (UTIs) in preschool children. The nurse should inform the parents that which of the following are common symptoms of UTI in this age group? Select all that apply. A) Urinary urgency B) Elevated blood pressure C) Dysuria D) Fever E) Headache

A, C, D Clinical manifestations of a UTI in a preschool-age child include fever, urgency, and dysuria. Headache and elevated blood pressure are not clinical manifestations of a UTI for a preschool-age child.

The nurse is conducting an assessment for a client on a medical-surgical unit. Which findings are indicative of a client who is experiencing tachypnea? Select all that apply. A) Excessive rapid breathing B) Chest pain C) Rapid breathing at rest D) Shallow breathing E) Cyanosis

A, C, D Excessive rapid breathing, rapid breathing at rest, and shallow breathing are all manifestations of tachypnea. Chest pain is a manifestation of a pneumothorax. Cyanosis is a late manifestation of hypoxemia

The nurse is creating a pain management plan using the three-step approach for a client with intractable pain. Which interventions should the nurse include in this plan? Select all that apply. A) Administer a nonopioid analgesic first. B) Administer an opioid analgesic first. C) Administer a nonopioid with an opioid second. D) Administer an opioid analgesic last. E) Administer analgesics upon client request.

A, C, D The first step in the three-step approach to pain management involves administering a nonopioid drug first. If pain is not adequately controlled with this mild intervention, clients should advance to step 2 and receive a mild opioid in combination with the same or a new nonopioid drug. If the client is still experiencing pain, the mild opioid should be replaced with a stronger opioid in step 3. Pain-relieving drugs should be given "by the clock" (every 3-6 hours) rather than on demand to maintain freedom from pain.

The nurse is providing teaching to a client who is about to undergo surgery. When discussing whom the client can expect to see in the operating room suite, which individuals should the nurse include? Select all that apply. A) Surgeon B) Postoperative nurse C) Circulating nurse D) Anesthesiologist E) Social worker

A, C, D The surgeon performs the procedure. The postoperative nurse will provide care to the client after the surgery is completed. The circulating nurse is a perioperative registered nurse who cares for the client during the surgical procedure. The anesthesiologist provides the anesthesia during the surgery and continually monitors the client's physiologic status. The social worker will not be in attendance during the procedure but may become involved in the client's care during the preoperative and postoperative phases

A school-age child, recently diagnosed with celiac disease, is underweight, vitamin deficient, and anemic and experiences frequent diarrhea. In addition to removing gluten from his diet, what other recommendations will the nurse provide for this child and family? Select all that apply. A) Fat restriction B) A high-carbohydrate diet C) Vitamin supplements D) High-calorie diet E) High-protein diet

A, C, D, E A child with celiac disease who is underweight, vitamin-deficient, anemic, and experiencing diarrhea will require a low-fat, high-calorie, high-protein diet. Vitamin supplements are also likely required given that this child is vitamin deficient. In celiac disease, gastrointestinal dysfunction may cause carbohydrates to be incompletely digested, leading to malabsorption and intolerance.

The nurse is preparing an older adult client for surgery. On which topics should the nurse focus when preparing this client's preoperative teaching? Select all that apply. A) Level of hearing B) Amount of anesthesia needed during surgery C) Teaching on deep breathing and coughing D) Plans for discharge care E) Actions to prevent pressure ulcers

A, C, D, E Clients do not need teaching related to intraoperative anesthesia amounts. For the older client, make sure the client can hear the information to be presented or provide information through alternative means. Deep breathing and coughing assist in the prevention of pneumonia and other respiratory conditions related to surgery, and deep breathing and coughing education should start in the preoperative phase. The older client is going to need assistance once discharged and should have the necessary medical equipment such as walkers and raised toilet seats, assistance with transportation, or extended care. The older client could be at risk for pressure ulcer formation because of poor nutritional status, diabetes, cardiovascular illness, or a history of steroid use

Parents of a child diagnosed with celiac disease have requested guidance on how to implement an appropriate diet. In addition to a list of foods to include and exclude, which interventions by the nurse are appropriate? Select all that apply. A) Obtaining a dietary prescription B) Implementing a recommended exercise program C) Training on how to read food labels D) Providing a referral to support groups E) Encouraging the use of a gluten-free cookbook

A, C, D, E Education that should be provided to the child and their family include obtaining a dietary prescription that will enable them to deduct the cost of special ingredients and commercially prepared products as a medical expense; education on how to read a food label; providing referrals to a support group; and encouraging the use of a gluten-free cookbook. Exercise is not considered an intervention that is specifically aimed at treating celiac disease.

The nurse is caring for a client with chronic constipation. Which findings in the client's health history could be the cause of the current constipation? Select all that apply. A) Bedrest B) High-fiber diet C) Low-fiber foods D) Chronic laxative use E) Depression

A, C, D, E Factors that contribute to chronic constipation include lack of activity, such as bedrest; a diet low in fiber; chronic laxative use; and emotional disturbances such as depression. A high-fiber diet is a treatment option for chronic constipation

The nurse is taking the time to reflect on a care situation in which a client sustained a cardiac arrest and died. On which areas should the nurse focus when performing this reflection? Select all that apply. A) Things that could have been done differently B) Gut reactions to the situation C) Things that were done well D) Resources that were used at the time E) Resources that were needed but not available

A, C, D, E Reflection is the action of making sense of occurrences, situations, or decisions by carefully considering the totality of the experience, such as what worked or did not work, what could have been done differently to achieve better outcomes, what was done well, and what necessary resources were available. In order to reflect on an experience, nurses need to learn what to pay attention to or notice. Reflective thinking can change a situation that is obscure, uncertain, and disturbing to one that is clear, understandable, and settled. A "gut reaction" describes intuition, which is not a part of reflective thinking.

A nurse is teaching a mother warning signs and symptoms to watch for in her child, who will be discharged with a full leg cast. Which statements by the mother indicate the need for further instruction? Select all that apply. A) "If her foot turns white and cold, I should call the physical therapist." B) "I can expect that my child will have some pain, but the medicine should help." C) "We can use a blow dryer on warm to help with the itching that my child will experience." D) "We can cut a hole in the cast if my child's foot swells until we get to the doctor's office." E) "It is okay if the plaster cast gets damp as long as I blow dry it."

A, C, D, E The only option that indicates appropriate understanding of cast care is the mother's statement that her child may have pain that will be relieved by medication. All of the other statements indicate a need for further instruction. If the child's foot turns white and cold, the family should contact the physician. Itching may be helped by use of a blow dryer on the cool setting. Holes should not be cut in the cast, and the plaster should stay dry at all times.

The public health nurse is providing community education aimed at promoting nutritional habits that decrease an individual's modifiable risk factors for heart disease. Which topics should the nurse include in this teaching session? Select all that apply. A) Benefits of consuming fruits and vegetables B) Importance of eliminating all fats C) Selecting lean protein sources D) Preparing balanced meals E) Strategies for maintaining recommended daily caloric intake

A, C, D, E The primary modifiable risk factors for nutrition alterations are food choice, portion size, and nutritional intake. Learning about the benefits of consuming fruits and vegetables, how to select lean protein sources, how to prepare balanced meals, and how to maintain the recommended daily caloric intake are all strategies for promoting good nutritional habits that decrease the risk for heart disease. In contrast, eliminating all fats should not be included, because some fats are considered "healthy" and need to be included in a heart-healthy diet.

The nurse is conducting a physical assessment for a client with a compromised immune system. Which actions by the nurse are appropriate? Select all that apply. A) Assessing general appearance B) Recommending increased fluid intake C) Inspecting the mucous membranes of the nose and mouth D) Palpating the cervical lymph nodes E) Checking joint range of motion (ROM), including that of the spine

A, C, D, E The techniques of inspection and palpation are especially important in assessing a client's immune system. The nurse will assess the client's general appearance, inspect the mucous membranes of the nose and mouth for color and condition, palpate the cervical lymph nodes for swelling or tenderness, and check the client's ROM, including that of the spine. Although recommending that the client increase fluid intake may be an appropriate nursing intervention, it is not an action that is conducted during the physical assessment for this client.

The nurse recognizes that which pathophysiologic changes are occurring when caring for the client with respiratory acidosis? Select all that apply. A) Increased CO2 B) Vasoconstriction C) Decreased O2 D) Decreased intracranial pressure (ICP) E) Increased pulse rate

A, C, E Respiratory acidosis is an alteration of acid-base imbalance that is caused by decreased oxygen intake, resulting in an excess of dissolved carbon dioxide (increased CO2). Vasodilatation, not vasoconstriction, occurs as a low pH results in relaxation of vascular smooth muscle by interrupting the normal function of calcium channels. Cerebral vasodilation results in increased intracranial pressure. The pulse rate increases in an attempt to compensate for oxygen deprivation.

While performing nasotracheal suctioning, the nurse notes the older adult client with an alteration in oxygenation is moving the head around and pulling at the nurse's hand to remove the suction catheter. Which actions by the nurse are appropriate? Select all that apply. A) Remove the suction catheter B) Lower the head of the bed C) Decrease the suction pressure D) Apply restraints to the client's arms and legs E) Hyperoxygenate the client

A, C, E The older adult client is demonstrating signs of hypoxemia. The nurse should remove the suction catheter, decrease the suction pressure, and hyperoxygenate the client. Restraining the patient does not address the hypoxemia. The client should be in the Fowler or high-Fowler position.

A postmenopausal client asks the nurse what she can do to prevent fracturing her hips, as her mother and grandmother both experienced this health problem. Which response by the nurse is most appropriate? A) "You should avoid all types of exercise." B) "You should consider a smoking cessation program." C) "You should limit your exposure to the sun." D) "You should use throw rugs throughout your home."

B) One modifiable risk factor for hip fractures is smoking. Women who smoke have a greater risk of fracture because smoking reduces bone density in menopausal and postmenopausal women. The client should not be instructed to avoid exercise; exercise will enhance the client's gait, balance, and musculoskeletal strength. Limiting sun exposure will not impact the client's risk of experiencing a hip fracture. The nurse should not instruct the client to use throw rugs throughout the home because this could cause tripping, leading to a fall.

A pediatric client with a history of anaphylactic hypersensitivity reactions will be discharged with a prescription for an EpiPen. Which statements are appropriate for the nurse to include in the discharge instructions for this client and family? Select all that apply. A) "It is recommended that the child wear a medical alert bracelet." B) "This medication does not come prefilled and must be measured." C) "Keep the medication in the car at all times." D) "Frequently check the expiration date of the medication." E) "Keep the medication in one location that is easy to remember."

A, D An EpiPen is a prefilled syringe-and-needle medication system used to treat an anaphylactic reaction. Because an anaphylactic reaction is a medical emergency, it is essential that the nurse provide thorough teaching regarding use of the EpiPen. The nurse should recommend that the client wear a medical alert bracelet. The medication should not be kept in the car at all times, as it needs to be stored away from high heat and direct sunlight. The client should have multiple EpiPens and they should be kept in multiple areas, not one location. Also, the EpiPens' expiration dates should be checked frequently to ensure accurate strength.

The nurse is teaching a client with cellulitis about ways to promote healing and avoid future infections. Which client statements indicate that the teaching has been effective? Select all that apply. A) "I should use antibiotic soap to cleanse my wound." B) "I must keep my wound completely dry between cleansings." C) "I should contact the doctor if I have a temperature of 99.5°F or higher." D) "I should avoid swimming in lakes when I have a wound." E) "I can stop taking antibiotics when the swelling subsides."

A, D Clients with cellulitis should wash affected areas with antibacterial soap to prevent the spread of infection. They should also avoid swimming in lakes when wounded, as the water may be contaminated and could cause an infection. The healthcare provider should be contacted for a body temperature of 101°F or higher. Antibiotics are taken for the full course ordered, not just until the swelling subsides. Wounds should be kept at a proper moisture level as instructed, because wet or moist wounds heal faster than dry wounds.

A registered nurse (RN) who now works as a nursing supervisor at a local hospital is asked to talk about nursing during career day at a local high school. When explaining to the students why nursing is a profession rather than a job, which criteria should the RN include? Select all that apply. A) Nurses engage in ongoing research. B) Nurses receive high salaries. C) More nurses are needed to meet current and predicted demand. D) Nursing has a service orientation. E) Nurses must have broad general knowledge of a variety of topics.

A, D Several characteristics make nursing a profession rather than a job. For instance, a service orientation differentiates nursing from occupations pursued primarily for profit. Also, as professionals, nurses engage in ongoing research to improve practice and expand the field's body of knowledge. As with other professions, nursing's knowledge base is well defined and specific rather than broad and general. Salary level is not a criterion for a profession, nor is the number of practitioners in the field.

A nurse is caring for a client with severe acute abdominal pain secondary to cholelithiasis. Which nursing actions promote effective pain management? Select all that apply. A) Withhold oral food and fluids. B) Insert nasogastric tube and connect to high suction. C) Educate the client about decreasing protein in the diet, because protein increases gallbladder contractions. D) Administer morphine, meperidine, or another opioid analgesic as ordered. E) Place the patient in supine position to relieve abdominal pain.

A, D The pain associated with cholelithiasis can be severe. Nursing interventions that help promote effective pain management include withholding oral food and fluids and inserting a nasogastric tube connected to low suction if ordered. The nurse should educate the client about decreasing fat in the diet, because fat entering the duodenum initiates gallbladder contractions, causing pain when gallstones are in the ducts. Administering morphine, meperidine, or another opioid analgesic as ordered also aids in pain management. In addition, the nurse should place the patient in Fowler position, not supine, to decrease pressure on the inflamed gallbladder.

A school nurse is providing care to a number of school-age children diagnosed with celiac disease. Which interventions are appropriate for the nurse to implement with this group of students? Select all that apply. A) Teaching about gluten-free food choices B) Emphasizing low-calorie food selections C) Implementing a school-based prevention program to eliminate the disease process D) Labeling gluten-free choices in the school lunch program E) Demonstrating coping strategies for living with celiac disease

A, D, E Appropriate interventions for the school nurse to implement include teaching the children with celiac disease about gluten-free food choices. The nurse would also implement the labeling of gluten-free food choices in the school lunch program to aid the children in choosing foods to eat while at school. School-age clients may also benefit from learning about coping strategies. Food selections should be high in calories and protein. Celiac disease cannot be prevented.

A client recovering from abdominal surgery tells the nurse that "something popped" in his abdominal incision. Upon inspection, the nurse finds that evisceration has occurred. What actions by the nurse are appropriate? Select all that apply. A) Notify the client's surgeon. B) Pack the client's wound with nonadherent gauze. C) Turn the client onto his abdomen. D) Position the client in bed with his knees bent. E) Cover the incision with a large, saline-soaked dressing

A, D, E Evisceration occurs when an abdominal wound opens and the internal viscera protrude through the incision. The nurse should cover the area with a large, saline-soaked dressing to keep the viscera moist. The nurse should also position the client with the knees bent and notify the surgeon. Nothing should be packed into this wound, and the client should not be turned onto his abdomen.

A nurse is working in a summer camp for children. One of the children comes to the clinic with several bee stings. Which clinical manifestations would necessitate injecting the child with epinephrine (EpiPen)? Select all that apply. A) Skin that is cold and clammy to the touch B) Skin that is warm and dry to the touch C) Hyperverbal behavior D) Extreme anxiety and agitation E) Facial swelling

A, D, E General symptoms of shock that would necessitate an epinephrine injection include cold and clammy skin (which is indicative of decreased perfusion), extreme anxiety and agitation, and facial angioedema. Clients who are experiencing shock are unlikely to be hyperverbal due to respiratory symptoms that make breathing and speaking difficult.

The nurse is caring for a client in the emergency department who is suspected of having appendicitis. Based on this data, which orders should the nurse anticipate from the healthcare provider? Select all that apply. A) A cephalosporin antibiotic B) A barium enema C) Regular diet D) Pain medication E) Complete white blood cell count

A, D, E Pain medications will be ordered, along with a cephalosporin, as third-generation cephalosporins are the antibiotics of choice for appendicitis. The provider will also order a complete white blood cell count, which will be elevated if the client has appendicitis. A barium enema would not be ordered, as this could cause perforation of the appendix and bowel. The client should take nothing by mouth, so a regular diet is contraindicated. B) Pain medications will be ordered, along with a cephalosporin, as third-generation cephalosporins are the antibiotics of choice for appendicitis. The provider will also order a complete white blood cell count, which will be elevated if the client has appendicitis. A barium enema would not be ordered, as this could cause perforation of the appendix and bowel. The client should take nothing by mouth, so a regular diet is contraindicated

The nurse is preparing to provide care to a group of clients. On which specific areas should the nurse focus in order to prioritize the clients' care needs? Select all that apply. A) Asking if any clients have complex issues B) Noting number of licensed staff assigned for the shift C) Noting time when the attending physicians make rounds D) Identifying clients with specific medication times E) Noting which clients have particular safety needs

A, D, E Setting priorities for nursing care always begins with assessment. Assessment includes making observations and asking questions to gather information necessary to make decisions. Helpful assessment data include knowing which clients have complex care issues, if any clients have particular medication times, and if any clients have safety issues that should be addressed. The number of licensed staff is not as important as knowing the number of unlicensed assistive personnel to whom the nurse can delegate client care activities. The time when attending physicians make rounds is not usually part of the criteria when prioritizing client care needs.

A nurse enters a client's room to evaluate the response to IV pain medication administered by request 20 minutes earlier. The nurse finds the client in the same position as when the medication was administered. The client states, "I do not want to move." The nurse asks the client to rate the current level of pain. Which aspects of the nursing process do these action represent? Select all that apply. A) Assessment B) Diagnosis C) Planning D) Implementation E) Evaluation

A, D, E The nurse is engaging simultaneously in an assessment, implementation, and evaluation as she (1) observes the client's behavior and asks about her current pain level (assessment), (2) implements an aspect of care by following up on the prior pain administration (implementation), and (3) evaluates the client's response to the earlier pain administration (evaluation). Phases of the nursing process overlap and are not separate entities.

The nurse is completing a physical assessment with a client. On which part of the body should the nurse focus when determining fluid and electrolyte​ status? (Select all that​ apply.) A. Oral Cavity B. Endocrine system C. Ears D. Skin E. Cardiovascular system

A, D, E ​Rationale: Physical assessment for fluid and electrolyte status focuses on the​ skin, oral cavity and mucous​ membranes, eyes, cardiovascular and respiratory​ systems, and neurologic and muscular status. The ears and endocrine system are not a focus of fluid and electrolyte status assessment.

A client is receiving hemodialysis for renal failure. Which clinical information should indicate to the nurse that the client is experiencing an excess in fluid​ volume? (Select all that​ apply.) A. Full and bounding pulse B. Temperature 100.1°F C. Pulse of 62 bpm D. Blood pressure​ 172/90 mmHg E. Weight gain of 2 kg

A, D, E ​Rationale: Pulse​ volume, blood​ pressure, and body weight all increase with fluid volume excess. Temperature and pulse rate both increase with fluid volume deficit

The charge nurse is observing a newly licensed nurse conduct an abdominal assessment on a client admitted with an abdominal mass that is affecting bowel elimination. Which actions by the newly licensed nurse would require the charge nurse to intervene? Select all that apply. A) Performing palpation before auscultation B) Performing auscultation before palpation C) Using inspection, auscultation, percussion, and palpation during the abdominal assessment of the client D) Using only inspection, percussion, and palpation during the abdominal assessment of the client E) Using deep palpation during the assessment process

A, D, E Physical examination of the abdomen in relation to bowel elimination problems includes inspection, auscultation, percussion, and palpation. Auscultation should precede palpation, because palpation can alter peristalsis. Never use deep palpation on a client who has an abdominal mass, renal transplant, or polycystic kidneys, or who is at risk for hemorrhage

The nurse is instructing a client who is prescribed ipratropium bromide (Atrovent) for asthma. Which should be included in this client's teaching? Select all that apply. A) Take no more than the prescribed number of doses each day. B) Rinse the mouth after taking this medication. C) Take on an empty stomach. D) Take with meals or a full glass of water. E) Use hard candy or drink extra fluids to help with a dry mouth.

A, E Appropriate teaching for a client prescribed ipratropium bromide (Atrovent) includes only taking the prescribed number of doses each day to prevent a drug overdose and the use of hard candy or extra fluids to decrease dry mouth. The mouth does not need to be rinsed after taking this medication. This medication does not need to be taken with meals or a full glass of water, or on an empty stomach.

A nurse faculty member is speaking to prospective students of the Bachelor of Science in Nursing (BSN) program at their educational institution. Which of the following reasons should the nurse faculty member cite as a major incentive for students to select a BSN program over an Associate of Science in Nursing (ASN) program? Select all that apply. A) Greater autonomy in the practice setting B) Receipt of a fuller liberal arts education C) Easier transition to graduate school D) Ability to work in critical care areas E) Better opportunity for career advancement

A, E Nurses who hold a BSN enjoy greater autonomy, responsibility, participation in institutional decision making, and career advancement than nurses who hold only an ASN. All RNs, regardless of their education level, can work in critical care areas. There are some institutions that offer RN-to-MSN educational programs, but the ease of transition is not the ultimate incentive, which is career advancement. Having a liberal arts education is also a plus, although not as major an incentive for career advancement.

A student nurse is trying to explain nursing to family members. Which contemporary aspects of nursing might the student nurse include in the explanation? Select all that apply. A) Nursing is a science. B) Nursing is easy. C) Nursing is a new profession. D) Nursing is focused on illness. E) Nursing is holistic.

A, E The American Nurses Association (ANA) recognizes the influence and contribution of the science of caring to nursing philosophy and practice. Nursing is complex and involves the interrelationship among nurses, nursing, the client, the environment, and the intended client outcome. Florence Nightingale defined nursing nearly 150 years ago. The nurse is concerned with both healthy and ill individuals and approaches care holistically, considering its physical, cognitive, psychological, and spiritual elements.

Which interventions should the nurse incorporate into the plan of care for a client diagnosed with influenza? Select all that apply. A) Placing droplet and contact precaution signs on the client's room door B) Placing the client in a negative air flow room C) Placing a ventilator in the room D) Notifying other departments of the diagnosis E) Using appropriate PPE

A, E To prevent the spread of influenza, the client is placed in a private room with signs for droplet and contact precautions. It is important for healthcare workers to use appropriate PPE for these transmission-based precautions. Placing signs on the door is the way to notify other departments of precautions; no additional notification is needed. Negative air flow rooms are for diseases such as chickenpox, measles, and severe acute respiratory syndrome (SARS). There is no indication that this client will need a ventilator.

The nurse reviews intake and output with a graduate nurse. Which statement by the graduate nurse should cause the nurse​ concern? A. "I would not count ice cream as fluid intake because it is frozen" B. "I should document the amount of tube irrigation as intake" C. "I would need to record liquid feces as output" D. "Any time the client vomits, I need to add that number to the output"

A. ​Rationale: Accurate measurement and recording of fluid​ I&O provides important data about the​ client's fluid balance. Ice cream would be considered intake because it is a food that becomes liquid at room temperature. The other answers are appropriate. Other intake includes all oral​ fluids, ice​ chips, IV​ fluids, IV​ medications, tube​ feedings, and catheter or tube irrigants. Output would include urinary​ output, vomitus, liquid​ feces, tube​ drainage, and wound drainage.

The nurse is teaching a client ways to prevent fluid imbalances. Which fluids should the nurse encourage the client to​ avoid? A. Coffee B. Juice C. Water D. Pedialyte

A. ​Rationale: Coffee should be avoided due to its diuretic effects.​ Water, Pedialyte, and juice are acceptable drinks to avoid fluid imbalances

The nurse is teaching a marathon runner about the importance of maintaining fluid and electrolyte balance. Which situation puts runners at a higher risk for fluid and electrolyte​ imbalances? A. The use of electrolyte replacement fluids during a race B. The significant loss of water during a lengthy exercise session C. The increase of protein intake prior to a race D. The additional calcium taken by using calcium tablets to strengthen bones

A. ​Rationale: It is common for athletes to use electrolyte replacement fluids during exercise. The nurse should be sure that the athlete understands that these fluids could alter the delicate balance of individual electrolytes. Supplemental protein and calcium intake do not typically affect fluid and electrolyte balance. Although water is lost during​ sweating, it does not usually create issues during exercise.

During an​ assessment, the nurse becomes concerned that an older adult client is at risk for dehydration. Which of the following assessment findings would cause the nurse to come to this​ conclusion? A. The client reports ingesting two glasses of water each day. B. The​ client's body mass index is 20.5. C. The​ client's blood pressure is​ 140/98 mmHg. D. The client has poor skin turgor.

A. Poor intake of water​ (in this​ case, only two glasses per​ day) could indicate a loss of the thirst​ response, which is a normal​ age-related change that increases the risk for dehydration. Skin turgor is a poor indicator of fluid balance in older adult​ clients, as the​ skin's elasticity naturally decreases with age.

A client is admitted with​ end-stage renal disease and a potassium level of 7.1​ mEq/L. The nurse anticipates which medication to be used to treat the electrolyte​ imbalance? A. Insulin and glucose B. Magnesium C. Lactated Ringer D. Sodium

A. Rationale: A level of 7.1​ mEq/L is a​ critically-high potassium level. Hyperkalemia is manifested by​ tall, peaked T waves and widened​ QRS, dysrhythmias, cardiac​ arrest, nausea and​ vomiting, abdominal​ cramping, diarrhea, and paresthesia. Pharmacologic treatment consists of administration of insulin and​ glucose, administration of calcium​ gluconate, and sodium polystyrene sulfonate​ (Kayexalate) orally or by enema. Diuretics may be indicated if renal excretion is normal. The other options are not appropriate prescriptions for a client experiencing hyperkalemia.

The nurse prepares intravenous fluid for a client. Which mechanism should the nurse recall that represents the movement of fluid across cell membranes from an area of less concentration to an area of higher​ concentration? A. Osmosis B. Filtration C. Diffusion D. Active transport

A. Rationale: Osmosis is the movement of water across cell​ membranes, from the​ less-concentrated solution to the​ more-concentrated solution. Filtration is the process by which fluid and solutes move together across a membrane from one compartment to another. Active transport is a process by which substances move across the cell membrane and must combine with a carrier for​ transportation, requiring metabolic energy. With​ diffusion, the molecules move from a solution of higher concentration to a solution of lower concentration.

The nurse instructs a client with fluid volume excess about dietary choices. Which meal choice should indicate to the nurse that teaching was​ effective? A. Egg whites, turkey bacon, oatmeal, and wheat toast B. Eggs, sausage, grits, and white bread C. Eggs, ham, mixed fruit, and wheat bread D. Egg whites, ham, grits, and white bread

A. ​Rationale: A meal of egg​ whites, turkey​ bacon, oatmeal, and wheat toast is the best choice to decrease the amount of​ sodium, because turkey bacon has the least amount of sodium. Choices that contain​ sausage, bacon, or ham are high in sodium and should be avoided.

A client is experiencing symptoms of severe gastroenteritis. Which intravenous fluid order should the nurse anticipate being prescribed for this​ client? A. Lactated Ringers B. 5% dextrose in​ 0.45% NaCl C. 0.45% NaCl D. 5% dextrose in water

A. ​Rationale: Clients with dehydration secondary to gastroenteritis​ (vomiting and/or​ diarrhea) are experiencing isotonic fluid loss and require isotonic electrolyte​ replacement, which includes either lactated Ringer or normal saline. A solution of​ 5% dextrose in​ 0.45% NaCl,​ 5% dextrose in​ water, and​ 0.45% NaCl is used to treat total body water​ deficits, not isotonic fluid loss.

A nurse is caring for a client who has lost a large percentage of circulating body fluids as a result of excessive diuresis. Which medication would the nurse anticipate this client​ needing? A. Crystalloid B. Diuretic C. Colloid D. Electrolyte supplement

A. ​Rationale: Colloids expand fluid volume by the replacement of proteins or other large molecules. Diuretics are used to promote urine​ output, particularly associated with fluid overload. Electrolyte supplements are used to replace lost electrolytes. Crystalloids contain both electrolytes and other substances that mimic the​ body's extracellular fluid. These medications will assist in the replacement of depleted fluids while promoting urine output.

The nurse is caring for a client admitted for dehydration. Which assessment finding confirms the​ diagnosis? A. Dry, sticky mucous membranes B. Polyuria C. Bradycardia D. Increase in tongue size

A. ​Rationale: Dry, sticky mucous membranes are an assessment finding indicating fluid loss over an extended period of time. The client would have decreased urine​ output, not​ polyuria; tachycardia, not​ bradycardia; and decreased tongue​ size, not increased.

The nurse is assessing a young child in the community clinic. Which sign indicates to the nurse that the child is experiencing mild​ dehydration? A. Restless B. Concentrated urine C. Dry mucous membranes D. Cool, dry skin

A. ​Rationale: Mild dehydration can be difficult to detect in young children because they tend to not show any​ symptoms, though they may be alert or restless. Mucous membranes and skin tends to remain warm and moist and urine does not always appear concentrated.

A pregnant client telephones the clinic for help because of vomiting for over 15 hours and is feeling lightheaded and dizzy. Which advice should the nurse provide to this​ client? A. "Head to the local emergency​ department." B. "Switch from water to ginger ale or ginger tea to prevent​ nausea." C. "Drink 14 cup of oral rehydration fluid every 15-20 minutes until vomiting​ stops." D. "See your healthcare provider for a prescription for an​ antiemetic."

A. ​Rationale: Pregnant clients with hyperemesis gravidarum are at significant risk for electrolyte imbalances. The client should be directed to the emergency department for evaluation and rehydration. Oral rehydration fluids and ginger products will likely not be effective to rehydrate the client. An antiemetic may be effective to minimize​ nausea, but it does not address the immediate issue of the potential electrolyte imbalance.

A client with heart failure has distended neck​ veins, dependent​ edema, and respiratory crackles on assessment. Which prescription should the nurse anticipate being prescribed for this​ client? A. Intravenous Lasix 20 mg now B. Infuse 1000 mL of normal saline C. Continuous EKG monitoring D. Chest​ x-ray

A. ​Rationale: The client is demonstrating signs of fluid volume excess. Treatment of this disorder includes diuretics to remove excess fluid. Normal saline would exacerbate this​ client's health problem. A chest​ x-ray may be required if the diuretic does not help remove excess fluid from the lungs. Continuous EKG monitoring does not address excess fluid.

A client's stroke volume (SV) is 85mL/beat and the heart rate (HR) is 71 beats per minute (bpm). What is the client's cardiac output (CO) rounded to the nearest liter?

Answer: 6 Liters (L) Explanation: CO = SV × HR 85mL = 0.085 L CO = 0.085 × 71 = 6.035 = 6 L

A client with cholelithiasis is in the clinic for a follow-up assessment after hospitalization. What lifestyle modification should the nurse teach the client to decrease the pain associated with the disease process? A) Reduce sodium intake B) Decrease fat consumption C) Increase fluids D) Decrease smoking

B) A client who is experiencing cholelithiasis should be instructed on the relationship between increased fat consumption and the severity of pain associated with cholelithiasis. Although all clients should be instructed to reduce sodium intake, decreasing sodium will not assist in reducing cholelithiasis or its pain. Increasing fluids will not assist in reducing cholelithiasis or its pain. Also, while all clients should be encouraged to cease smoking, smoking has no relationship to cholelithiasis.

A client is admitted to the hospital with a gunshot wound to the leg. Which nursing diagnosis is a priority? A) Situational Low Self-Esteem B) Risk for Infection C) Anxiety D) Ineffective Coping

B) A client with a gunshot wound is at risk for infection because the wound is severe and caused by trauma. The other nursing diagnoses may or may not be appropriate for the client at this time.

Which best describes the effects of the renal system on blood pressure? A) "The release of the catecholamines epinephrine and norepinephrine cause an increase in blood pressure." B) "The release of renin causes an increase in blood pressure." C) "The release of atrial natriuretic peptide (ANP) and brain natriuretic peptide (BNP) cause an increase in blood pressure." D) "The synthesis and release of adrenomedullin causes an increase in blood pressure."

B) A drop in renal perfusion stimulates renin release. Renin converts angiotensinogen to angiotensin I, which is then converted to angiotensin II in the lungs. Angiotensin II is a vasoconstrictor and also promotes sodium and water retention, raising blood pressure. Catecholamines, ANP, BNP, and adrenomedullin do help regulate blood pressure, but they are not released from the kidneys.

The nurse is selecting sensory aids for a client with deficits in hearing and sight. Which aid would address both sensory deficits? A) Adequate room lighting with night lights B) Flashing alarm clock with large numbers C) Amplified telephone D) Large-print reading material

B) A flashing alarm clock would be helpful for a client with a hearing deficit, and a clock with large numbers would be helpful for a client with a sight deficit. Adequate room lighting with night lights and large-print reading material help with a sight deficit but not a hearing deficit, and an amplified telephone helps with a hearing deficit but not a sight deficit.

A prothrombin time (PT) test measures which of the following? A) Time required for the client's blood to clot B) Time required for the client's plasma to clot C) Time required for platelets to effectively stop bleeding D) Time required for a surgical procedure

B) A prothrombin time (PT) test measures the time required for the client's plasma to clot. A partial thromboplastin time (PTT) test measures the time required for the client's blood to clot. A bleeding time test measures the time required for platelets to effectively stop bleeding. No diagnostic test measures the time required for a surgical procedure

The nurse, caring for an older school-age client recovering from an appendectomy, is preparing to help the family ambulate the child for the first time after surgery. Which nonpharmacologic nursing strategy would be most appropriate for this client? A) Placing a warm, moist pack over the site of the incision B) Holding a splint pillow against the abdomen when moving or coughing C) Administering appropriate narcotic analgesics D) Applying an ice pack over the site of the incision

B) A splint pillow placed on the abdomen is a nonpharmacologic strategy to decrease discomfort after an appendectomy. Heat and ice are not used on the incision area, as they can impair the healing process of the wound. Administering a narcotic is considered a pharmacologic nursing strategy

What stage of pressure injury presents as a shallow open ulcer with a viable, moist wound bed that is red or pink? A) Stage 1 B) Stage 2 C) Stage 3 D) Stage 4

B) A stage 2 pressure injury is characterized by partial-thickness skin loss involving the dermis. It presents as a shallow open ulcer with a viable, moist wound bed that is red or pink. Granulation tissue, slough, and eschar are not present. A stage 2 injury may also present as an intact or open serum-filled blister.

The nurse is preparing to assess a client who is experiencing difficulty breathing. Before palpating the client's abdomen, which nursing action is appropriate? A) Administering 10 L of oxygen to the client B) Having the client remain upright C) Placing the client in a modified Sims position D) Asking the client to bend over a table

B) Abdomen palpation is usually done in the supine position, but a client with difficulty breathing would not tolerate that position well. Instead, the nurse should position the client with the head elevated to the point of comfortable breathing to perform the assessment. Having the client lie in a modified Sims position could compromise the client's ability to breathe effectively. A client who is experiencing dyspnea would not be asked to bend over a table. Depending on the client's underlying condition, administering 10 L of oxygen may be excessive.

Which of the following treatment options would least likely be considered for a 71-year-old client with osteoarthritis (OA)? A) Physical therapy B) Administration of nonsteroidal anti-inflammatory drugs (NSAIDs) C) Weekly tai chi sessions D) Administration of narcotics

B) Acetaminophen is a first-line medication for older adults due to its efficacy and safety. Narcotics are a second-line choice, because they are safer than NSAIDs for older adults. Mindfulness exercises and complementary health approaches such as yoga or tai chi may assist older adults in increasing mobility and reducing pain levels. Physical therapy is especially important in older adults to maintain or improve mobility of joint(s).

The nurse is caring for a client who is experiencing acute chest pain that is rated as a 9 on a 0 to 10 pain scale. Based on this data, which medication does the nurse expect to administer? A) Acetaminophen B) Morphine C) Ibuprofen D) Naproxen

B) Acute pain is often treated with an opioid such as morphine. Morphine is often used to treat chest pain that is associated with a myocardial infarction. Acetaminophen, ibuprofen, and naproxen are more appropriate for other types of pain, not acute chest pain.

The nurse is reviewing the admission orders for an older adult client who is being admitted for a hysterectomy. The client, who has been diagnosed as having uterine cancer, has chronic pain caused by arthritis. The healthcare provider has prescribed long-acting oral narcotic medication to be administered every 4 hours. What should the nurse do when providing the medication to the client? A) Administer the medication if the client requests it. B) Administer the medication every 4 hours around the clock. C) Consult the provider to order intravenous pain medication. D) Administer the medication sparingly to avoid narcotic addiction.

B) Administer analgesics as prescribed. Analgesics should be administered around the clock or by self-administration with a patient-controlled analgesic (PCA) pump to keep the pain from becoming severe. In this case, the nurse should follow the prescribed administration plan of every 4 hours around the clock. The client shouldn't need to request the medication, the medication should be administered around the clock as prescribed rather than sparingly, and the nurse should administer the medication orally as prescribed rather than consult the provider for intravenous medication.

While caring for a client with respiratory alkalosis caused by hyperventilation, the nurse decides that having students in the room may increase the client's anxiety. Therefore, he decides to have the students watch the assessment of a different client. Which type of knowledge is the nurse demonstrating when assigning another client for care? A) Empirical B) Aesthetic C) Ethical D) Personal

B) Aesthetic knowledge is the art of nursing and is expressed by nurses in their creativity and style in meeting the needs of clients. This nurse is demonstrating aesthetic knowledge by being sensitive to the client's needs during the assigned shift. Empirical knowledge ranges from factual, observable phenomena to theoretical analysis. Personal knowledge is concerned with the knowing, encountering, and actualizing of the concrete, individual self. Ethical knowing focuses on matters of obligation or what ought to be done, and goes beyond simply following the ethical codes of the discipline.

The nurse is caring for an older adult client with cholecystitis. The client has been admitted to the hospital for diagnostic testing and pain control. Which nursing diagnosis would be of highest priority for this client? A) Anxiety B) Risk for Infection C) Impaired Comfort D) Imbalanced Nutrition: Less than Body Requirements

B) All of these diagnoses are appropriate for the client with gallbladder disease. However, because older adults do not have as effective an immune system as younger clients, the nurse should prioritize care around preventing infection in this client.

An adult decides to return to school to pursue a second career as a nurse. Which degree program is recommended for this learner as entry-level education for professional nursing? A) Associate's degree B) Baccalaureate degree C) Master's degree D) Doctoral degree

B) Although an associate's degree is usually sufficient for nursing licensure, the ANA recommends a bachelor's degree to enter professional practice. Having a bachelor's degree can also lead to more career opportunities, because many magnet hospitals and academic health centers require that their RNs have at least this level of education. A master's degree is usually undertaken to provide specialized nursing education, such as that required to become a nurse practitioner. A doctorate involves advanced training, which may focus on clinical or organizational skills and usually involves research.

The nurse is caring for clients in an assisted living facility. Which resident would the nurse identify as being at the highest risk for the development of fractures from a fall? A) A resident who participates in resistance training exercises three times a week and takes a calcium supplement B) A resident who hikes in the woods once a week and smokes 14 cigarettes per day C) A resident who line dances twice per week and has a glass of wine with dinner D) A resident who teaches yoga four times per week and is lactose intolerant

B) Among older adult clients, smoking is the highest-risk behavior. Although exercise helps prevent fractures, hiking on an uneven surface can be a risk. Resistance training, line dancing, yoga, and taking a calcium supplement all decrease the risk of fracture with a fall. Consuming one glass of wine each day is not a risk factor for fractures from a fall. Lactose intolerance can lower calcium intake, although there are other sources of dietary or supplemental calcium that lactose-intolerant clients can use to reduce their fracture risk.

A client is scheduled for a diagnostic test to determine digestion status. Which test does not require fasting or other preparation? A) Barium swallow B) Amylase C) Endoscopy D) Lipid panel

B) An amylase test does not require any special preparation. A barium swallow, an endoscopy, and a lipid panel require fasting.

A preadolescent client who fell from a balance beam in physical education class injured her ankle. Given this information, which action by the nurse is appropriate? A) Referring the client to physical therapy B) Placing an ice pack on the client's ankle C) Planning for a corticosteroid injection D) Ordering an x-ray of the ankle

B) An appropriate intervention for a client who experiences an ankle injury is placing ice on the ankle to limit swelling. If physical therapy is needed, the referral would be given after the ankle has had time to heal. A corticosteroid injection would be more appropriate for a client with osteoarthritis, not an acute ankle injury. Ordering an x-ray of the ankle is outside the nurse's scope of practice.

The nurse suspects that the client is experiencing a reaction to a specific antigen. Which laboratory result supports the conclusion made by the nurse? A) Indirect Coombs test showing no agglutination B) Patch test with a 1-inch area of erythema C) 2% eosinophils in the WBC count D) Rh antigen test with negative results

B) An area of erythema after a patch test indicates a positive response to a specific antigen. In contrast, an indirect Coombs test detects the presence of circulating antibodies against RBCs; no agglutination is considered a normal finding. Similarly, an eosinophil count of 2% is within the normal range. Finally, an Rh antigen test with a negative result indicates that the client does not carry the antigen; accordingly, this result is not an indicator of a reaction to a specific antigen.

Why are second-generation antihistamines often preferred to first-generation histamines in the treatment of hypersensitivity reactions? A) Second-generation antihistamines are faster acting than first-generation antihistamines. B) Second-generation antihistamines are less likely than first-generation antihistamines to cause drowsiness. C) Second-generation antihistamines are available over the counter, whereas first-generation antihistamines require a prescription. D) Second-generation antihistamines can be administered either orally or parenterally, whereas first-generation antihistamines can only be given via the oral route.

B) An important difference between first- and second-generation antihistamines is that unlike the first-generation drugs, the newer second-generation drugs do not cause drowsiness. Both first- and second-generation antihistamines are available by prescription and over the counter. The preferred route of administration for both first- and second-generation antihistamines is oral, although diphenhydramine (a first-generation drug) and some other medications may be given parenterally. Second-generation antihistamines are not universally faster-acting than their first-generation counterparts.

The x-ray of a client 14 weeks post-ulnar fracture exhibits no callus formation. Based on this data, which collaborative intervention should the nurse anticipate? A) The physical therapist will set up Buck traction. B) The surgeon will schedule a consultation with the client. C) The pharmacist will educate the client on antibiotics. D) The nurse will counsel the client on starting range-of-motion exercise.

B) An ulnar fracture that does not show callus formation after 14 weeks would be classified as experiencing nonunion. Nonunions frequently require surgical correction. Buck traction, antibiotics, and exercise are not indicated for nonunion of a fracture.

An older adult client is admitted to the hospital after a fall. The client is intermittently confused. Based on age and current data, which of the following conditions is the client most at risk for developing? A) Kidney damage B) Dehydration C) Stroke D) Bleeding

B) As an adult ages, the thirst mechanism declines. Also, an altered level of consciousness can increase the risk of dehydration and high serum osmolality. The risks for kidney damage, stroke, and bleeding are not specifically related to aging or fluid and electrolyte issues.

A nurse is providing instruction to the parents of a pediatric client who is experiencing inflammation due to respiratory infection. Which of the following points would be most appropriate for the nurse to include as part of this teaching? A) "Try to keep your child in a supine position to promote more effective oxygenation." B) "Adequate fluid intake is even more important for children with inflammation than for adults with inflammation." C) "If your child seems to be gasping for air, encourage him to tuck his chin against his chest." D) "Rapid heartbeat is an early sign of dehydration and low blood volume in children."

B) As compared to adults, children have a larger tongue relative to the oral cavity, decreased airway muscle tone, a shorter epiglottis, a more anteriorly positioned larynx, a shorter and narrower trachea, and prominent adenoid and lymphoid tissue. These factors increase a child's risk of airway obstruction, especially when the child is supine and/or the neck is hyperflexed. Hence, these positions should be avoided. Also because of children's small size, absolute volumes of fluid loss represent a larger proportion of total body fluid. For this reason, adequate fluid intake is especially important for pediatric clients with inflammation. Finally, whereas adults typically respond to dehydration and hypovolemia with a compensatory increase in heart rate, tachycardia is frequently a late symptom of hypovolemia in children.

The nurse is caring for a client who is 28 weeks pregnant. The client says she has recently begun to experience frequent lower back pain and asks the nurse what can be done to control this pain. What is the nurse's best response? A) "Back pain is common during pregnancy and can usually be managed by taking nonsteroidal anti-inflammatory drugs (NSAIDs)." B) "Let's talk about some postural adjustments that might help alleviate your pain." C) "Back pain during pregnancy is often related to kidney infection. Have you experienced any recent urinary problems, including pain when voiding?" D) "The physician will likely order an x-ray to investigate potential causes of your pain."

B) Back pain is common during pregnancy due to strain on the back from the growing uterus and fetus; abdominal weakness from stretched abdominal muscles; and hormonal changes that loosen the ligaments in the joints of the pelvis. Kidney infection is not a leading cause of back pain in pregnant women. Pregnancy-related back pain is usually managed conservatively. Postural changes or other adaptations can help increase mobility and decrease discomfort. The recommended pain medication is acetaminophen, because NSAIDs are contraindicated during pregnancy. Although diagnostic imaging may be useful, x-rays should be avoided because they deliver ionizing radiation to the fetus.

An older adult client receiving medication for hypertension had a recent fall at home. Which intervention should the nurse include in this client's plan of care? A) Monitor serum sodium levels B) Assess postural blood pressures C) Monitor serum creatinine levels D) Monitor blood pressure every 2 hours

B) Baroreceptors are less efficient with aging. Therefore, orthostatic hypotension is more likely to occur. Also, clients treated for hypertension could have an increase in sensitivity to the medications. Postural blood pressure assessment allows the nurse to prevent orthostatic hypotension and falls. Every 2 hours is too frequent for assessments of a noncritical client. Sodium and creatinine levels assess renal function

The nurse is caring for a 72-year-old client who has presented to the emergency department for the third time in 8 months with acute asthma exacerbations. The client states that he has trouble holding his inhaler, and sometimes he forgets to take his medication. He is also worried because he thinks his new drugs are adversely interacting with medications for his other conditions. What nursing diagnosis is appropriate for this client? A) Deficient Knowledge B) Ineffective Health Management C) Risk for Aspiration D) Ineffective Coping

B) Based on his repeated trips to the emergency department, his reported trouble holding his inhaler, and his inconsistency with taking his medications, an appropriate nursing diagnosis for this client is Ineffective Health Management. The client appears to have adequate knowledge about how to cope with his diseases; he is just unable to follow through with managing his medications at all times. Therefore, Deficient Knowledge and Ineffective Coping are not appropriate diagnoses based on this information. Not enough information is provided to determine if the client is at risk for aspiration.

The mother of three teenagers is diagnosed with fibromyalgia and asks the nurse to how to keep up with all of the children's activities. Which suggestion by the nurse is the most appropriate? A) Attempt to attend all the functions of the children. B) Negotiate with the children to alternate attendance of their functions. C) Avoid attending any afterschool functions for the children. D) Ask the children to limit their activities.

B) Because it is too difficult to attend all of the children's functions, the nurse suggests alternating attendance of the children's functions. In this manner, the client feels that she is partially meeting the needs of each child. Not attending any functions will only add to the client's stress and may worsen symptoms. It is not reasonable for a client with fibromyalgia to try to run the home and attend all of the functions of each child. The children should not have to limit their activities because of the client's illness.

A client has a blood pressure of 142/92 mmHg. Which classification is appropriate for the nurse to use when documenting this data? A) Normal B) Hypertension stage I C) Prehypertension D) Hypertension stage II

B) Blood pressure values in the adult are classified as either normal (<120/<80 mmHg), prehypertension (120-139/80-89), hypertension stage I (140-159/90-99), or hypertension stage II (≥160/≥100).

During an assessment, the nurse learns that a client who is seeking emergency treatment for a headache and nausea works in a mill without air conditioning. The current air temperature outside is 88 degrees, and the client reports drinking water several times throughout the day because of heavy sweating. Based on this data, which instruction is most appropriate for the nurse to give the client? A) "Eat something sweet when drinking water." B) "Eat something salty when drinking water." C) "Double the amount of water you are drinking." D) "Drink juices and carbonated sodas instead of water."

B) Both salt and water are lost through sweating. When only water is replaced, the individual is at risk for salt depletion. Symptoms include fatigue, weakness, headache, and gastrointestinal symptoms such as loss of appetite and nausea. Thus, the client should be instructed to eat something salty when drinking water to help replace the lost sodium. Eating something sweet will not help replace sodium, nor will consuming juices and carbonated sodas. Doubling the amount of water being ingested could lead to hyponatremia and further manifestations.

The nurse instructs a client with asthma on bronchodilator therapy. Which statement indicates client understanding of how the drug works? A) "The medication widens the airways by causing airway muscle contraction." B) "The medication widens the airways by causing airway muscle relaxation." C) "The medication widens the airways by decreasing histamine production." D) "The medication widens the airways by decreasing mucus production."

B) Bronchodilators stimulate bronchiolar smooth muscle relaxation, not contraction. Smooth muscle relaxation increases the diameter of the airway lumen to enhance airflow. Bronchodilators do not decrease the production of mucus or the production of histamine.

The nurse who uses clinical decision making to start CPR on a client is concerned about what other nursing concept? A) Cognition B) Perfusion C) Thermoregulation D) Acid-base balance

B) Cardiopulmonary resuscitation (CPR) is used to help restore circulation to major organs, which is a function of perfusion. Cognition, acid-base balance, and thermoregulation may also benefit from CPR, but this is not the nurse's primary concern when CPR is needed.

Which of the following medications is used to reduce the cholesterol content of gallstones and lead to their gradual dissolution? A) Cholestyramine B) Chenodiol C) Meperidine D) Amoxicillin

B) Chenodiol (Chenix) is administered to reduce the cholesterol content of gallstones and lead to their gradual dissolution. These drugs act by reducing cholesterol production in the liver, thus reducing the cholesterol content of bile. In comparison, cholestyramine (Questran) is administered to relieve jaundice and pruritus related to accumulation of bile salts on the skin; meperidine is given to alleviate pain; and amoxicillin is given to reduce the likelihood of infection.

The nurse is preparing to assess a 1-year-old client for signs of discomfort. When conducting the assessment, which action by the nurse is the most appropriate? A) Asking the client to rate the pain on a scale of 0-10 during the assessment process B) Asking the parent to hold the client in the lap during the assessment process C) Reading a book to the client during the assessment process D) Recommending that the parent leave the room during the assessment process

B) Children may be fearful of physical assessment. To promote comfort, allow the child to sit on the parent's or guardian's lap during the assessment process, rather than asking the parent to leave the room. A numeric pain scale is not appropriate until the client is older; a faces pain scale would be better. Reading a book during the assessment process is not age appropriate.

The nurse is caring for a group of clients with influenza. Which client should the nurse identify as being at greatest risk for complications? A) A 53-year-old woman B) A 72-year-old man C) A 35-year-old woman D) A 12-year-old boy

B) Children under the age of 5 and older adults over the age of 65 are at highest risk of developing complications related to influenza infection. Older adults are more at risk because their immune defenses become weaker with age.

A mother brings in her 6-year-old daughter with signs and symptoms of fever, reduced voiding, uncontrolled voiding, and pain during urination. The daughter is diagnosed with a urinary tract infection. Which nursing outcome is most appropriate for this client? A) The client's bilirubin levels will remain within normal limits. B) The client will report no episodes of enuresis. C) The client will remain afebrile for 12 hours prior to discharge. D) The client will void at least 300 mL of urine over 24 hr.

B) Children who are potty trained often develop enuresis, or involuntary passage of urine, during a urinary tract infection. An appropriate nursing outcome would be that the child would recover enough from the infection that the child no longer experiences enuresis. An outcome related to bilirubin levels would be more appropriate for a neonate, not a school-aged child. The client should be afebrile for 24 hours prior to discharge, not 12 hours. The client should be voiding 500 to 1000 mL of urine in 24 hours, depending on weight. Only voiding 300 mL of urine would be cause for concern.

An adult female client is diagnosed with chronic fatigue syndrome. What statement best accompanies the diagnosis? A) Continued bed rest will relieve the condition. B) The client's physician has ruled out an obvious cause for the fatigue. C) The client is suffering complications from a severe stress reaction. D) The client needs a rigorous exercise regimen.

B) Chronic fatigue syndrome occurs when an individual experiences severe tiredness that lasts more than 6 months, is not caused by a primary condition, and is not relieved by stress reduction. Because this syndrome is not caused by a primary condition and lasts for at least half a year before it is diagnosed, the physician will have ruled out an obvious cause for the fatigue by this point. Bed rest will not relieve the condition, and exercise therapy has produced conflicting results in patients with chronic fatigue syndrome and so should be approached with caution

An older adult client diagnosed with chronic obstructive pulmonary disease (COPD) is scheduled for a total knee replacement. What should the nurse include in this client's plan of care to address the risk of an alteration in tissue integrity? A) Monitor urine output. B) Assess postoperative wound healing. C) Restrict protein intake. D) Expect purulent drainage.

B) Chronic lung disease reduces the amount of oxygen delivered to the tissues, which could delay wound healing. Furthermore, regardless of their chronic disease status, older adults often experience slowed healing as a result of normal cellular and molecular changes. Thus, it is critical that the nurse regularly assess the postoperative wound for healing. The client may or may not need to have urine output monitored. Purulent drainage is a sign of infection and would not be expected. Postoperative clients need adequate protein for wound healing, so protein intake should not be restricted.

The nurse is reviewing information about four clients who are coming in to the office today due to concerns about bowel elimination. Which of these clients is most likely to have a daily stool softener added to their treatment regimen? A) A 3-month-old client who is exclusively breastfed B) A 43-year-old client who takes opioid medication for chronic pain C) A 92-year-old client who experiences frequent leakage of feces from the anus D) A 28-year-old client who is anemic and has blood in the stool

B) Clients taking opioids have an increased risk of developing constipation and may prevent it by taking daily stool softeners. Breastfed infants typically have soft, liquid stools and would not benefit from a stool softener. Leakage of feces from the anus is indicative of bowel incontinence—not constipation—and would not be treated with a stool softener. Anemia and blood in the stool are indicators of potential bowel cancer or other serious conditions; this client would likely undergo testing rather than be prescribed a stool softener

A client who has multiple sclerosis is complaining of fatigue. In recommending an exercise program to this client, what is the most likely factor the nurse will need to consider other than physician approval? A) Complementary dietary recommendations B) The intensity of the workout C) Complementary therapy recommendations D) Pharmacologic therapy

B) Clients who report persistent fatigue should be encouraged to begin a physician-approved mild to moderate exercise regimen. Higher intensity exercise does not appear to produce a greater reduction in fatigue, so clients should be instructed to avoid intense workouts that may increase feelings of fatigue. Exercise reduces fatigue in chronic conditions such as multiple sclerosis.

During which stage of commitment do nursing students tend to focus on the negative elements of nursing and weigh their ability to handle those elements? A) Passionate stage B) Testing stage C) Exploratory stage D) Quiet-and-bored stage

B) Commitment to a profession develops in five stages: exploratory, testing, passionate, quiet and bored, and integrated. The testing stage is the period in which individuals discover negative elements of the profession. During this stage, individuals start to assess their willingness and ability to deal with those negative elements.

When a client's or family's wishes about a client's care clash with what the nurse believes would be the best possible care for the client, this could cause a conflict between caring interventions and what other nursing concept? A) Accountability B) Ethics C) Quality Improvement D) Communication

B) Conflict between providing the best possible care for the client and the family's or client's wishes could lead to the nurse experiencing moral distress, which could lead to burnout. This is an ethical conflict for the nurse. Even in the midst of this conflict, the nurse should maintain the highest standards of accountability and communication. The nurse may use this situation to analyze institutional policies for quality improvement, but this process would not involve conflict.

A home health nurse is precepting a new nurse during a routine wound care visit. The new nurse is assessing the client's wound and notes that it is showing signs and symptoms of infection. The client's spouse asks the new nurse how the wound looks. The new nurse responds by stating, "It looks fine," but the new nurse's face indicates a different story. When evaluating the new nurse, the preceptor should note a need to work on which aspect of communication? A) Credibility B) Congruence C) Timing D) Clarity and brevity

B) Congruence is adjusting one's tone of speech and facial expression to match one's spoken message. If a nurse's facial expression and words are not congruent, the client and family will often suspect that something is wrong. Credibility is the quality of being truthful, trustworthy, and reliable. Although the nurse's words in this situation may not be completely true, they do not necessarily reflect a larger problem with credibility. Timing means that a message is delivered when the client and family are capable of processing it fully and correctly. Clarity and brevity are characteristics involving preciseness and use of few words.

A nurse is teaching environmental control to the parents of a child with asthma. Which statement by the parents indicates effective teaching? A) "We'll be sure to use the fireplace often to keep the house warm in the winter." B) "We will replace the carpet in our child's bedroom with tile." C) "We'll keep the plants in our child's room dusted." D) "We're glad the dog can continue to sleep in our child's room."

B) Control of dust in the child's bedroom is an important aspect of environmental control for asthma management, and replacing the carpeting in the child's bedroom with tile flooring will reduce dust. When possible, pets and plants should not be kept in the home. Smoke from fireplaces should be eliminated.

What word best describes fatigue from culture shock? A) Linguistic B) Temporary C) Geographical D) Mild

B) Culture shock involves adjustment to changes in culture, and so should be temporary in most cases. Differences between cultures may include a number of small changes, which build over time to cause physical and mental fatigue that should decrease the longer the individual lives within the new culture. Culture shock may involve a reaction to changes in geography or language, but not necessarily, and it may be mild or more severe.

A client has an excoriated skin area with purulent drainage. Which diagnostic test does the nurse anticipate being ordered? A) Skin biopsy B) Culture C) Wood's lamp D) Patch test

B) Cultures to identify infections may be conducted on tissue samples, on drainage and exudates from lesions, and on serum. Skin biopsies are used to differentiate a benign skin lesion from a skin cancer. A Wood's lamp is used to identify infections through immunofluorescent studies. Patch tests are used to determine allergies

A nurse educator is talking to a group of staff nurses about the importance of continued competence in nursing practice. One of the staff nurses asks about activities that can help professional nurses maintain competence. Which action should the nurse educator recommend? A) Working overtime whenever hours are available B) Designing a poster presentation on current research on care for the dying client C) Volunteering to take blood pressures at a health and wellness fair D) Organizing a seminar to educate new nurses about hospital policies

B) Designing a poster presentation on current research involves gathering information on a nursing topic. The nurse may be preparing to teach others, but in doing so, he or she is gaining more knowledge and furthering personal competence. Working overtime does not necessarily contribute to continued competence and may not be beneficial for the nurse or clients if the nurse experiences burnout. Volunteering to take blood pressures will not further the nurse's competence. Organizing an educational seminar involves scheduling, recruiting speakers, and arranging for a location, but it does not contribute to continued competence; however, attending such a seminar may

Which agent can be used to destroy pathogens other than spores? A) Antiseptic B) Disinfectant C) Sterilizing agent D) Isolating agent

B) Disinfectants destroy pathogens other than spores. Antiseptics only inhibit the growth of some organisms. A sterilizing agent destroys all pathogens, including spores. Isolation is used to prevent the spread of infection but does not destroy any pathogens.

The nurse is caring for a client who has been diagnosed with orbital cellulitis. Which assessment finding should the nurse anticipate? A) Sunken eyes B) Edema of the affected eye C) Increased acuity of the affected eye D) Elevated blood pressure

B) Edema of the affected site is a common symptom of cellulitis. This may cause bulging eyes, not sunken eyes. The client may complain of decreased vision, not increased acuity of vision. Blood pressure is unrelated to orbital cellulitis.

In holistic nursing, the nurse should emphasize the client's personal responsibility in maintaining health. This idea is most closely related to which concept in caring interventions? A) Nursing presence B) Empowerment C) Compassion D) Competence

B) Empowerment is the process whereby the client develops the autonomy to identify her own health needs in lieu of being instructed how to do so. This helps the client take personal responsibility in maintaining health. Nursing presence, compassion, and competence are less likely to help clients take personal responsibility for their own health.

An older adult client with bilateral osteoarthritis of the knees tells the nurse, "I know I need to lose weight, but exercising makes my knees ache." What instruction should the nurse provide to this client? A) "You should discuss knee replacement surgery with your physician." B) "Exercising the muscles in your legs might be hard now, but over time, it will help protect your knees." C) "Try eating a reduced-calorie diet for several months before attempting exercise." D) "You need to stretch your muscles, because stretching is the only form of exercise that improves osteoarthritis."

B) Encouraging exercise is an important aspect of nursing care for clients with osteoarthritis (OA). Exercise can increase flexibility, improve blood flow, and help clients lose weight. Over time, these factors can help protect the joints against further deterioration and pain. The nurse should not counsel the client to follow a reduced-calorie eating plan for several months before attempting exercise. The client may or may not want to have knee replacement surgery. Stretching is just one type of exercise that will benefit clients with OA. The other components, strengthening and aerobic exercise, can be obtained through walking, swimming, and isometric, isotonic, and resistive exercises

On which region of the body would the nurse most expect to observe erysipelas? A) Abdomen B) Ankles C) Neck D) Back

B) Erysipelas, a superficial cellulitis of the skin caused by group A streptococcus, usually affects the lower extremities or the face. The involved area is bright red and raised with well-defined borders.

The nurse is assessing a client with a surgical wound. Which finding indicates that care has been effective for this client? A) The client's temperature is 100°F. B) The client performs wound care independently. C) There is only a scant amount of purulent drainage on the dressing. D) A small area of erythema and edema is present.

B) Evidence of effective care for a client with a surgical wound includes the client performing wound care independently. Purulent drainage and an elevated temperature could mean the wound is infected. Erythema and edema could indicate the wound is inflamed or infected.

The nurse is evaluating care provided to a client with osteoarthritis (OA). Which client statement indicates to the nurse that interventions for OA have been successful? A) "I had to take early retirement and now stay at home all day and rest my legs." B) "I am sleeping throughout the night and have not missed any work because of knee pain." C) "I am moving from my two-story house into the first floor of my daughter's home so I won't have to walk steps anymore." D) "I changed my work hours so now I work part time and have a nursing assistant who helps me bathe twice a week at home."

B) Expected outcomes for the care of a client with OA include independence with activities of daily living, minimal lifestyle impact because of OA, and controlled pain that allows for rest and sleep. Of the client statements provided, only the one about improved sleep and pain not interfering with work indicates achievement of these outcomes. A client who changes work hours and has a nursing assistant for bathing is experiencing a reduction in activities of daily living and a significant impact in lifestyle. A client who is moving in with a daughter is experiencing significant lifestyle impact. A client who retires early and stays at home all day is also experiencing a significant impact in lifestyle.

Which of the following fractures presents the greatest risk for development of fat embolism syndrome? A) Open fracture of the fibula B) Closed fracture of the femur C) Open fracture of the humerus D) Closed fracture of the clavicle

B) Fat embolism syndrome may occur in conjunction with closed fractures of the long bones or pelvis. Of the closed fractures listed here, only the fracture of the femur involves a long bone, so this is the injury that presents the greatest risk for development of fat embolism syndrome.

The nurse is caring for an older adult client who is receiving intravenous fluids at 150 mL/hr. Upon assessment, the nurse notes crackles, shortness of breath, and jugular vein distention. Based on this data, which complication of IV fluid therapy does the nurse anticipate? A) Speed shock B) Fluid volume excess C) Pulmonary embolism D) An allergic reaction

B) Fluid volume excess may occur if clients, especially the very young or old, receive IV fluid rapidly. The findings given in this scenario do not support the other options.

Which of the following statements best describes the therapeutic approach to acute and chronic pain, fatigue, fibromyalgia, and sleep disorders? A) Therapy is primarily psychosocial in nature. B) Therapy involves both pharmacologic and nonpharmacologic approaches. C) Therapy is essentially physiologically focused. D) Therapy mostly involves the client avoiding risk behaviors.

B) For all of these conditions, therapy involves both pharmacologic and nonpharmacologic approaches. Therapy for these conditions is both physiological and psychosocial, addressing all components of client's conditions. Therapy involves treatment of existing conditions, not just risk prevention measures on the client's part

The nurse is providing care to a 12-year-old child with special needs and his caregiver. What strategies should the nurse help the caregiver teach the child to improve the child's safety? A) Teach the child to schedule routine immunizations B) Teach the child how to use a telephone to call for help C) Teach the child to maintain airway with suctioning D) Teach the child to avoid secondhand smoke exposure

B) For children with special needs, the caregiver can work with the child to teach the child how to use a telephone to call for help when needed. The other actions are typically the responsibility of the caregiver or require the caregiver's help, including scheduling routine immunizations, keeping the child away from secondhand smoke, and suctioning the airway.

A client is experiencing visual overstimulation. What can the nurse do immediately to reduce this client's visual sensory overload? A) Suggest the client wear sunglasses that block UVA rays only. B) Reduce the amount of light in the room by lowering shades and turning off overhead lights. C) Provide the client with large-print reading materials. D) Encourage the patient to employ relaxation techniques to reduce anxiety and stress.

B) For clients who are at risk of overstimulation, nurses should assist with reducing the number and types of environmental stimuli. Appropriate measures for addressing visual overstimulation include lowering the shades and turning off overhead lights. Dark glasses that block both UVA and UVB rays are also useful. Relaxation techniques would be good for the client to employ but would require teaching for the client to properly implement them. Large-print reading materials would be helpful for a client with visual impairment but not a client who is experiencing visual sensory overload.

The nurse in the clinic is assessing an adult client who has signs and symptoms of heart failure. Which of the following lifestyle habits would be useful for the nurse to assess before developing the client teaching plan? A) The client's occupation B) The client's diet C) The client's usual sleep schedule D) The client's marital status

B) For clients who have heart failure, the nurse should ask questions aimed at obtaining information about lifestyle habits that may be contributing to the heart failure, such as smoking and diet. Although sleep schedule is a lifestyle habit about which the nurse should inquire, it is less likely than diet to be a contributing factor to heart failure. Gathering psychosocial information such as the client's marital status and occupation is also important, but in this case, it is not directly related to the client's current problem and teaching needs.

A nurse in a rural community is employed in a facility that has had a shortage of nurses for several years. As a result, several nurses have left the institution citing burnout. To avoid risking burnout, the nurse regularly works out, practices yoga, socializes with friends once or twice a week, and participates in at least one annual national or state nursing conference. This approach to work-life balance reflects which concept within the framework of Caring Interventions? A) Self-compassion B) Self-care C) Self-control D) Self-actualizing

B) Given the circumstances in which the nurse finds himself, the nurse has initiated the Caring Intervention of self-care to help him better cope with a stressful work environment. Although self-care involves aspects of having compassion for the self, exercising self-control, and being self-actualizing, self-care is the term included as part of the Caring Interventions framework that enhances nurses' professional practice and helps them to avoid burnout.

________ establishes and maintains the social, political, and economic arrangements that give professionals the means to control their professional affairs. A) Autonomy B) Governance C) Socialization D) Accountability

B) Governance establishes and maintains the social, political, and economic arrangements that give professionals the means to control their professional affairs. In contrast, accountability involves being answerable for the outcomes of a task or assignment; autonomy refers to a profession's ability to regulate itself and set standards of practice for its members; and socialization is the process by which individuals learn how to behave, feel, and see the world as members of their chosen profession.

A client presents in the emergency department exhibiting signs indicative of the onset of a bowel obstruction. Which bowel sounds should the nurse anticipate when auscultating the client's abdomen? A) Gurgling or clicking sounds B) High-pitched tinkling, rushing, or growling sounds C) Absence of sounds D) Continuous medium-pitched hum

B) High-pitched tinkling, rushing, or growling bowel sounds-known as borborygmus-are indicative of the onset of bowel obstruction. Gurgling or clicking sounds are considered normal. Absence of bowel sounds is indicative of late bowel obstruction, not onset of bowel obstruction. A continuous medium pitched hum-called a venous hum-is indicative of a cirrhotic liver.

A client who is living independently but needs skilled nursing services may take advantage of what type of healthcare? A) Long-term care B) Home healthcare C) Telehealth D) Assisted living

B) Home healthcare provides a variety of medical, therapeutic, and nonmedical services, such as wound care, dietary counseling, physical therapy, occupational therapy, skilled nursing services, and homemaker services. These services are available in private homes from healthcare professionals. Telehealth would not be adequate for providing skilled nursing services. Assisted living facilities do not typically include skilled nursing services. One aspect of long-term care is skilled nursing services, but the clients do not live independently.

The nurse is caring for a client who is 3 days postoperative following an emergency appendectomy. The nurse is reviewing the client's lab values and notes that the client's calcium levels have increased since before the surgery. Which intervention should the nurse implement to decrease the client's possibility of developing hypercalcemia? A) Measure the client's vital signs every 8 hours. B) Assist the client in ambulating around the room at least three times daily. C) Irrigate the client's Foley catheter daily. D) Help the client turn, cough, and deep breathe every 2 hours.

B) Hypercalcemia can occur from immobility. Ambulation helps prevent leaching of calcium from the bones into the serum. None of the other options listed here is related to the development of hypercalcemia.

Which client is exhibiting hypersomnia? A) The client only gets about 5 or 6 hours of sleep each night. B) The client gets roughly 8 hours of sleep each night but can't stay awake during the day. C) The client consistently has trouble getting to sleep and often lies awake for hours after bedtime. D) The client experiences repetitive involuntary leg movements that interfere with sleep.

B) Hypersomnia is a condition of getting enough sleep at night but still exhibiting daytime drowsiness. A client getting 5 or 6 hours of sleep each night is experiencing sleep loss. Difficulty falling asleep is insomnia. Repetitive involuntary leg movements is restless leg syndrome.

An 8-year-old female client complains of chronic fatigue that has persisted for several months despite an adequate amount of sleep each night. The nurse plans the care based on a nursing diagnosis of Fatigue. What intervention is the nurse most likely to try first? A) Recommend a sleep study. B) Recommend a healthy and well-balanced diet with adequate water intake. C) Recommend that the client's parents keep a journal of her activities and sleep patterns. D) Recommend cognitive-behavioral therapy.

B) If the client appears to be getting an adequate amount of sleep each night, then ensuring the client is eating a proper diet would be the first response to fatigue. A sleep study is not indicated for a client who is getting adequate sleep, and keeping a journal of the client's behavior is a step that would follow dietary recommendations if those don't address the fatigue. Cognitive-behavioral therapy is an approach that would follow other, less intensive approaches should they fail to address the fatigue.

The pathophysiologic stimulus that initiates asthma is A) bronchoconstriction. B) inflammation in the airways. C) airway edema. D) mucus secretion

B) In asthma, the airways are in a persistent state of inflammation. This inflammation can lead to bronchoconstriction, airway edema, and increased mucus secretion. Therefore, inflammation is the primary stimulus that initiates asthma.

The process of ________ involves the acquisition of lifelong learning, experience, technical expertise, and interdependent collaboration required to become a professional nurse. A) maturation B) formation C) transformation D) socialization

B) In recent years, nursing scholars have developed the concept of formation, a process that facilitates the transformation of an individual from a layperson to a professional nurse. Formation is an evolutionary process that requires the acquisition of lifelong learning, experience, technical expertise, and interdependent professional collaboration. It marks a departure from the prior model of socialization, in which nurses were said to become part of the profession through internalization of nursing's long history of subservient female roles, the acceptance of dominant behaviors for males, and a hierarchical structure in healthcare organizations.

A nurse forgets to return a client's bed to the low position after performing a bed bath. When a colleague points this out, the nurse states, "I should have returned the bed to the low position. Thank you for pointing out my error." With which characteristic is the nurse's response most consistent? A) Compassion B) Integrity C) Fidelity D) Justice

B) Integrity involves adherence to a strict moral or ethical code. Nurses demonstrate integrity in various ways, such as by accepting feedback as a tool for improving their delivery of client care and by maintaining accountability for their actions and freely admitting when they make mistakes. By admitting to an error, this nurse is demonstrating integrity. Justice has to do with being fair. Fidelity means to be faithful to agreements and promises. Compassion is an awareness of and concern for the suffering of others.

The nurse is providing care for a client who experienced a fracture requiring a plaster cast. Which nursing intervention is appropriate for this client? A) Prescribing opioid pain medication B) Assessing the client's neurovascular status C) Discouraging client ambulation D) Encouraging the client to keep the cast damp

B) It is appropriate for the nurse to assess the client's neurovascular status to monitor for compartment syndrome related to the fracture. The nurse can administer an opioid pain medication but cannot prescribe one. The nurse should encourage the client to ambulate and to keep the plaster cast dry.

Which of the following statements is true with regard to food allergies and children? A) Over the past decade, the prevalence of peanut allergy has decreased in the pediatric population. B) Many children eventually outgrow egg, milk, and soy allergies. C) Teenagers with food allergies are at lower risk for an allergic reaction than younger clients because they are more aware of their trigger foods and how to avoid them. D) Peanut allergies are most common in pediatric clients over 5 years of age.

B) It is not uncommon for people to outgrow allergies to egg, milk, soy, and wheat as they age; however, allergies to shellfish, peanuts, and fish usually persist throughout an individual's life. Among pediatric clients, the prevalence of peanut allergy has increased in recent years, with children under age 3 most commonly affected. As compared to younger children, teenagers with food allergies have the highest risk for an allergic reaction because they have a greater tendency to eat meals outside the home and are less likely to carry their medication.

Within the human body, which type of connective tissue connects bones to other bones to form a joint? A) Tendon B) Ligament C) Cartilage D) Myelin

B) Ligaments, tendons, and cartilage are all connective tissues. Ligaments connect bones to other bones to form a joint. Tendons connect bones to muscles and carry the contractile forces from the muscle to the bone to cause movement. Cartilage is a type of flexible connective tissue found in many locations throughout the body. Myelin is not a type of connective tissue but rather a fatty substance that insulates neuronal axons and promotes faster signal transmission.

The nurse teaches a client about lifestyle modifications to help manage hypertension. Which client statement indicates teaching has been effective? A) "I won't be able to run in marathons anymore." B) "I know I need to give up my cigarettes and alcohol." C) "I need to get started on my medications right away." D) "My father had hypertension, did nothing, and lived to be 90 years old."

B) Limiting intake of alcohol and discontinuing tobacco products are important nonpharmacologic methods for controlling hypertension. Implementing lifestyle modifications may eliminate the need for pharmacotherapy, so the client may not have to take medication right away. Increasing physical activity is an important lifestyle modification for controlling hypertension. The fact that the client's father had hypertension and lived to be 90 years old does not mean that the client will have the same experience; the client is in denial.

The nurse is caring for a client who is scheduled to receive metoprolol (Lopressor). What should the nurse teach the client about this medication? A) Expect a rapid heart rate. B) Change positions slowly. C) Reduce protein intake. D) Increase fluids.

B) Metoprolol is a beta blocker. The client should be instructed to use care when ambulating and to change positions slowly because this medication causes orthostatic hypotension. This medication does not cause a rapid heart rate. Protein restriction is not indicated with this medication. The client should not be instructed to increase fluids.

The nurse is caring for a client who has recently been diagnosed with fibromyalgia. The client has complained of pain, fatigue, and sleep disruptions. Which medication should the nurse anticipate will most likely be prescribed as part of the client's treatment plan? A) Duloxetine B) Milnacipran C) Pregabalin D) Acetaminophen

B) Milnacipran is a clinical pharmacologic therapy for pain, fatigue, and sleep disruptions for fibromyalgia. Duloxetine and pregabalin are not prescribed for fatigue, and acetaminophen can be used for pain but will not assist with the sleep disturbances.

The nurse is caring for a client who was admitted to the hospital 1 day prior with cholelithiasis. Which new assessment finding indicates that the stone has probably obstructed the client's common bile duct? A) Nausea and vomiting B) Jaundice C) Right upper quadrant (RUQ) pain D) Elevated cholesterol level

B) Nausea and RUQ pain occur in cystic duct disease, but obstruction of the common bile duct results in reflux of bile into the liver, which produces jaundice. Cholesterol levels do not increase with biliary obstruction

A novice nurse has accepted a position on a medical-surgical unit at a local university hospital. In order to provide safe care to clients, the nurse should plan to develop which competency? A) Creating a culture of trust within the hospital B) Functioning as a member of the healthcare team C) Promoting appropriate values that clients should adopt D) Reporting families for bringing food to the client's room

B) New nurses should learn about the healthcare team members and determine whom to collaborate with in certain situations. Rather than reporting families, the nurse would work with families to help meet their needs if food is not allowed in the room. The nurse would respect the values of clients and not seek to impose any on the clients. Creating a culture of trust is a system change that is implemented by the administration.

The nurse is providing instructions to a client who has been prescribed a nonsteroidal anti-inflammatory drug (NSAID). Which information is highest priority for the nurse to explain to the client about this medication? A) "Take your medication on an empty stomach." B) "Drink at least 8-10 glasses of water a day while taking this medication." C) "Constipation is common with your medication, so include roughage in your diet." D) "Take your medication with food."

B) Nonsteroidal anti-inflammatory drugs (NSAIDs) are nephrotoxic; maintaining adequate hydration when taking these medications is important because it will help prevent kidney damage. Taking NSAIDs with food is recommended because doing so will decrease gastrointestinal (GI) irritation, but preventing kidney damage is more of a priority. Constipation is not an issue with NSAIDs.

The nurse is caring for a hospitalized client who is experiencing​ anxiety-related hyperventilation. When calculating the​ client's intake and​ output, where would the nurse anticipate the need for an adjustment in fluid​ loss? A. Urine B. Insensible loss C. Sweat D. Feces

​B. Rationale: With increased​ respirations, the client will experience a​ greater-than-normal insensible loss of fluid through the lungs. Hyperventilation will not affect the amount of fluid lost through the​ urine, sweat, or feces.

A client is experiencing​ fatigue, headache, and nausea and vomiting and has a decrease in deep tendon reflexes. Which electrolyte imbalance should the nurse suspect is causing this​ client's symptoms? A. Hypokalemia B. Hypercalcemia C. Hyperchloremia D. Hypomagnesemia

​Rationale: Hypercalcemia is an increase in serum calcium level. Clinical manifestations of this condition include​ fatigue, weakness, decreased tendon​ reflexes, headache, impaired​ cognition, anorexia, nausea and​ vomiting, lethargy,​ polyuria, muscle​ weakness, constipation, and cardiac dysrhythmias.​ Hypomagnesemia, hyperchloremia, and hypokalemia do not produce these clinical manifestations.

The nurse is teaching a class on infection control. Which nursing measure is most appropriate in breaking a link in the chain of infection? A) Place contaminated linens in a paper bag. B) Use personal protective equipment (PPE). C) Cover one's cough by placing the mouth in the hand. D) Wear sterile gloves for client care.

B) PPE, according to Occupational Safety and Health Administration (OSHA) standards, has to be used whenever the situation dictates and is a nursing measure to break a link in the chain of infection. Placing linens in a paper bag would allow germs to come out through the bag, and the linen would act as a fomite, thus allowing the chain to continue. Covering one's mouth when coughing prevents airborne droplets from escaping into the air for others to contract in the chain of infection. However, the cough should be covered in the elbow, not in the hand. Nonsterile gloves have to be worn when providing certain aspects of client care. The gloves should be changed between clients, and hands are to be washed.

Which of the following triggers pain? A) The central nervous system B) The peripheral nervous system C) The musculoskeletal system D) The cardiovascular system

B) Pain is triggered by the peripheral nervous system, which lies outside the brain and spinal cord of the central nervous system and does not involve the musculoskeletal or cardiovascular systems.

The most common source for bacteria that cause a urinary tract infection is A) a catheter. B) the mucous membranes of the perineal area. C) the hands. D) clothing such as underwear.

B) Pathogens usually enter the urinary tract by ascending from the mucous membranes of the perineal area into the lower urinary tract. The hands and clothing are potential sources of bacteria that cause a UTI, but they are not the most common. The presence of a catheter is a risk factor for development of a UTI, but the source of bacteria is still often the mucous membranes of the perineal tract or some other source, not the catheter itself.

Which of the following sounds would not be detected during percussion of a healthy client? A) Tympany B) Hyperresonance C) Dullness D) Flatness

B) Percussion elicits five types of sound: flatness, dullness, resonance, hyperresonance, and tympany. Flatness is an extremely dull sound produced by very dense tissue, such as muscle or bone. Dullness is a thudlike sound produced by dense tissue such as the liver, spleen, or heart. Resonance is a hollow sound, such as that produced by lungs filled with air. Hyperresonance is not produced in the healthy body. It is described as booming that can be heard over an emphysematous lung. Tympany is a musical or drumlike sound produced from an air-filled stomach.

The nurse is preparing to administer 20 mEq of potassium chloride to a client who has been vomiting. Which information about the purpose of this medication should the nurse explain to the client? A) It is vital in regulating muscle contraction and relaxation. B) It is needed to maintain skeletal, cardiac, and neuromuscular activity. C) It controls and regulates water balance in the body. D) It is used to synthesize protein and DNA within the body's cells.

B) Potassium is the major cation in intracellular fluids, with only a small amount found in plasma and interstitial fluid. Potassium is a vital electrolyte for skeletal, cardiac, and smooth muscle activity. Calcium is vital in regulating muscle contraction and relaxation. Sodium controls and regulates water balance in the body Magnesium is used in the cells to synthesize protein and DNA.

Which of the following barriers to communication involves asking a client for information chiefly out of curiosity rather than with the intent to assist the client? A) Challenging B) Probing C) Testing D) Rejecting

B) Probing involves asking a client for information chiefly out of curiosity rather than out of a desire to assist the client. Probing often places clients in a defensive position and violates their privacy. Challenging refers to giving a response that makes clients prove their statement or point of view. Testing involves asking questions that make a client admit to something. Rejecting is refusing to discuss certain topics with a client. All four of these behaviors are barriers to effective nurse-client communication.

After completing an assessment, the nurse determines a client is at risk for safety issues. Which data supports the nurse's conclusion? A) Lives with adult married daughter and family B) Occasional dizziness with walking C) Follows a vegetarian diet D) Receives an annual ophthalmologic examination

B) Risks to safety include factors that can impact falls such as mobility issues or balance. Living with family, eating a vegetarian diet, and having annual eye examinations do not increase the client's risk for safety issues.

Which of the following lifestyle changes would most likely increase a client's risk for cholelithiasis? A) Reducing intake of high-fat foods B) Increasing intake of high-cholesterol foods C) Beginning a regular exercise routine D) Discontinuing use of hormonal birth control

B) Several factors increase a client's risk for gallbladder disease, including consuming foods that are high in fat and cholesterol and using medications that contain estrogen, such as hormonal birth control. Exercise can aid in weight control, thereby reducing a client's risk for gallbladder disease.

Which of the following is most characteristic of the clinical manifestations of sleep apnea? A) Difficulty thinking B) Difficulty breathing C) Difficulty waking up D) Difficulty controlling motor functions

B) Sleep apnea can manifest as snoring or gasping during sleep, and treatment may involve devices to assist with ventilations during sleep. Frequent awakenings typically accompany sleep apnea, not difficulty waking up. Cognitive deficits are more typical of insomnia. Difficulty controlling the body is more typical of restless leg syndrome

What impact might corticosteroids have on tissue integrity? A) It may increase sensitivity to sunlight, leading to sunburns. B) It may cause thinning of the skin, making skin more easily injured. C) It may make skin appear shiny and lose its hair distribution. D) It may cause the skin to become overly dry.

B) Some medications, such as corticosteroids, cause thinning of the skin, making it much more easily damaged. Antibiotics, chemotherapy drugs, and some psychotherapeutic drugs increase sensitivity to sunlight and can predispose the individual to sunburns. Impaired peripheral arterial circulation in the lower extremities may produce skin that appears shiny and has lost its hair distribution. Excessive cleansing can cause the skin to become overly dry.

The nurse is admitting a client to an inpatient psychiatric unit. The client is speaking wildly and is obviously very agitated. Which action by the nurse would be appropriate to calm the client? A) Placing the client in a private room, away from others B) Speaking to the client in a soft, calm tone C) Administering a prn medication to sedate the client D) Using short sentences when talking to the client

B) Speaking in a soft, calm manner is the first step in attempting to soothe an excited client. The nurse's tone may calm the excited client. Using short sentences is a useful approach, but in the case of an excited client, the nurse would attempt to calm the client first. Isolating the client may be necessary if the client's behavior escalates to violence, but that is not evident here and is not the first choice of action. Giving a sedative is the last resort and is used only if the client is threatening to hurt self or others.

A 6-month-old is in the clinic for a well-child visit. As part of the visit, the nurse will assess his development using the Ages and Stages Questionnaire (ASQ). Which will the nurse rely on to make her assessment? A) Observation of the child's skills in a variety of areas B) Parent reports of communication and motor skills, social skills, and problem-solving ability C) Parental reports and observation of the child's skills in a variety of areas D) Childcare provider reports of communication and motor skills, social skills, and problem-solving ability

B) The ASQ relies on parental reports to assess the child's communication and motor skills, social skills, and problem-solving ability. The Battelle Developmental Inventory Screening Test (BTDIST) uses both observation and parental reports to assess a variety of areas. Tests specifically described in this chapter do not include those based solely on observation or childcare provider reports

What is one possible conclusion the nurse could draw after assessing a client with the Braden Scale? A) The client is at risk for falls. B) The client is at risk for pressure ulcers. C) The client is at risk for malnutrition. D) The client may be unable to complete activities of daily living.

B) The Braden Scale is used to determine an individual's risk for developing pressure ulcers. Fall risk would be assessed with a tool such as the Get-Up-and-Go Test, Berg Balance Test, Single Leg Stance Test, or Fullmer SPICES instrument. Risk for malnutrition would be assessed with a tool such as the Mini-Nutritional Assessment. Ability to complete activities of daily living (ADLs) would be assessed with a device such as Katz's ADL Scale or the Instrumental ADL tool.

For which client would the Denver II tool not be an appropriate choice for the nurse to use during a well-child assessment? A) A male client B) A client aged 7 or older C) A female client D) A client aged 6 or younger

B) The Denver II tool is a screening test administered to well children between birth and 6 years of age. It is designed to test 20 simple tasks and items in four sectors: personal-social, fine motor adaptive, language, and gross motor. This tool is used with both male and female clients, but it would not be used for clients aged 7 or older.

The nurse is administering a bed bath to a client in a long-term care facility. The nurse is careful to cover the client during the bath. Which nursing role does this action reflect? A) Communicator B) Caregiver C) Client advocate D) Teacher

B) The caregiver role includes those activities that assist the client physically and psychologically while preserving the client's dignity. In this scenario, the nurse is acting in the role of a caregiver. As a communicator, the nurse identifies client problems, then communicates these verbally or in writing to other members of the health team. As a teacher, the nurse helps clients learn about their health and the healthcare procedures they need to perform to maintain or restore their health. As a client advocate, the nurse acts to protect clients and represents their needs and wishes to other health professionals.

The charge nurse is observing a newly licensed nurse conduct an admission assessment on a client with asthma. Which action by the newly licensed nurse requires immediate intervention? A) The newly licensed nurse is observed obtaining the pulse oximetry reading 10 minutes after the client used an albuterol inhaler. B) The newly licensed nurse is observed continuing to ask the client questions regarding history while the client demonstrates difficulty breathing and signs of respiratory impairment. C) The newly licensed nurse is observed assessing the client's thoracic wall, skin, and nail beds. D) The newly licensed nurse is observed auscultating breath sounds with a stethoscope.

B) The charge nurse should intervene immediately if the nurse observes the client is demonstrating impairment at or near respiratory failure; the client will not be able to respond to questions. Assessment questions should be tailored and asked of any family member or friend accompanying the client. Although the pulse oximetry reading may not be a true indicator of the level of respiratory distress of the client because of the use of an albuterol inhaler within 30-60 minutes of this assessment, it is still an appropriate action for the newly licensed nurse to take and does not require the charge nurse to intervene immediately. The charge nurse may speak to the newly licensed nurse later with regard to this assessment. Assessing the client's thoracic wall, skin, and nail beds is an appropriate action at this time. Auscultating the client's breath sounds with the use of a stethoscope is appropriate

A toddler being prepared for a lumbar puncture begins to cry when carried into the treatment room by the mother. Which nursing diagnosis is most appropriate for the client at this time? A) Knowledge Deficient of the procedure B) Anxiety related to anticipated painful procedure C) Fear related to the unfamiliar environment D) Ineffective Coping related to an invasive procedure

B) The child associates the treatment room with a painful procedure, and the reaction to entering the treatment room is based on anticipation of repeat discomfort. The child's behavior is appropriate for coping in a child of this age. This child is not old enough to understand the need for a lumbar puncture. The child's fear is related not to the unfamiliar environment but to the anticipated pain associated with the treatment room from having undergone painful procedures there.

The nurse is concerned that a client with an alteration in perfusion is at risk for inadequate oxygenation. What should the nurse consider when planning for this client's potential health problem? A) Encouraging ambulation every 30 minutes B) Instructing on deep breathing C) Administering medications appropriate to increase heart rate D) Positioning to increase blood return

B) The client is at risk for inadequate oxygenation. The nurse should consider teaching the client the importance of deep breathing to increase the amount of oxygen in the body tissues. Encouraging ambulation every 30 minutes would negatively impact oxygenation. Periods of rest should occur between activities, and no activity should be too strenuous. The client with oxygenation issues will have tachycardia. The nurse should consider medications that would reduce instead of increase the heart rate. The client should be in the high-Fowler position to improve oxygenation. Positions to increase blood flow to the heart include Trendelenburg, which would negatively impact oxygenation.

The nurse is caring for a client who will be discharged with an indwelling catheter. The nurse has provided education to the client and family in regard to catheter care once the client is discharged. Which client or family action indicates a correct understanding of the information presented? A) Hanging the drainage bag on a towel rod B) Taking a shower each day instead of taking a tub bath C) Restricting the amounts of fluids per day D) Emptying the drainage bag twice a day

B) The client should take a shower rather than a tub bath because sitting in a tub allows bacteria to easily access the urinary tract. The drainage bag should be emptied regularly, not just once a day but at least three times a day. Hanging the drainage bag on the towel rod is too high. The drainage bag should be hung below the bladder. Adequate amounts of fluids should be consumed to help prevent sediments and infections

The nurse is caring for a group of clients on a medical-surgical unit. Which client does the nurse anticipate to be at the greatest risk for alterations in urinary elimination? A) The client with hypertension who takes a diuretic to manage blood pressure B) An 80-year-old male client reporting frequent urination at night C) A 25-year-old female client with low self-esteem D) A client who had bladder cancer and now has a newly created ileal conduit

B) The client who is 80 years old with frequent urination at night may be having problems with his prostate. Older male adults experience urinary retention due to prostate enlargement, causing an alteration in urinary elimination. The 25-year-old experiencing low self-esteem has a psychological problem and will need therapy to find the root of the problem. The client who had bladder cancer and now has an ileal conduit doesn't have kidney damage, only the bladder removed. Continued urine production through the ileal conduit will need to be observed and assessed frequently by the staff. The client with high blood pressure takes medication to remove excess fluid from the body, and as long as urine elimination increases, there should be no problems.

An adult client is diagnosed with fibromyalgia. The client asks the nurse whether a recent of infection with the Coxsackie B virus could have caused fibromyalgia. Which response by the nurse is the most appropriate? A) The Coxsackie B virus has nothing to do with fibromyalgia. B) The Coxsackie B virus may have triggered the fibromyalgia. C) The Coxsackie B virus definitely caused the fibromyalgia. D) The Coxsackie B virus probably caused the fibromyalgia.

B) The exact cause of fibromyalgia is unknown. Infections such as hepatitis C virus (HCV), HIV, Coxsackie B, and parvovirus may trigger fibromyalgia. However, over 70% of clients have no precipitating factor for the disease. The best response to this client is simply that the Coxsackie B virus may have triggered the fibromyalgia without speculating as to the likelihood of the infection being the cause.

Which statement best exemplifies the etiology of fibromyalgia? A) Stress and poor sleep hygiene are the main precipitating factors for fibromyalgia. B) No exact cause is known, but some clients with fibromyalgia may have precipitating factors. C) The primary cause for fibromyalgia in nearly all clients who have it is infection. D) Precipitating factors for fibromyalgia are primarily physiological.

B) The exact cause of fibromyalgia is unknown; over 70% of clients with fibromyalgia have no precipitating factor for the disease. Infections, physical trauma, psychosocial stressors, vaccinations, and chemical substances all might be causative factors for fibromyalgia in some patients.

A client with aspiration pneumonia is diaphoretic, pale, and taking gasping breaths. Which is the priority nursing action? A) Notify the healthcare provider. B) Complete a thorough cardiopulmonary assessment. C) Administer 10 L of oxygen per face mask. D) Reposition the client to help with breathing.

B) The first step in the nursing process is to complete an assessment of the client. The client is indeed experiencing difficulty, but the nurse needs to assess the extent of the need and the reason for the problem before taking action. The healthcare provider will ask the nurse to identify the reason for the problem and the extent of the problem. Administering oxygen could be dangerous to the client in some cases, such as if the client has chronic obstructive pulmonary disease. The client may have simply slipped down in the bed and need repositioning; on the other hand, the client may be in trouble, so making the decision to simply reposition the client without assessment could cause harm.

The nurse is preparing a presentation to a group of adolescent clients regarding proper nutrition. Which of the following teachings is appropriate for this group? A) The high metabolism of the typical adolescent lowers nutritional requirements. B) It is normal for adolescents to consume a lot of calories, but their diet should still be balanced. C) Roughly half of an adolescent's daily caloric intake should come from fats. D) The taste preferences of adolescents typically correlate to the nutritional value of what they eat.

B) The high metabolism of the typical adolescent raises nutritional requirements. The adolescent growth spurt is accompanied by rapid gains in height and weight. Bodily demands for calories and nutrients increase dramatically during this time. Adolescent girls require about 2200 calories per day, and adolescent boys require about 2800 calories per day. Roughly 50% of calories should come from complex carbohydrates, 30% from fats, and 20% from proteins. Taste preferences are one of the contributing factors to adolescents exceeding fat intake requirements and not getting enough other nutrients or vitamins, minerals, and fiber.

Which score would a nurse select from the muscle function grading scale if the client has full strength and range of motion in a given joint? A) 0 B) 5 C) 8 D) 10

B) The muscle function grading scale ranges from 0 to 5. A score of 0 indicates paralysis, meaning that the client cannot contract the muscles associated with a given joint. In contrast, a score of 5 indicates that the client can move a joint through the full range of motion under full resistance.

The nurse is caring for a 43-year-old client. What education should the nurse implement to best address the overall health promotion needs of someone in this age group? A) Teach the client about ergonomic aids for computer use B) Teach the client about age-appropriate medical screenings C) Tell the client to seek medical help for injuries D) Encourage the client to not drive while distracted

B) The nurse can provide education in many areas related to disease prevention, including teaching about age-appropriate medical screenings. Teaching the client about ergonomic aids for computer use, telling the client to seek medical help for injuries, and encouraging the client to not drive while distracted are all methods to prevent injury, not disease.

The nurse is caring for a young adult client after a cervical biopsy. The client has expressed anxiety about the results. The healthcare provider peeks into the client's room and says, "The biopsy is negative." The nurse later finds the client sobbing. Which response by the nurse is most appropriate? A) "What did the healthcare provider tell you about the biopsy?" B) "You seem upset. Do you want to talk to me about the test results?" C) "Why are you crying after getting such good news?" D) "In this case, the term 'negative' is good!"

B) The nurse does not know specifically what the client is upset about and should ask the client an open-ended question so she can talk. The healthcare provider, in delivering important news to the client, should have taken time to sit with her and discuss the test results. In telling the client that the test was negative, the provider did not clarify what "negative" actually meant. Using medical jargon without explanation can lead to misinterpretation by the client. Asking the client why she is crying about good news does not allow the client to express concern regarding the results. Asking the client what the provider told her assumes that she is crying because of what the provider said and does not allow her to express her concern in an open-ended manner. Saying that the test results are good in this case assumes that the client has misunderstood the results, which may be true but does not allow the client to express her concerns.

A nurse would like to implement an evidence-based practice change that will influence client care on the medical-surgical unit. The nurse works with the nurse manager and other members of the leadership team to write a new policy and produce educational materials for the unit's staff and clients. In carrying out these actions, the nurse is practicing which standard of professional performance? A) Leadership B) Collaboration C) Evaluation D) Collegiality

B) The nurse is practicing collaboration by working with other staff members to implement a policy change. Collaboration involves working with clients, their families, and others in the conduct of nursing practice. Collegiality describes interaction with and contributions to the professional development of peers and colleagues, as would be the case in a mentoring relationship. Leadership involves providing direction in a professional practice setting. Evaluation involves a comparison between one's own nursing practice and professional practice standards.

The nurse is conducting a health history as part of a nursing assessment. The client says to the nurse, "I am allergic to penicillin." Which assessment question would best help the nurse learn more about the client's allergy? A) "Where did you experience the reaction?" B) "What type of reaction occurred?" C) "How long did your symptoms last?" D) "Do any other family members have this same allergy?"

B) The nurse should ask the client to provide more information about the type of reaction that occurred when the penicillin was administered. The location of the reaction and how long the symptoms lasted are important, but the priority is determining the type of reaction the client experienced. Asking whether any other family members have the same allergy will not provide the nurse with useful information.

The nurse is caring for a client with a new colostomy. The client has been taught how to perform colostomy care and has been successful with return demonstration to the staff. Although the client is able to perform care independently and has asked to do so, the charge nurse has instructed the nursing staff to continue performing colostomy care for this client. When addressing this issue directly with the charge nurse, which statement by a staff nurse is the most appropriate? A) "The client will change the apparatus whether you like it or not." B) "The client has been trained to change the apparatus and has expressed interest in performing this procedure independently." C) "You have no right to continue delegating this task to nurses when the client has been trained to change the apparatus." D) "I am going to tell the nurse manager that you won't allow the client to change the apparatus independently."

B) The nurse should make a clear, assertive statement saying that the client learned the procedure and wishes to execute it. Saying "You have no right . . ." is a challenge to the charge nurse and will only result in escalation of the argument. Telling the charge nurse that the nurse manager will be notified is a threat and inflames the situation. Insisting that the client will continue to change the apparatus will likely result in the charge nurse taking further action to prevent the client from performing self-care.

The nurse is caring for a client who has not been adhering to treatment with anti-hypertension medication. Which approach to addressing this issue should the nurse use? A) Indifference B) Nonjudgmental C) Demanding D) Confrontational

B) The nurse who listens to the client openly and nonjudgmentally will both validate the client's self-esteem and communicate the idea of partnership in the treatment plan for the client. Indifference or demanding or confrontational attitudes will likely elicit a negative response from the client, and the client will be less likely to change his behavior.

A female nurse is caring for a 21-year-old male client with a questionable gastrointestinal blockage. The healthcare provider prescribes an enema. Which reaction by the client would the nurse anticipate when planning care? A) "May I have a visitor in the room with me for support during the procedure?" B) "I would rather have my doctor perform this procedure." C) "I don't know what an enema is." D) "I am afraid of having an enema."

B) The nurse would anticipate that most young adult clients will be embarrassed by this procedure when the nurse and client are of different genders. When the client states that he would rather have his doctor perform the enema, he is probably motivated by embarrassment and acting on the assumption that the doctor is male. The nurse should approach the client beforehand to address the issue. Most clients would only experience annoyance, not fear, in relation to this procedure. Most clients in this age group would also be familiar with what an enema is, even if they have not had an enema themselves. The nurse would definitely not expect the client to request the presence of another individual in the room for this procedure.

The nurse is conducting a home risk assessment for a family with toddler and preschool-age children. Which should the nurse identify as the priority safety hazard? A) Safety plugs in electrical outlets B) Medications on the kitchen counter C) Lack of helmets next to bicycles D) Child locks on the doors

B) The nurse would instruct the parents to keep medications out of the children's reach. Medication poisoning happens easily with young toddlers and preschool-age children who think the medication is candy. Safety plugs are appropriate for this age group. Child locks are appropriate to keep toddlers from wandering out to the street. A lack of a helmet next to a bike does not mean there are no helmets in the house. This finding would cause the nurse to ask more questions but is not considered a definite safety risk.

A client with chronic obstructive pulmonary disease (COPD) is prescribed oxygen 24% 2 L/min. Which is the best method to administer oxygen to this client? A) Face mask B) Nasal cannula C) Nonrebreather mask D) Venturi mask

B) The oxygen delivery device that would safely administer 24% oxygen at the flow rate of 2 liters per minute is through nasal cannula. The other delivery devices are better suited for higher percentages of oxygen and higher flow rates

A client with a BMI of 35 is recovering from total hip replacement surgery and experiencing pain that is exacerbated with movement. The client says to the nurse, "I live alone. How will I ever be able to return to my home?" Based on this information, which is the priority nursing diagnosis for this client? A) Overweight B) Acute Pain C) Impaired Physical Mobility D) Ineffective Coping

B) The priority nursing diagnosis is Acute Pain. Unless this pain is controlled, the client will not be able to participate in interventions to address the nursing diagnosis of Impaired Physical Mobility. The diagnoses of Ineffective Coping and Overweight can be addressed after Acute Pain and Impaired Physical Mobility have been addressed.

A nurse working in the intensive care unit (ICU) is caring for a client who is 10 days postoperative after open abdominal surgery. The client has a well-approximated midline surgical incision that has numerous staples, and the nurse notes a "healing ridge" is present. Based on this information, the incision is currently in which phase of the healing process? A) Inflammatory phase B) Proliferative phase C) Maturation phase D) Synthesis phase

B) The proliferative phase, which is the second phase in the healing process, extends from day 3 or 4 to about day 21 postinjury. If the wound is sutured, a raised "healing ridge" appears under the intact suture line. There is no synthesis phase in the healing process, and the other choices are incorrect.

A client sustained multiple fractures in a motor vehicle crash. Of the various fracture types sustained by the client, which places the client at highest risk for osteomyelitis? A) Avulsion fracture B) Open fracture C) Comminuted fracture D) Depression fracture

B) The risk for osteomyelitis, or bone infection, is highest with an open fracture, in which the bone breaks through the skin. Comminuted, avulsion, and depression fractures are closed from the environment and present a lower risk of infection.

A client hospitalized with an open reduction and internal fixation of a fractured femur reports right calf pain. The nurse notes that the client's right calf is 3.5 cm larger than the left calf with generalized posterior erythema. The right calf is tender to touch, and the dorsalis pedis pulse is 3/4+ bilaterally. Which of the following is the priority action by the nurse? A) Use a Doppler stethoscope to confirm pedal pulses. B) Notify the healthcare provider of the findings. C) Prepare to apply a cast to the right leg. D) Prepare to administer intravenous heparin.

B) These findings indicate possible deep vein thrombosis (DVT). The nurse's first action upon assessing these signs and symptoms should be to notify the healthcare provider immediately. If a pedal pulse can be palpated, then a Doppler stethoscope is not needed; however, a Doppler ultrasound test may be ordered by the provider. A cast is not indicated with internal fixation. Intravenous heparin will likely be ordered after the condition is confirmed by the provider.

The nurse is assessing a client who is recovering following surgery. Which factor would increase this client's susceptibility to infection? A) Intact mucous membranes B) Presence of an incision C) Dry skin D) Active bowel sounds

B) This client has a surgical incision, so the body's first line of defense, the skin, is not intact. Active bowel sounds, dry skin, and intact mucous membranes are factors that help defend the body against infection.

The nurse is caring for an adult client who sustained a right distal radial fracture and a left tibia fracture. Which mobility aid does the nurse anticipate being used for this client? A) Lofstrand crutches B) Platform crutches C) Walker D) Axillary crutches

B) This client has fractures in both the leg and wrist. Platform crutches are used for clients who are unable to bear weight on their wrists. A walker, axillary crutches, and Lofstrand crutches all require use of the wrists.

A nurse obtains certification to provide direct client care, educate others, consult, conduct research, and manage oncology care. Which expanded nursing role best describes this nurse's career position? A) Nurse anesthetist B) Clinical nurse specialist C) Nurse educator D) Nurse researcher

B) This nurse is acting in the role of clinical nurse specialist. A clinical nurse specialist has an advanced degree or expertise in a specialized area of practice; has passed a certification exam and obtained any other required credentialing; and works to provide direct client care, educate others, consult, conduct research, and manage care. Nurse anesthetists have completed advanced education in an accredited program in anesthesiology. Nurse educators are employed in nursing programs, at educational institutions, or in hospital staff education and may have a baccalaureate degree or more advanced preparation. Nurse researchers usually have advanced education at the doctoral level.

The nurse is assessing an older adult client during a routine health maintenance visit. To assess the client's range of motion of the knees, which action by the nurse is appropriate? A) Seat the client and extend each knee until the client alerts the nurse of severe pain. B) Seat the client and extend each knee until the client alerts the nurse of any pain whatsoever. C) Place the client prone and gently lift the entire right leg, followed by the entire left leg. D) Have the client stand and extend each knee as far as it will go.

B) To assess the range of motion of the knees, the client should be placed in a sitting position. The client should be instructed to alert the nurse at the first sign of discomfort when checking range of motion of the knees. The knee should not be forced beyond the pain limit. Placing the client prone and lifting the entire leg checks the range of motion of the hips. The client should not be asked to stand when assessing range of motion of the knees.

The nurse is caring for a 3-year-old child who is in the hospital for the first time. The child appears frightened and is clinging to her parents. What action can the nurse take to help the child feel more secure if the child needs to stay at the hospital without her parents? A) Stay with the child when the parents go home B) Have the parents bring comfort items from home to leave with the child C) Keep dangerous medications and equipment out of the child's reach D) Make sure the child wears proper identification at all times

B) To help a child feel more secure, the nurse can suggest that parents bring in a few comfort items the child is familiar with, such as photos, a favorite blanket, or a favorite toy. Having the nurse stay with the child at all times once the parents have left is not practical and could cause harm to other clients under the nurse's care. Although the nurse should keep dangerous medications and equipment out of the child's reach and make sure the child wears proper identification at all times, these actions will likely not help the child feel more secure.

Several nurses are discussing the Joint Commission's 2016 National Patient Safety Goals during a staff meeting. Which element of performance should the nurses implement to meet the goal of identifying clients correctly? A) Labeling all medications with the client's name B) Consistently using two methods to identify the client C) Asking the client's name before conducting assessments D) Marking the intended surgical site on the client

B) Two elements of performance that accompany the goal to identify clients correctly include consistently using two methods to identify the client and ensuring that clients receiving blood transfusions are correctly identified prior to transfusion. Labeling medications with the medication information helps prevent medication errors, and marking the intended surgical site on the client helps prevent surgical errors. Asking the client's name before conducting assessments is not associated with a National Patient Safety Goal.

Transfusion reactions and Rh incompatibility are both examples of which type of hypersensitivity reaction? A) Type I B) Type II C) Type III D) Type IV

B) Type I, or immediate hypersensitivity, reactions are characterized by rapid development of symptoms after exposure to an antigen; an example is anaphylaxis. Type II, or cytotoxic hypersensitivity, reactions involve the rupture of cells targeted by the immune response that may affect a variety of organs and tissues; examples include transfusion reactions and Rh incompatibility. Type III, or immune-complex, reactions include inflammatory response in the targeted tissues that leads to tissue damage; examples include autoimmune disorders such as systemic lupus erythematosus (SLE) and rheumatoid arthritis (RA). Finally, type IV, or delayed-type hypersensitivity, reactions involve a major histocompatibility complex and are characterized by tissue damage at the site of antigen contact within 24-48 hours of exposure; an example is allergic contact dermatitis.

The home healthcare nurse is traveling to a client's home for the first time. What observation would suggest a safety hazard for the nurse? A) Neighbor walking a dog on a leash B) Porch steps that are broken and rotting C) Client medications on the kitchen counter D) Absence of street lights in the neighborhood

B) Unhygienic or dangerous surroundings, such as broken and rotting porch steps, may pose a safety hazard for the nurse. Unrestrained and hostile animals may pose a safety threat, but a neighbor walking a dog on a leash is not a safety hazard for the nurse. Client medications on the kitchen counter may be a safety hazard if small children are present in the home, but this does not pose a safety hazard for the nurse. The nurse should only make home visits during daylight hours to maintain safety, so the absence of street lights in the neighborhood should not affect the nurse.

A client requests a small inflated doughnut-style device to sit on to relieve pressure. Which response by the nurse is most appropriate? A) "I will need to get an order from the physician." B) "Using the doughnut can cause skin breakdown." C) "You will need to wait until discharge, then use the doughnut at home." D) "I will obtain the device for you."

B) Use of a doughnut-style device applies pressure and results in tissue anoxia. The client may indeed feel that pressure is lessened with use of the device, but this is due to the loss of sensation. Use of a doughnut-style device should be avoided whether at the hospital or at home

What is vertigo? A) Involuntary rapid eye movements B) A feeling of rotation or imbalance C) An infection of the vestibular nerve D) Impaired olfaction

B) Vertigo is a feeling of rotation or imbalance. Nystagmus is involuntary rapid eye movements. Vestibular neuritis is an infection of the vestibular nerve that is a common cause of vertigo. Impaired olfaction, or impaired sense of smell, has nothing to do with vertigo.

Which of the following statements best describes the vital signs the nurse collects during the preoperative phase? A) They are the only vital signs collected during the perioperative period. B) When later vital signs are taken, they are compared against the preoperative set. C) Generally preoperative vital signs are only relevant during the intraoperative process. D) These are not essential and may be omitted for emergency surgeries.

B) Vital signs, including blood pressure, pulse, respiration, and oxygen saturation, should be taken throughout the perioperative period, with the baseline vital signs being obtained during the preoperative phase. Later vital signs will be compared against this baseline. Preoperative vital signs are not the only vital signs collected during the perioperative period, they are relevant through the perioperative period as a baseline, and they are important to record for any client.

An adult client diagnosed with sleep apnea has been prescribed a continuous positive airway pressure (CPAP) machine as treatment. The nurse is instructing the client on how to use the machine. Which instruction least relates to ensuring the patient's comfort with the device? A) Use a properly sized mask with the straps tight. B) Instruct the client to wear the mask with air pressure while sleeping. C) Show how to adjust the pressure to reduce difficulty exhaling. D) Demonstrate relaxation techniques to reduce a claustrophobic feeling when wearing the mask.

B) Wearing the right size mask and keeping the straps tight, doing relaxation exercises to reduce the claustrophobic feelings caused by wearing the mask, reducing the difficulty of exhalation by properly adjusting pressure may all help safeguard the patient's comfort with the CPAP device, but wearing the mask with air pressure while sleeping is simply the general way the device should be used and doesn't make any special allowances for the comfort of the patient.

________ is a state of being in which individuals engage in behaviors that enhance their quality of life and maximize their personal potential. A) Health promotion B) Wellness C) Prevention D) Health restoration

B) Wellness can be described as a state of being in which individuals engage in activities and behaviors that enhance their quality of life and maximize their personal potential, including their physical fitness and emotional health. In comparison, health promotion is a process that enables individuals and communities to increase their control over the determinants of well-being, thereby improving their overall health; prevention focuses on maintaining health by working to stop illnesses and injuries from occurring; and health restoration involves efforts to return clients to their optimal state of health following illness or injury.

The nurse is preparing to conduct a cardiac assessment for a pediatric client. Which location will the nurse use when auscultating the apical pulse? A) At the fifth intercostal space B) At the left nipple C) At the right nipple D) At the eighth intercostal space

B) When assessing a pediatric client, it may be more beneficial to auscultate the apical pulse in the area of the left nipple at the fourth intercostal space. The other answer options are not appropriate.

An adult client is diagnosed with a degenerative bone disease that is impairing mobility. Based on this information alone, which of the following actions should be the nurse's first priority? A) Implementing a low-level exercise program for the client B) Assessing the client's pain management C) Teaching the client relaxation techniques D) Referring the client to a dietitian

B) When caring for a client with a degenerative bone disease that is impairing mobility, the nurse should assess pain management prior to implementing an exercise program, teaching relaxation exercises, or referring to a dietitian.

The nurse is planning care for a client with osteoarthritis. Which nursing diagnosis would have the highest priority? A) Fatigue B) Chronic Pain C) Ineffective Coping D) Disturbed Body Image

B) When providing care to a client diagnosed with osteoarthritis, priority diagnoses would include Chronic Pain, Impaired Physical Mobility, and Self-Care Deficit. Thus, of the diagnoses identified for this client, Chronic Pain would be the highest priority. Once this diagnosis has been addressed, the nurse and client can focus on the lower priority diagnoses of Fatigue, Ineffective Coping, and Disturbed Body Image.

List the pathophysiologic processes involved in appendicitis in sequential order. A) The appendix becomes distended with fluid secreted by its mucosa. B) The proximal lumen of the appendix becomes obstructed. C) Purulent exudate forms and causes further distention of the appendix. D) Pressure within the lumen of the appendix increases. E) Tissue necrosis occurs and gangrene develops.

B, A, D, C, E Obstruction of the proximal lumen of the appendix is apparent in most acutely inflamed appendices. Following obstruction, the appendix becomes distended with fluid secreted by its mucosa. Pressure within the lumen of the appendix increases, impairs its blood supply, and leads to inflammation, edema, ulceration, and infection. The purulent exudate formed causes further distention of the appendix. If treatment is not initiated, tissue necrosis and gangrene result within 24-36 hours, leading to perforation (rupture).

A nurse is caring for a client with seasonal hypersensitivity reactions. What teachings should the nurse provide to improve this client's comfort? Select all that apply. A) Keep doors and windows open on high-allergen days to circulate air. B) Remain indoors if possible on high-allergen days. C) Maintain a clean, dust-free environment. D) Take antihistamine and leukotriene medications as ordered. E) Stop taking oral corticosteroids immediately once symptoms disappear.

B, C A client with seasonal hypersensitivity should be educated regarding prevention and comfort measures. The nurse should instruct the client to keep doors and windows closed on high-allergen days and to remain indoors if possible. The nurse should also include teaching on maintaining a clean, dust-free environment. Medication instruction should include information about taking antihistamine and antileukotriene medications, not leukotriene medications. The client should also be instructed to taper oral corticosteroids as ordered, not to immediately stop taking them.

The nurse is caring for a client who is hospitalized for cellulitis of the foot. Which nursing diagnoses should the nurse use to plan this client's care? Select all that apply. A) Social Isolation related to skin infection B) Impaired Skin Integrity related to skin infection C) Acute Pain related to skin infection D) Disturbed Sleep Pattern related to skin infection E) Powerlessness related to inability to control the infection

B, C Clients with cellulitis have Impaired Skin Integrity and will also have Acute Pain at the site. Controlling pain is a priority. Disturbed Sleep Pattern, Social Isolation, and Powerlessness are not supported by the information given

The nurse is preparing to triage victims of a train derailment who are being transported to the emergency department. Which victims would need immediate care? Select all that apply. A) Holding broken arm, sitting in a chair B) Respiratory rate of 8 and irregular C) Bleeding from fractured limb with a blood pressure of 78/40 mmHg D) Bleeding from superficial facial wounds and talking to family E) Walking with a slight limp, asking for something to drink

B, C Emergent or immediate care is needed for life-threatening issues that require prompt treatment and care. Stabilization of the client's condition is critical. A respiratory rate of 8 and a blood pressure of 78/40 mmHg would be emergent. Urgent or delayed care is for serious health conditions in which a delay of treatment and care would not result in life-threatening situations. Holding a broken arm sitting in a chair and bleeding from superficial facial wounds while talking with family would be urgent. Nonurgent or minor issues do not require prompt care. Many of these clients can ambulate and are stable in their conditions. The client walking with a limp and asking for something to drink would be nonurgent.

The nurse is instructing a client on lifestyle changes to promote a healthy cardiovascular system. Which of the following should be included in this teaching session? Select all that apply. A) Limit exercise to 15 minutes a day B) Reduce saturated fats in the diet C) Avoid cigarette smoking D) Wear elastic hose E) Limit fluid intake

B, C Interventions that help promote a healthy cardiovascular system are to avoid cigarette smoking and reduce saturated fats in the diet. Clients should exercise for at least 30 minutes most days of the week to maintain a healthy cardiovascular system. Wearing elastic hose and limiting fluid intake are not known to contribute to a healthy cardiovascular system

The nurse is prioritizing care needed for a group of clients according to urgency. Which care should the nurse identify as being medium priority? Select all that apply. A) Instructing on changing ostomy appliance B) Performing passive range of motion every 4 hours C) Removing splints and providing complete skin care every 2 hours D) Administering 2 units of fresh frozen plasma E) Performing endotracheal suction

B, C Medium-priority interventions are those that would not cause a life-threatening situation if performed later throughout the shift. These interventions include passive range of motion exercises and removing splints for skin care. Instructing on ostomy appliance changes is a low-priority intervention. Administering fresh frozen plasma and endotracheal suctioning would be considered critical interventions because if these interventions were not performed, the client's health status could quickly deteriorate.

Which statements accurately reflect the distinction between nursing diagnoses and medical diagnoses? Select all that apply. A) A nursing diagnosis is determined following an assessment and analysis of data gathered only by registered nurses; a medical diagnosis is determined following an assessment and analysis of data gathered only by physicians. B) A nursing diagnosis changes as the client's responses to an illness or health situation change; a medical diagnosis remains the same as long as the disease process persists. C) A nursing diagnosis describes a client's physical, sociocultural, psychological, and spiritual responses to an illness or health condition; a medical diagnosis refers to disease processes. D) A nursing diagnosis considers the etiology of the health problem to give direction to required nursing care; a medical diagnosis does not consider the etiology of the health problem to give direction to medical care. E) A nursing diagnosis requires the nurses to consider standards and norms as well as cues from clients in discerning an appropriate nursing diagnostic label; a medical diagnosis uses standards and norms only.

B, C Nursing diagnoses consider physical, psychosocial, and spiritual responses to illnesses or conditions, whereas medical diagnoses are concerned primarily with disease processes. Nursing diagnoses change as the client's response to the illness or situation changes, whereas medical diagnoses remain in place as long as the disease process persists. Both nursing and medical diagnoses involve considering client cues as well as standards and norms. Both consider etiology (e.g., "Viral Pneumonia" and "Disturbed Sleep Pattern related to frequent coughing"). Both nursing and medical diagnoses are not only based on information gathered, respectively, by the nurse and physician directly during their assessments, but often include data from other sources.

The nurse is caring for a client who is hospitalized on a medical unit for a systemic infection. The client asks the nurse which defenses the body has against infection. Which physiologic barriers that protect the body against microorganisms should the nurse include in the response to the client? Select all that apply. A) The spleen B) Adequate urinary output C) Intact skin D) Generalized inflammation E) The thymus gland

B, C Physiologic barriers include adequate urinary output and intact skin. The act of voiding flushes out organisms that might try to access the body through the urinary meatus. Intact skin is a physiologic barrier that prevents a variety of microorganisms from entering the body. Although the inflammatory response helps fight infection, it occurs when the body's physiologic barriers to microorganisms have already been breached. Similarly, while the spleen and thymus gland are part of the immune system, they are not physiologic barriers but rather internal organs in which lymphoid cells proliferate and/or mature.

The nurse is preparing to examine a toddler's ear canals with an otoscope. Which actions by the nurse are appropriate? Select all that apply. A) Having the child sit on the examination table B) Having the child play with the equipment C) Having the child sit on the parent's lap D) Telling the child the examination will not hurt E) Asking the child to tilt the head

B, C The best way to get a child of this age to cooperate is to let the child play with the equipment and perhaps use the equipment on a doll, as well as to have the child sit on the parent's lap. Asking the child to tilt the head does not encourage cooperation. Most children of this age need to see for themselves that a procedure will not hurt; simply telling the child that the procedure won't be painful is not effective.

The nurse is caring for a client with hypertension. Which diagnostic tests should the nurse anticipate being ordered to rule out secondary causes? Select all that apply. A) Cerebral angiogram B) Intravenous pyelogram C) Renal ultrasonography D) Cardiac catheterization E) Myelogram

B, C When secondary hypertension is suspected, diagnostic tests include an intravenous pyelogram and renal ultrasonography to determine if the renal system is the cause of the hypertension. Cerebral angiogram, cardiac catheterization, and myelogram are not diagnostic tests to determine the cause for secondary hypertension.

A nurse educator is teaching a group of students about professionalism. The educator informs the students that a profession is distinguished from other kinds of occupations by a number of characteristics. Which of the following are among those characteristics? Select all that apply. A) The members of a profession are financially liable for their actions. B) The members of a profession participate in ongoing research. C) The members of a profession must acquire specialized education. D) The members of a profession possess autonomy. E) The members of a profession regularly socialize with one another.

B, C, D A profession is generally distinguished from other kinds of occupations by its requirement of prolonged, specialized training to acquire a specific body of knowledge; its emphasis on service to others; its support of ongoing research to expand the profession's body of knowledge; its development of a code of ethics; the autonomy of its members; and the existence of a professional organization. Financial liability and socialization with other nurses are not among the criteria that distinguish nursing as a profession.

The nurse is caring for a client who is experiencing limited mobility related to a musculoskeletal alteration. Which laboratory tests would be useful to diagnose the client appropriately? Select all that apply. A) Magnetic resonance imaging (MRI) B) Alkaline phosphatase (ALP) C) Human leukocyte antigen-B27 (HLA-B27) D) Rheumatoid factor (RF) E) Electromyography (EMG)

B, C, D ALP, HLA-B27, and RF are all laboratory tests that are used to diagnose clients with musculoskeletal disorders that can cause alterations in mobility. ALP is produced by bone and other organs. Increased ALP may indicate bone disease, bone fracture, bone tumors, osteomalacia, Paget disease, or rickets. Decreased ALP may indicate Wilson disease. The presence of HLA-B27 indicates an increased risk for ankylosing spondylitis and arthritis. Elevated levels of RF may indicate rheumatoid arthritis, scleroderma, lupus erythematosus, and adult Still disease. MRI and EMG are both diagnostic, not laboratory, tests use to diagnose the cause of alterations in mobility.

The nurse is developing a plan of care for a client who is at risk for falls. Which interventions would be appropriate for the nurse to include in the plan of care? Select all that apply. A) Apply physical restraints if the client gets out of bed. B) Assess the client's vision and make sure he is using any prescribed eyewear. C) Use side rails on client beds. D) Keep frequently used items within easy reach. E) Provide slippers for the client to wear while ambulating.

B, C, D Assessing the client's vision and making sure he is using any prescribed eyewear is an appropriate action. Poor and blurry vision increases the client's risk of falling. Using side rails on the client bed to prevent falls while the client is sleeping is an appropriate intervention. Furthermore, it is appropriate to keep frequently used items within easy reach in an effort to prevent falls. It is not appropriate for the nurse to apply physical restraints if the client gets out of bed. The nurse should ensure that the client wears shoes with adequate traction while ambulating. Slippers may increase the risk for falls.

The nurse is caring for an adult client with a BMI of 26.8 who complains of sleep apnea and gout. The nurse anticipates that treatment of this patient for obesity will consist of which therapies? Select all that apply. A) Pharmacotherapy B) Diet C) Exercise D) Behavior modification E) Surgery

B, C, D Clients with a BMI of 25-26.9 with two or more comorbidities-sleep apnea and gout in this case-would be treated with the therapies of diet, exercise, and behavior modification, but likely not pharmacotherapy or surgery

A client is recovering from surgery to repair a fractured hip. Which actions by the nurse may reduce this client's risk for osteomyelitis in the postoperative period? Select all that apply. A) Assess for pain every 1-2 hours. B) Use sterile technique for dressing changes. C) Assess wound for size, color, and drainage. D) Administer antibiotics as prescribed. E) Administer anticoagulants as prescribed.

B, C, D Interventions that can reduce the client's risk for infection include using sterile technique for dressing changes; assessing the wound for size, color, and drainage; and administering antibiotics as prescribed. Assessing for pain every 1-2 hours is appropriate for the nursing diagnosis of Acute Pain, but it does not help reduce the risk of osteomyelitis. Administering anticoagulants per order is appropriate for the client who is at risk for deep vein thrombosis (DVT), but again, it does not help reduce the risk of osteomyelitis.

Which actions should the nurse take to help the client with bowel and bladder dysfunction reduce the risk of infection? Select all that apply. A) Isolate the client using transmission-based precautions. B) Monitor intake and output. C) Provide hygienic care after episodes of incontinence. D) Use standard precautions when handling linen after episodes of incontinence. E) Limit fluid intake

B, C, D Monitoring intake and output is important because it can help reveal whether a client is experiencing dehydration. Dehydration may lead to urinary stasis, which increases the risk of infection. Similarly, intake and output levels can help reveal urinary retention, which also heightens the risk of infection. Providing hygienic care after episodes of bowel or bladder incontinence will ensure that the skin remains intact, reducing the risk of infection. Using proper biohazard precautions after episodes of incontinence will also reduce the risk of transmitting an infection. In contrast, limiting fluid intake increases infection risk by putting the client at greater risk for dehydration. Isolating the client using transmission-based precautions is not necessary because these precautions are meant to prevent the spread of infection from the client to others, and this client is not currently experiencing infection.

The nurse is providing care to a client who is diagnosed with stress incontinence. Which data would nurse expect to collect during the client's health history and physical assessment? Select all that apply. A) Urine leakage while talking B) Urine leakage while coughing C) Urine leakage while laughing D) Skin breakdown on the buttock E) A urinary catheter

B, C, D Stress incontinence involves a small leakage of urine when a client laughs, coughs, or lifts something heavy, not if a client just carries on a conversation. A client with incontinence would wear some kind of undergarment pad; a urinary catheter is not an expected finding. If the client has been experiencing incontinence, the nurse might expect to see the skin inflamed and irritated because urine is very irritating to the skin

The nurse is concerned that a client diagnosed with a fluid imbalance is at risk for an alteration in perfusion. Which assessment data should indicate to the nurse that the client is not currently experiencing an alteration in perfusion? Select all that apply. A) Skin turgor 20 seconds B) Peripheral pulses present and full C) Capillary refill of nail beds 3 seconds D) Oriented to person, place, and time E) Bowel sounds sluggish in all four quadrants

B, C, D To determine whether the client's perfusion status is being affected, the nurse should assess pulses, nail beds, and orientation. Full and present peripheral pulses, capillary refill of 3 seconds or less, and orientation to person, place, and time indicate that the client's perfusion status is being maintained. Skin turgor and bowel sounds would be used to determine whether the fluid imbalance is affecting the client's elimination status. Skin turgor that takes 20 seconds to return to normal and sluggish bowel sounds indicate that the fluid imbalance is affecting the client's elimination status.

The school nurse cares for students with physical challenges and suspects that the needs for physical safety are not being adequately met for several students in the home environment. Which of the following assessment findings support the nurse's concern? Select all that apply. A) Wearing the same clothes to school several days of the week B) Limited arm range of motion C) Scrapes on knees caused by falling from a bicycle D) Hand burn from touching a hot stove E) Lunch contains leftovers from previous evening dinner

B, C, D Wearing the same clothes to school several days of the week and eating leftovers for lunch could be an indication of the family's financial status. Evidence that physical safety needs are not being adequately met would include the formation of contractures, which limits arm range of motion; scrapes on knees after falling from a bicycle, indicating the lack of safety or protective equipment for the head and extremities; and hand burns obtained from touching a hot stove, which could mean the child was not attended in the kitchen at home.

A home healthcare nurse is providing care to an older adult client who lives alone and has limited financial resources. The client has a history of celiac disease. When planning care for this client, which nursing diagnoses are appropriate? Select all that apply. A) Risk for Constipation B) Imbalanced Nutrition: Less than Body Requirements C) Risk for Imbalanced Fluid Volume D) Diarrhea E) Chronic Pain

B, C, D, E Client with celiac disease often have nutritional imbalance, including anemia and vitamin deficiencies; impaired absorption of fluids and electrolyte, which leads to diarrhea and fluid imbalance; and pain related to abdominal bloating and cramping. Constipation is not a normal manifestation of celiac disease.

The nurse is providing teaching to a client who is scheduled to undergo surgery in 2 weeks. Which topics should the nurse include that will prepare the client to help reduce complications during the postoperative phase? Select all that apply. A) Maintaining a patent airway B) Deep breathing and coughing C) Caring for the surgical incision D) Managing constipation E) Managing pain

B, C, D, E Maintaining a patent airway is a nursing action that is performed during and after surgery; the client would not need client teaching about how to maintain a patent airway. In the preoperative phase, when the client is alert and oriented, the nurse should focus teaching on deep breathing and coughing exercises, care of the surgical incision, managing constipation, and managing pain. This knowledge will help the client reduce complications after the surgery

The nurse is caring for a client who is exhibiting signs of a systemic infection following surgery. Which diagnostic tests would the nurse anticipate being ordered? Select all that apply. A) Serum electrolyte levels B) Urinalysis C) White blood cell differential D) White blood cell count E) Wound culture

B, C, D, E Serum electrolyte levels are not used to determine the presence of a systemic infection. Urinalysis is used to assess for the presence of bacteria or blood in the urine. An elevated WBC and 15% bands are indicative of an infection. Wound cultures are used to identify probable microorganisms.

The nurse is preparing educational materials for a client with hypertension. Which of the following elements should the nurse include when preparing this material? Select all that apply. A) Advising the client to avoid all sodium in the diet B) Explaining the effects of sodium on blood pressure C) Teaching the client how to read nutritional labels D) Helping the client to recognize foods that are low in sodium E) Showing the client how to follow the DASH eating plan

B, C, D, E Sodium is important in the diet, and not all sodium should be avoided; however, consumption of sodium should be according to the USDA dietary guidelines for recommended intake. In some people, an increase in sodium intake leads to fluid retention and increases blood pressure. The nurse should prepare to teach the client how to read nutritional labels, how to identify foods that are low in sodium, and the DASH eating plan.

The nurse is preparing a client for emergency surgery to repair liver and colon lacerations caused by a motor vehicle crash. Which information about this type of surgery will the nurse use to guide the client's care? Select all that apply. A) An organ is going to be removed. B) This is an emergency surgery. C) The client will be hospitalized longer. D) The client is at risk for blood loss. E) The client is at risk for hypothermia

B, C, D, E The client is having surgery to repair lacerations. No organ is identified for removal. Emergency surgery is performed when a condition is life threatening. Surgery to control internal hemorrhage from lacerations is an example of emergency surgery. An open procedure usually requires a longer hospital stay. Open procedures place the client at a higher risk for blood loss. Larger incisions place the client at a higher risk for complications, such as hypothermia and surgical site infections (SSIs).

The community health nurse reviews data collected during interviews with community members during a health fair and decides to create a brochure on how to improve iron intake. Which of the following action items might the nurse include that would help vegans and vegetarians increase their iron intake? Select all that apply. A) Take calcium supplements. B) Consume tofu. C) Consume lentils. D) Increase intake of vitamin C. E) Consume Swiss chard.

B, C, D, E Vegan diet plans can lead to deficiencies in certain nutrients, including iron. All vegetarians should ensure that they get adequate amounts of iron, and to facilitate the absorption, vitamin C should also be plentiful in the diet. Tofu, lentils, and Swiss chard are all foods that both vegans and vegetarians might eat that can provide needed iron. However, although calcium supplements are good for vegetarians and vegans to take to ensure adequate calcium intake, they do not specifically provide iron.

The nurse is assessing a client who has recently had several blood tests done. Which laboratory results would the nurse identify as indicators of malnutrition? Select all that apply. A) PAB 30 mg/dL B) PAB 100 mg/L C) Cholesterol 120 mg/dL D) Cholesterol 180 mg/dL E) PAB 12 mg/dL

B, C, E A normal prealbumin (PAB) level for adult men and women is 15-36 mg/dL or 150-360 mg/L. A 30 mg/dL PAB level would not be a cause for concern, but a 100 mg/L or 12 mg/dL level would be. A normal cholesterol range is between 160 and 200 mg/ dL in adult men and women. A 120 mg/dL cholesterol level would be a possible indicator of malnutrition, but a 180 mg/dL level would not be.

The nurse is caring for an older adult. Which age-related changes should the nurse identify as increasing the risk of dry skin? Select all that apply. A) Reduction in elastin B) Depleted moisture in epidermal cells C) Decreased size of sebaceous glands D) Thinner subcutaneous skin layer E) Poor nutrition

B, C, E As the individual ages, moisture transfer from the dermis to the epidermis declines. This contributes to a dry, rough skin appearance. Sebaceous glands also decrease in size with age, resulting in skin that is dry and easily bruised, damaged, or broken. Poor nutrition could also cause dry skin. Reduction in elastin leads to wrinkling and sagging of the skin. The older adult's thinner subcutaneous skin layer increases the risk for hypothermia and pressure ulcer formation.

During a health assessment, a client states, "I only eat carbohydrates and low-fat foods. I don't understand why I am still gaining weight!" Which principles of nutrition should guide the nurse's response? Select all that apply. A) Carbohydrates should only be eaten at breakfast. B) Eating too many carbohydrates leads to excess glucose, which is converted to fat. C) Excess carbohydrates can lead to obesity. D) A carbohydrate-limited diet is the only way to not gain weight. E) Carbohydrates should be high in fiber and low in sugar.

B, C, E Carbohydrates should be eaten throughout the day. Carbohydrates are converted to glucose; when carbohydrates are consumed in excess, the excess glucose is converted to glycogen, or fat, and stored in adipose tissue, which can lead to weight gain and an increased risk for obesity. Carbohydrate deficiencies lead to protein tissue wasting. Carbohydrates should come from the consumption of foods high in fiber and low in added sugars. A carbohydrate-limited diet is not the only way to avoid weight gain.

A client has been prescribed iron supplements to treat fatigue and a hemoglobin level of 9. What additional information should the nurse suggest to the client to help reduce feelings of fatigue associated with a low red blood cell count? Select all that apply. A) Increasing intake of ice cream B) Drinking a glass of orange juice each day C) Increasing intake of red meat D) Increasing intake of wax beans E) Increasing intake of green, leafy vegetables

B, C, E Iron is absorbed better when accompanied with vitamin C in foods such as orange juice. Green, leafy vegetables and red meat are good sources of iron that the client can consume in a healthy diet. Ice cream and wax beans are not sources of dietary iron and do not enhance the absorption of iron.

A client is admitted to the hospital with airway edema, bronchoconstriction, and increased mucus production after being exposed to an allergen. Which nursing interventions are appropriate to address this inflammation to the client's respiratory system? Select all that apply. A) Turn and reposition every 2 hours. B) Monitor oxygen saturation. C) Administer oxygen as prescribed. D) Restrict fluids. E) Monitor lung sounds.

B, C, E Turning and repositioning every 2 hours would be appropriate to maintain tissue integrity but not to address respiratory inflammation. In contrast, monitoring oxygen saturation, administering oxygen, and monitoring lung sounds would all be appropriate care for a client who is experiencing inflammation of the respiratory system. Restricting fluids could cause respiratory secretions to thicken and hinder the client's ability to maintain a clear airway. Fluids should be encouraged.

The nurse is caring for a client who is experiencing intermittent constipation. The client has been advised to increase the amount of dietary fiber. Which food selections by the client indicate that teaching has been effective? Select all that apply. A) Rice B) Carrot slices C) Spinach salad D) Bananas E) Peas

B, C, E Dietary fiber increases stool bulk. Vegetable fiber—such as that in carrot slices, spinach, and peas—is an excellent source of dietary fiber. The remaining selections are examples of constipating foods

The nurse is caring for a client with functional incontinence. Which conditions are factors in the development of this type of incontinence? Select all that apply. A) Fecal impaction B) Dementia C) Confusion D) Prostate surgery E) Impaired mobility

B, C, E Functional incontinence occurs when the ability to respond to the need to urinate is impaired. Contributing factors may include confusion, dementia, or impaired mobility. Fecal impaction is a contributing factor to overflow incontinence and prostate surgery is a contributing factor to stress incontinence.

The healthcare provider prescribes a client to have peak and trough blood levels drawn to evaluate the therapeutic effect of an IV antibiotic. When should the nurse schedule the blood samples to be drawn? Select all that apply. A) Prior to the discontinuing the antibiotic B) A few minutes before the next scheduled dose of medication C) During the infusion of the antibiotic D) 30 minutes after the IV administration E) 1 to 2 hours after the oral administration of the medication

B, D Antibiotic peak and trough levels monitor therapeutic blood levels of the prescribed medication. The therapeutic range-that is, the minimum and maximum blood levels at which the drug is effective-is known for a given drug. By measuring blood levels at the predicted peak (1-2 hours after oral administration, 1 hour after intramuscular administration, and 30 minutes after IV administration) and trough, usually a few minutes before the next scheduled dose, it is also possible to determine whether the drug is reaching a toxic or harmful level during therapy, increasing the likelihood of adverse effects.

A nurse is providing care for a client who has vocal cord damage and wants to implement strategies that will promote communication with this client. Which interventions would be appropriate? Select all that apply. A) Facing the client when speaking B) Having pen and paper on hand for the client C) Making sure that the language spoken is the client's dominant language D) Using a picture board to facilitate communication E) Employing an interpreter

B, D The client who is nonverbal would respond best to use of a picture board or pen and paper. Because the client cannot communicate verbally, facing the client when talking, using an interpreter, or using the client's dominant language would not address the client's inability to communicate.

A client who has just been diagnosed with type 2 diabetes mellitus is being instructed by the nurse regarding diet and exercise. Which client statements indicate that further teaching is required? Select all that apply. A) "I should talk to the doctor about an exercise program." B) "I don't need to watch my diet as long as I take my insulin." C) "I need to limit the amount of fat in my diet." D) "I should eat a candy bar when my energy is low." E) "I will test my blood sugar before meals and at bedtime."

B, D The nurse would revise the plan and add more teaching if the client states that candy is necessary for energy and that no dietary planning is needed as long as the client takes insulin. Testing blood sugars before meals and at bedtime is appropriate understanding by the client. Every client is different, and the client should consult the physician regarding starting an exercise program. Limiting fat in the diet is encouraged for this client.

The nurse is caring for an older adult client with gallbladder disease who is recovering from a cholecystectomy. Which risk factors increase this client's susceptibility to infection? Select all that apply. A) Dry skin B) Advanced age C) Intact mucous membranes D) Nonintact skin E) Active bowel sounds

B, D This client is more susceptible to infection due to advanced age and the presence of a surgical incision. A surgical incision indicates that the body's first line of defense, the skin, is not intact. Active bowel sounds, dry skin, and intact mucous membranes are all factors that help defend the body against infection.

A school nurse in a large urban high school regularly delivers presentations on nutrition, smoking cessation, and prevention of sexually transmitted infections (STIs). What area(s) of nursing competence is this nurse demonstrating? Select all that apply. A) Health restoration B) Health and wellness promotion C) Caring for the dying D) Illness prevention E) Care cost savings

B, D This nurse is demonstrating competence in health and wellness promotion, because these programs enable students to increase their control over the determinants of health and thus improve their health. The nurse is also demonstrating competence in illness prevention in that the programs are designed to maintain optimal health by preventing disease. Health restoration focuses on ill clients, which is not the case in this situation. Similarly, caring for the dying involves helping clients live as comfortably as possible until death, and it is not applicable in this situation. Care cost saving is not a core area of nursing competence

The nurse is providing care to an adolescent client who presents at the clinic for a routine health assessment. Which immunizations should the nurse anticipate administering to the client during this visit? Select all that apply. A) Herpes zoster vaccine B) Papillomavirus vaccine C) Rotavirus vaccine D) Meningococcal vaccine E) Hepatitis B vaccine

B, D The papillomavirus and meningococcal vaccines are administered during adolescence. In contrast, the herpes zoster vaccine is typically given to older adults, while the rotavirus and hepatitis B vaccines are usually administered to infants.

A nurse educator is providing information to a group of nursing students regarding appropriate assessment techniques that can be applied across the life span. Which statements should the educator include in the teaching session? Select all that apply. A) "Auscultate the chest while the client is sleeping to obtain the most accurate assessment of the heart." B) "Use standard precautions during the history and physical examination process." C) "Perform invasive procedures like pharyngeal and otic exams at the end of the assessment." D) "Use age-appropriate terminology for explaining procedures and actions." E) "Use the assessment process to teach about exam procedures and findings."

B, D, E Following standard precautions, employing age-appropriate terminology, and using the examination to provide teaching are all actions that the nurse can implement across the life span when assessing clients. Conducting a cardiovascular assessment during sleep is appropriate for some pediatric clients, but it would be considered an intrusion or violation of privacy by older pediatric and adult clients. Performing invasive procedures at the end of the assessment is appropriate for pediatric clients but not necessary for adult clients.

A hospital's nursing staff is deciding whether to unionize. The hospital CFO tells a nurse supervisor to make sure all leaders of the unionization effort are scheduled for client care during union meetings. Which actions by the nurse supervisor are appropriate? Select all that apply. A) Schedule the organizers for client care during union meeting times. B) Schedule the organizers for client care according to clinical staffing needs. C) Reprimand the staff nurses for their attempts to unionize. D) Continue to implement the usual staffing procedures. E) Discuss the need for professional nursing integrity with the CFO.

B, D, E In some cases, administrators may serve their careers and no longer the priorities of client-centered care. This can lead to potential abuses of power, such as the one described in this situation. Nurses who are in positions of authority have a duty to address such abuses, thereby maintaining their integrity and that of their profession. They can do this by taking steps to emphasize principles over personality and by focusing on client needs over political ends. In this situation, examples of such behavior would include following regular staffing procedures that reflect client care needs, as well as discussing the need for professional nursing integrity with the CFO.

The postoperative care nurse is reviewing the documentation from the intraoperative phase of a client's surgical procedure. Which information should the nurse anticipate finding on the intraoperative documentation? Select all that apply. A) Pain assessment B) Start and stop times of anesthesia C) Medication review D) Antibiotic infusion times E) Start and stop times of the procedure

B, D, E Intraoperative documentation is to include documentation about specific times, such as the start and stop times of anesthesia, antibiotic infusion times, and start and stop times of the procedure. The pain assessment and medication review are documented during both the preoperative and postoperative assessments.

A client begins to vomit blood. The nurse immediately measures the blood pressure and prepares to insert a nasogastric tube while directing others to notify the healthcare provider and prepare to perform iced saline lavage. Which features of the Tanner Clinical Judgment Model did this nurse demonstrate? Select all that apply. A) Presencing B) Noticing C) Reflecting D) Interpreting E) Responding

B, D, E The four features of the Tanner model include noticing, interpreting, responding, and reflecting. The nurse noticed that something was wrong with the client, interpreted the cues to gain an understanding of the situation, and responded by choosing the best course of action. The nurse has not yet reflected on the situation because she is still caring for the client. Presencing is being present for a client and engaging in face-to-face contact, which is not part of the Tanner model.

The nurse notes that an infant does not seem to respond to noises in the environment and has difficulty following the movement of toys. Which diagnostic tools does the nurse anticipate will be used to further assess this client? Select all that apply. A) CT scan of the brain B) Vision test C) Abdominal x-rays D) Nerve conduction studies E) Audiology testing

B, E The child is not responding to environmental noise and is having difficulty tracking the movement of toys. The two tests that should be considered for this child are vision and hearing testing. There is not enough evidence to support that the child would need a CT scan of the brain, abdominal x-rays, or nerve conduction studies

The son of a client with fibromyalgia asks the nurse if he will also experience the health problem. Which responses by the nurse are appropriate for this situation? Select all that apply. A) If your diet is high in fatty foods, you have a greater chance of developing fibromyalgia. B) Having a family member with fibromyalgia increases the risk for developing it. C) If you exercise often enough you'll be fine. D) Only people aged 20-50 develop fibromyalgia. E) Fibromyalgia is more prominent in women.

B, E There are many theories about the causes and risk factors about fibromyalgia. It is generally conceded that it is more of a problem for women aged 20—50 than for men. Having a family member with the disease also increases the risk for development. The disease has not been linked to lack of exercise. Diets have been suggested as possible treatment but are not thought to cause the disease.

The nurse is prioritizing care for a client with several problems. List the order in which the nurse should address the client's needs. A) Bleeding through nasogastric tube B) Audible wheezes C) Not understanding how to complete the menu D) Requesting medication for arthritis pain E) Dyspnea F) Asking questions about teaching provided the other day

B, E, A, D, F, C The nurse's priority should follow the ABCs, or airway, breathing, and circulation. Audible wheezes could indicate an obstruction and should be attended to first. Dyspnea indicates a problem with breathing and should be addressed next. Bleeding could impact circulation and should be addressed third. Pain control should be addressed fourth. Reinforcement of teaching can occur fifth, and helping with menu completion can occur last.

The nurse is caring for an older client. Which early sign of a fluid volume deficit should the nurse identify in this​ client? A. Dry skin B. Change in mental status C. Brittle hair D. Poor skin turgor

B. ​Rationale: Change in mental status or mentation is an early sign of FVD in the older adult. Skin turgor can be difficult to assess due to normal changes with aging. Dry skin and brittle hair are signs of chronic dehydration.

A client is prescribed furosemide. Which information should the nurse provide about this​ medication? A. decreased potassium in the diet B. Check weight daily C. Take the medication at bedtime D. Increase sodium intake

B. ​Rationale: Daily weight is recommended for a client taking furosemide. Increasing sodium intake and decreasing potassium intake can lead to fluid and electrolyte imbalances. It would be recommended to take furosemide in the morning due to the diuresis effect of the medication.

The nurse is caring for a client with suspected fluid volume excess. Which change in the serum osmolality should the nurse use as confirmation of this health​ problem? A. Slight increase B. Remains the same C. Large increase D. Slight decrease

B. ​Rationale: During fluid volume​ excess, the body retains both sodium and water. This causes the blood serum to remain isotonic and serum osmolality to remain the same.

The nurse is evaluating the laboratory work of a client who is receiving replacement therapy for hypokalemia. Which value should the nurse identify that evaluates the effectiveness of the replacement​ therapy? A. Serum calcium 9.2​ mEq/L B. Serum potassium 4.2​ mEq/L C. Serum chloride 100​ mEq/L D. Serum potassium 2.3​ mEq/L

B. ​Rationale: Hypokalemia is a potassium level less than 3.5​ mEq/L. A serum potassium of 4.2​ mEq/L indicates improvement in hypokalemia. Serum chloride and serum calcium are not used to evaluate potassium level.

The nurse reviews the care needs for a group of clients. Which condition should the nurse realize causes a fluid volume​ deficit? A. Hypertension B. Diarrhea C. Water intoxication D. Kidney failure

B. Diarrhea​ Rationale: Fluid volume​ deficit, or​ dehydration, can occur when excessive amounts of fluids are lost through diarrhea or vomiting. Kidney failure causes water​ retention, leading to fluid volume​ excess, not deficit. Water intoxication results from excessive fluid intake and leads to fluid volume excess. Fluid volume​ excess, not​ deficit, can result in hypertension.

The nurse is reviewing the fluid needs for a group of clients. Which characteristic of the intracellular fluid compartment of the body should the nurse​ identify? A. Includes cerebrospinal and peritoneal fluids B. Serves as a medium for metabolic processes C. Makes about one third of total body fluid in adults D. Divides into intravascular, interstitial, and transcellular fluids

B. ​ Rationale: The intracellular fluid compartment makes up about two thirds of total body fluid in adults and is found within cells. It is a medium for metabolic processes. Extracellular fluid makes up the other one third of total body fluid and is divided into​ intravascular, interstitial, and transcellular fluids. Cerebrospinal and peritoneal fluids are examples of transcellular fluids.

An adult client and her spouse are seen in an urgent care clinic. The client presents with a temperature of 102°F, complains of nausea, and has experienced vomiting and diarrhea for 12 hours. The nurse notes that the client's mucous membranes are pale and dry and suspects that the client is dehydrated. Which action by the nurse is the most appropriate? A) Ask the spouse for more information. B) Assess for pedal edema. C) Assess skin turgor. D) Repeat the temperature measurement.

C) A client who presents with hyperthermia, vomiting, diarrhea, and pale, dry mucous membranes is likely dehydrated and requires assessment to confirm this suspicion. An appropriate action by the nurse is to assess the client's skin turgor, which can provide more support for a diagnosis of dehydration. Pedal edema would indicate fluid volume overload and not dehydration. Asking the spouse for more information will not provide adequate support for the treatment of dehydration. Only measurements that are extremely abnormal need to be repeated in stable clients.

A nurse is caring for a client with who is experiencing leukocytosis. When providing care to this client, which action by the nurse is most appropriate? A) Instructing the client on the use of an electric razor and soft toothbrush B) Evaluating the client for bleeding and bruising C) Assessing the client for the source of infection D) Placing the client in reverse isolation

C) A client with leukocytosis has a white blood cell (WBC) count that is elevated above normal (>10,000 mm3), which is an indication of infection. The appropriate action by the nurse is to assess the client for the source of infection. Providing instruction on the use of an electric razor and soft toothbrush and evaluating for bleeding and bruising would be appropriate actions for a client with thrombocytopenia, or decreased platelet levels. Reverse isolation would be appropriate for a client with neutropenia, or a decrease in the number of neutrophils.

A type of infection that is associated with the delivery of healthcare services in a facility such as a hospital or nursing home is called a(n) A) etiologic infection. B) latent infection. C) healthcare-associated infection. D) hospital-associated infection.

C) A healthcare-associated infection, not a hospital-associated infection, is an infection that is associated with the delivery of healthcare services in a facility such as a hospital or nursing home. Nurses must be diligent about hand hygiene and other safety practices to prevent healthcare-associated infections. A latent infection is an infection that is present but not active. All infections are etiologic; they are produced by a specific organism.

The pediatric nurse is caring for a 17-year-old client who has type 1 diabetes. What caring intervention can the nurse implement as the client transitions to adulthood? A) Advocate for the client to receive pain-relieving interventions for routine procedures such as blood glucose tests. B) Encourage the client to use a retail clinic for acute care problems unrelated to diabetes. C) Emphasize the importance of having a medical home to routinely monitor and treat chronic conditions. D) Provide recommendations for a provider who can integrate all findings from specialists into a comprehensive overview.

C) A large care gap exists as young adults transition from pediatric care, monitored by parents, into autonomous primary care. Many young adults neglect to get care for chronic childhood conditions such as asthma or diabetes, so nurses should emphasize the importance of having a medical home and routinely monitoring and treating chronic conditions. Because of the need for consistent care and follow-up, a retail clinic would not be beneficial for this client. The use of pain-relieving interventions for routine procedures is more appropriate for infants, not adolescents or young adults. Finding a provider who can integrate all findings from specialists is more important for older adults than for young adults.

A home health nurse is providing care for a client diagnosed with heart failure. The client is taking furosemide (Lasix). The nurse reviews the client's most recent serum potassium, which was 3.4 mEq/L. Based on this data, which food should the nurse encourage the client to consume? A) Baked fish B) Iced tea C) Banana D) Peas

C) A potassium level of 3.4 mEq/L is low, so the client should be encouraged to consume potassium-rich foods. Of the foods listed, banana is the highest in potassium

A client has a pressure injury on the medial malleolus. The client's skin is intact with purple discoloration and a blood-filled blister. When documenting this finding, which terminology is appropriate for the nurse to use? A) Partial-thickness loss of dermis B) Nonblanchable erythema C) Suspected deep tissue injury D) Full-thickness tissue loss

C) A suspected deep tissue injury manifests as intact skin with purple discoloration or a blood-filled blister. Nonblanchable erythema refers to a stage 1 pressure injury. Partial-thickness loss of dermis refers to a stage 2 pressure injury. Full-thickness tissue loss refers to stage 3, stage 4, and unstageable pressure injuries.

The nurse is providing care to a client at a local clinic. The nurse suspects that the client is experiencing a urinary tract infection. Which urinalysis result supports the nurse's suspicions? A) pH 5.2 B) Negative glucose C) WBC 10-15 D) Specific gravity 1.012

C) A urinalysis typically consists of the pH, glucose, specific gravity, protein, and WBC count. The pH, glucose, and specific gravity are all within normal limits. A normal WBC is 3-4. The WBC count for this client is high and indicates infection

A man brings his wife to the clinic and states, "I want you to fix my wife and tell her that there is nothing wrong with her." The client has symptoms of pain, sleep disorders, and stiffness. Which would be most appropriate for the nurse to include in a plan of care for this family? A) Medications used to treat fibromyalgia B) An exercise program to increase energy C) Information and literature on fibromyalgia D) Suggested dietary changes to help with the pain

C) Accepting a disease like fibromyalgia may be difficult due to the vagueness of the disease. Information and written literature may help the family understand that the disease is real. The physician orders medication and diets. There is no proof that exercise, or lack thereof, causes fibromyalgia.

According to Bandura's social learning theory, which of the following should the nurse advise the parent of a preschool-aged child to do in order to help the child become independent in activities of daily living? A) Punish the child if he does not complete the personal care tasks independently. B) Refuse to help the child with the tasks and insist he do them on his own. C) Model the tasks and positively reinforce the child for completing the tasks independently. D) Wait until the child reaches the concrete operational phase before asking him to complete the tasks.

C) According to Bandura, children imitate the behavior they see; if the behavior is positively reinforced they tend to repeat it. Negative reinforcement like punishment is not part of Bandura's theory; it is more closely aligned with Skinner's behaviorist theory. Refusing assistance to the child does not align with Bandura's belief in the need for modeling. The concrete operational phase is part of Piaget's theory and not Bandura's.

A nurse, who has been working in a small rural hospital for 4 years since obtaining a nursing license, participates on an interdisciplinary task force to improve client care. Which skill level is this nurse demonstrating according to Benner's stages of nursing expertise? A) Advanced beginner B) Competent C) Proficient D) Expert

C) According to Benner's stages, this nurse would be considered proficient, because he or she has 3 to 5 years of experience and a holistic understanding of the client, which improves decision making. In comparison, an advanced beginner nurse has less than 2 years of experience and demonstrates marginally acceptable performance; a competent nurse has 2 or 3 years of experience and demonstrates organizational and planning abilities; and an expert nurse has more than 5 years of experience and demonstrates highly skilled intuitive and analytic ability in new situations.

A client presents at the emergency department reporting a burning pain in the chest of a 7 on a 0 to 10 pain scale. Gastroesophageal reflux disorder (GERD) secondary to hiatal hernia is diagnosed. Based on this data, which is the priority nursing diagnosis? A) Dysfunctional Gastrointestinal Motility B) Anxiety C) Acute Pain D) Ineffective Health Maintenance

C) Acute pain management is the priority of nursing care. Anxiety may be decreased by relieving pain. Dysfunctional gastrointestinal motility and ineffective health maintenance are less urgent.

Which client is most likely to reject attempts at comfort? A) An infant crying B) A school-age child with abdominal pain who is anxious about a procedure C) An adolescent with a sleep disorder who doesn't want his parents to be near him D) An older adult with end-stage renal disease

C) Adolescents may respond to treatment and comfort better if you interact with them as adults rather than as children. Some adolescents may reject any offer of comfort, and an adolescent with a sleep disorder who has displayed antagonism toward his parents' presence is probably irritable from his condition and may immediately reject attempts at comfort, at least at first. An infant crying is verbalizing the need for comfort. A school-age child anxious about a medical procedure craves reassurance. An older adult with a terminal illness likely will welcome comfort measures even if she has accepted that she is going to die.

The nurse conducts education for a client who is experiencing urinary incontinence. Which statement by the client indicates the need for further education? A) "Relaxation of pelvic muscles may be a factor in incontinence." B) "Reduced urethral resistance can be a cause of incontinence." C) "Incontinence is normal with aging." D) "A disturbance of my bladder is a factor in the development of incontinence."

C) Age is a risk factor for incontinence, but incontinence is not a normal result of aging. A disturbance of the bladder, relaxation of the pelvic muscles, and reduced urethral resistance are all potential factors in the development of incontinence

The nurse is conducting a class for a group of pregnant clients. Which topics should the nurse include when teaching this group about safety of the fetus? A) Pedestrian accidents B) Suffocation in the crib C) Alcohol consumption D) Drowning

C) Alcohol consumption is a safety hazard for the fetus, and pregnant women should be educated about the importance of not drinking alcoholic beverages while pregnant. Suffocation in the crib is a safety hazard for both newborns and infants. Drowning is seen in toddlers and preschoolers, and pedestrian accidents are seen in the older adult.

What statement made by the client would indicate understanding of discharge teaching for self-care after hospitalization for acute pancreatitis? A) "I will avoid onions, caffeine, and spices." B) "I will take the antibiotics for 2 weeks." C) "I will avoid alcoholic beverages." D) "I will get immunized prior to my vacation."

C) Alcohol increases the risk of pancreatitis. Antibiotics are used to prevent pyloric stenosis. Onions, caffeine, and spices increase the risk of GERD. Immunizations can prevent specific types of hepatitis.

During a home visit, the family of a client with fibromyalgia asks the nurse what they can do to help the client with painful episodes. What should the nurse suggest to the client and family? A) Protect the client from injury. B) Plan a family reunion or vacation. C) Divide household chores among each member of the family. D) Keep the client in bed.

C) Although the causes and treatments are not all known, there is general agreement that reducing stress may help lessen the effects of fibromyalgia. The nurse could help the family by suggesting ways to decrease stress on the client by having the family pitch in on responsibilities. A family reunion or vacation might cause more stress to the client, who would more than likely be planning and packing for such an event. Keeping the client in bed would not be therapeutic. There is no reason to believe that this client is at higher risk for injury than another member of the family.

The nurse is explaining the alteration in normal function to a client recently diagnosed with gastrointestinal reflux disease (GERD). Which etiology contributing to GERD will the nurse include in the teaching session? A) Transient constriction of the lower esophageal sphincter B) Decreased pressure within the stomach C) Incompetent lower esophageal sphincter D) Prolonged constriction of the upper esophageal sphincter

C) An incompetent lower esophageal sphincter remains open, allowing gastric acid to reflux into the esophagus. The lower esophageal sphincter is normally constricted except during swallowing. Increased pressure in the stomach can cause acid to reflux into the esophagus. The action of the upper esophageal sphincter is not a cause of GERD.

Urge incontinence is the involuntary loss of urine associated with a strong urge to void and an increased rate of urination. Which condition can contribute to urge incontinence? A) Weakness of the urethra and surrounding tissue leading to decreased urethral resistance B) Disruption to neuronal control of the sacral micturition centers due to tissue damage C) An overactive detrusor muscle leading to increased pressure within the bladder D) Outlet obstruction leading to the overfilling of the bladder and increased pressure

C) An overactive detrusor muscle leading to increased pressure within the bladder is a contributor to urge incontinence. Weakness of the urethra and surrounding tissue leading to decreased urethral resistance is a contributor to stress incontinence. Disruption to neuronal control of the sacral micturition centers due to tissue damage is a contributor to reflex incontinence. Outlet obstruction leading to the overfilling of the bladder and increased pressure is a contributor to overflow incontinence

A client receiving intravenous antibiotics for 3 days as treatment for cellulitis is being prepared for discharge. Which discharge order should the nurse anticipate for this client? A) Low-sodium diet prescribed B) Home healthcare aide for the client C) Oral antibiotics to be continued at home D) Orders for evaluation by physical therapy

C) Antibiotics should be taken for 10 days; therefore, the nurse anticipates oral antibiotics to be continued at home. A low-sodium diet is not indicated for cellulitis. The client may or may not need a home health aide. There is no evidence to suggest the client needs physical therapy.

The nurse is assessing a client who is receiving IV antibiotics. Which item in the client's health history increases the risk for experiencing a hypersensitivity reaction? A) 26 years of age B) Caucasian race C) Previous antibiotic therapy D) Concurrent chronic illness

C) Anyone can have a hypersensitivity reaction. However, risk generally increases with previous exposure, because antigens must be formed with the first exposure before hypersensitivity is likely to occur. Age, sex, concurrent illnesses, and previous reactions to related substances have all been identified as having a role in risk for hypersensitivity; however, previous exposure presents the greatest risk.

The nurse has implemented a care plan for an adult client with gastroesophageal reflux disorder (GERD). On the next clinic visit, which statement by the client indicates adherence to the plan of care? A) "I can wear snug spandex camisoles if I feel comfortable." B) "I have switched from margaritas to wine." C) "I've lost 6 pounds because I eat every 3 hours and never before bed." D) "I take a Tums with the ranitidine to make it work better."

C) Appropriate client outcomes are freedom from pain and knowledge of lifestyle changes to manage GERD. Weight loss, small, frequent meals, and avoiding lying down within 3 hours of eating indicate correct management. Tight-fitting clothing such as spandex camisoles can worsen GERD, regardless of whether the client feels comfortable wearing them. Changing from margaritas to wine will not improve GERD. Antacids such as Tums should be avoided within 1 hour before or after an H2-receptor blocker like ranitidine.

The nurse is caring for an older adult client on a medical-surgical unit. The client tells the nurse, "I don't get any sleep at night because I have to get up and use the bathroom every couple of hours!" When providing an explanation for the nocturia, which statement by the nurse is the most appropriate? A) "As you get older, there is a decrease in number of nephrons." B) "As you get older, there is a decrease in the blood supply to your bladder." C) "As you get older, you may have a decreased bladder capacity." D) "As you get older, there is a decrease in cardiac output, causing these symptoms."

C) Approximately 70% of older women and 50% of older men have to get up two or more times during the night to empty their bladders due to decreased bladder capacity. A decrease in blood supply causes an increase in urine concentration. A decrease in the number of nephrons decreases the filtration rate. A decrease in cardiac output decreases peripheral circulation, which would decrease urinary output day or night.

Which of the following statements best describes the body's adaptation to pain? A) The worse pain becomes, the more obvious it is. B) Once the body adapts to pain, its detrimental effects cease. C) The observation of pain's effects may become more difficult. D) The body's sympathetic response increases.

C) As the body adapts to pain, visible and physiological symptoms of pain may be harder to detect. The sympathetic response returns to baseline levels unless the client experiences breakthrough pain, and some visible signs of pain, such as crying, cease. Pain fibers may become sensitized so that the intensity and perception of pain increase over time.

A client with osteoarthritis tells the nurse she has difficulty walking to the bathroom first thing in the morning. Which nursing action would assist this client? A) Suggesting a family member provide the client with a bedpan B) Discussing the option of residing in an assisted-living facility C) Consulting with physical therapy for an assistive walking device such as a walker or cane D) Suggesting the client use a bedside commode at home

C) Assistive devices are items used to maintain, increase, or improve function. The client describes difficulty walking to the bathroom in the morning. The best intervention to help this client would be to consult with physical therapy for an assistive walking device such as a walker or cane. The use of a bedside commode or bedpan may help with the immediate need to use the bathroom, but the client will still have difficulty ambulating in the morning. The option of residing in an assisted-living facility might be premature for this client.

Which strategy to prevent hypertension is correct? A) Increase salt intake B) Reduce physical activity C) Decrease stress D) Take hot baths

C) Baths that are too hot can increase blood pressure, so they should be avoided. Avoiding cool baths will not help prevent hypertension. Reducing salt intake, increasing physical activity, and decreasing stress are all strategies to prevent hypertension.

A client has a documented stage 3 pressure injury on the right hip. Which nursing diagnosis is most appropriate for this client? A) Impaired Skin Integrity B) Risk for Injury C) Impaired Tissue Integrity D) Ineffective Peripheral Tissue Perfusion

C) Because a stage 3 pressure injury involves tissue, not just skin, this client has criteria that qualify for impaired tissue integrity. Although it is true that pressure injuries result from ineffective peripheral tissue perfusion, the diagnosis of Impaired Tissue Integrity is the more specific diagnosis. A diagnosis of Impaired Skin Integrity involves the epidermal and dermal layers only and does not extend into the tissue. This client has already suffered injury, so Risk for Injury does not apply.

A client in the ambulatory care clinic tells the nurse about experiencing frequent constipation. The nurse inquires about the client's diet. Which statement from the client would be of greatest concern for the nurse? A) "I like to eat a bran muffin and applesauce every morning for breakfast." B) "I like to eat popcorn for an afternoon snack." C) "I like to eat cheese, a banana, and a turkey sandwich for lunch." D) "I like to eat baked chicken, whole grain rolls, and a small salad for dinner."

C) Both cheese and bananas are constipating foods that should be limited. The remaining selections are not associated with constipation

The nurse is caring for a client who has extensive deep tissue damage. The nurse notes that the client is also vegan. Which dietary information should the nurse teach this client to enhance the healing process? A) "A low-fat, high-carbohydrate, low-protein diet is best for healing." B) "A high-fat, low-carbohydrate diet is best for healing." C) "A high-carbohydrate, high-protein diet is best for healing." D) "A diet high in protein and vitamin D is best for healing."

C) Carbohydrates are important to meet the energy demands of healing, and protein is needed for cell growth. This client needs to be taught to eat proteins that provide the essential amino acids that can be lacking in a vegan diet. Fats are needed in moderation for the development of cell membranes. Vitamins necessary to promote healing are C, K, A, and the B-complex vitamins.

A nurse caring for a client with cellulitis can expect to collaborate with which other member of the healthcare team? A) Clinical psychologist B) Social worker C) Wound care specialist D) Respiratory therapist

C) Cellulitis is an acute bacterial infection of the dermis and underlying connective tissue. It usually occurs as a complication of a wound infection; thus, a wound care specialist is often used to promote a positive wound care plan. Social workers, clinical psychologists, and respiratory therapists are typically not involved in the care of most clients with cellulitis.

Which of the following statements best explains why young children develop infections more often than older children and adolescents? A) Cell-mediated immunity doesn't achieve full function until a child is roughly 5 years old. B) The thymus doesn't begin to function until adolescence, so prior to this time, children don't produce enough T cells to adequately protect them from infectious agents. C) Children don't develop all of the immunoglobulins they need to protect against infection until they are about 6 or 7 years of age. D) Young children have comparatively small lymphoid tissues, which means they are less able to fight infection than are older children.

C) Children do not develop adequate levels of certain immunoglobulins (namely, IgA, IgE, and IgG) until they are 6 or 7 years old, so prior to this time, they are at increased risk of infection. Note, however, that cell-mediated immunity achieves full function early in life. This occurs in part because the thymus begins producing T cells during the fetal period, and also because young children have lymphoid tissues that are quite large in comparison to the rest of their bodies

How often should a client be monitored during the intraoperative phase? A) Every 5 minutes B) Every 30 minutes C) Constantly D) Occasionally

C) Constant monitoring of both the surgical environment and the client is necessary to ensure client safety. Monitoring the client every 5 minutes, every 30 minutes, or just occasionally could miss an important change in the client's status that could result in client harm or adverse complications.

The nurse is preparing to discharge a client with diarrhea. The healthcare provider prescribes loperamide to manage the client's diarrhea. After providing the client with information on this medication, which client statement indicates the need for further education? A) "If my diarrhea does not get better within 2 days, I will need to call my healthcare provider for further advice." B) "I will need to take the medication after each loose stool." C) "I should continue to take this medication daily until my stools are firm and dry." D) "If I start to have a fever, I need to contact my healthcare provider about continuing to take this medication."

C) Continuing to take the medication daily until the stools are firm and dry could result in constipation. If constipation occurs, the client will have another issue for resolution. The other statements are correct.

The nurse is assessing a 20-month-old and learns that he is unable to stand alone. Which aspect of development does the nurse identify as altered? A) Behavior B) Height C) Motor D) Growth

C) Development is an increase in the complexity of function and skill progression, the capacity of an individual to adapt to the environment. It includes changes in a person's cognitive, psychosocial, and gross and fine motor skills. In this case, the child's inability to stand is indicative of a delay in motor development. Growth refers to physical change and increase in size; it does not entail changes in skills and abilities. Height is one indicator of growth. Behavior is a component of each developmental stage and can sometimes indicate that development has taken place, but its absence does not mean that development has not occurred.

Clients experiencing diarrhea often lose electrolytes. Which of the following best describes the reason for this loss? A) Decreased secretion of intestinal mucus inhibits the absorption of electrolytes from the chyme by the intestine. B) Pathogenic microorganisms that cause diarrhea consume the electrolytes in the chyme, resulting in fewer electrolytes being available for absorption. C) Diarrhea causes rapid passage of chyme through the large intestine, reducing the time available for absorption of electrolytes. D) Intestinal bacteria break down electrolytes during diarrhea and make them unfit for absorption by the intestine.

C) Diarrhea causes rapid passage of chyme through the large intestine. This reduces the time available for the large intestine to absorb electrolytes and results in the electrolytes being lost with feces. Diarrhea typically increases secretion of intestinal mucus rather than decreasing it. Pathogenic microorganisms result in inflammation of the mucosa. Bacteria in the intestine cannot break down electrolytes

The charge nurse observes that a fellow charge nurse on the unit seems impaired and unable to perform client care. Which action by the charge nurse is most appropriate in this situation? A) Notifying security B) Calling the unit's nurse manager C) Discussing the situation with the nursing supervisor D) Confronting the other nurse directly

C) Discussing the situation with the nursing supervisor is the appropriate decision because the supervisor is the next highest link in the chain of command. As such, the supervisor is responsible for making the appropriate decision about how to deal with the potentially impaired charge nurse. Confronting the other nurse would not be appropriate given that the nurse seems impaired. Although the nurse manager will need to be notified, the charge nurse must first notify the nursing supervisor so he or she can determine how to proceed. Security may need to be notified eventually, but again, that decision would be made by the nursing supervisor.

The nurse is reviewing objective data obtained during the assessment of a pregnant woman in her 34th week of gestation. Which finding would be cause for concern? A) Pulse 103 bpm B) Blood pressure 108/70 C) Hematocrit 24% D) WBC count 10,340/mm3

C) During pregnancy, red blood cell (RBC) production and plasma volume increase, but because plasma volume increases more than RBC volume, the hematocrit decreases slightly. However, this client is experiencing a significant decrease in hematocrit, indicating that she is not producing adequate RBCs. The pulse normally increases by 10-15 bpm during pregnancy, blood pressure decreases slightly, and WBC count increases. Findings within the given ranges are normal during pregnancy and are not cause for concern at this point.

A client asks why asthma medication is needed even though the client's last attack was several months ago. Which response by the nurse is appropriate? A) "The medication needs to be taken or your lungs will be severely damaged and we will not be able to prevent an acute attack." B) "The medication needs to be taken indefinitely according to your doctor, so you should discuss this with him." C) "The medication is still needed to decrease inflammation in your airways and help prevent an attack." D) "The medication needs to be taken for at least a year; then, if you have not had an acute attack, you can stop it."

C) Effective treatment of asthma includes long-term treatment to prevent attacks and decrease inflammation, as well as short-term treatment when an attack occurs. Long-term treatment of asthma continues indefinitely, not for just 1 year. Telling a client that lungs will be severely damaged is nontherapeutic; the inability to prevent an acute attack in this client is not true. The nurse is able to answer the client's question; it does not need to be referred to the physician.

What is the most powerful factor in encouraging adoption of the attitudes and behaviors that characterize professionalism in nursing? A) Employment opportunities B) Licensure requirements C) Interaction with peers D) Specialized training

C) Employment opportunities and licensure requirements are not associated with adopting the attitudes and behaviors of professional nursing. Specialized training provides nurses with the knowledge and skills necessary to do their job, but it doesn't necessarily lead a nurse to internalize the attitudes and behaviors that characterize professionalism in nursing. Rather, one of the most powerful mechanisms of professional socialization is interaction with fellow students and nurses

A client is undergoing surgery for a fractured hip. The surgeon has stated that careful attention will be paid to preserving the epiphyseal plate. Which client will require this precaution during surgery? A) A postmenopausal woman with paraplegia B) A 32-year-old man who is a competitive body builder C) A prepubescent girl who is a vegetarian D) An 85-year-old woman with osteoporosis

C) Epiphyseal plates are unique joints that produce growth of bone length in children. There is an epiphyseal plate that lies between the head and neck of the femur that must be preserved during hip surgery in pediatric clients to prevent obstruction of bone growth. Of the clients listed here, only the prepubescent female is young enough to have an epiphyseal plate. All of the other clients are older than 18-25 years of age, when the epiphyseal plate closes.

The nurse in an orthopedic outpatient clinic expects to see several clients with fractures for follow-up. Based on the information provided below, which of the nurse's clients is at highest risk for a delayed union? A) A 20-year-old college student with type I diabetes mellitus who sustained a fractured tibia in a bicycle accident. The nutrition recall tool completed during the client's last visit was consistent with American Diabetic Association (ADA) guidelines. B) A 62-year-old bartender with a history of peptic ulcer disease who sustained a fractured clavicle breaking up a fight at work. During his prior visit, the client stated he was upset that his injury required him to abstain from upper body resistance training. C) A 49-year-old teacher with osteoporosis who sustained an open ulnar fracture in a motor vehicle crash. At her last visit, the client reported that she had cut down smoking to 10 cigarettes per day. D) A 55-year-old accountant who sustained fractures to the 4th and 5th right metatarsals. The client has a history of hypertension that is well controlled with medication.

C) Evaluating the risk of delayed union requires knowledge of the factors that impact bone healing. The client at greatest risk of delayed union has two factors that decrease the likelihood of proper healing: an open fracture and osteoporosis. This client also uses tobacco, which decreases blood supply to the healing bone. Although diabetes does increase the risk of delayed union, this client is young and exercised on a bicycle prior to the crash. If the client is following an ADA diet, there is adequate intake of vitamin D and calcium, which fosters bone healing. Neither peptic ulcer disease nor controlled hypertension are risks for delayed bone healing.

The nurse presses a finger into swollen skin tissue on a client's feet and ankles and notes that it creates an indentation. The nurse should correctly document a finding of which alteration in skin integrity? A) Poor turgor B) Ascites C) Peripheral edema D) Hypothermia

C) Excess fluid trapped in bodily tissue, such as the feet and ankles, creates edema. To assess for the amount of edema, the nurse presses a finger into the edematous area to create an indentation. The amount of indentation indicates the level of edema. Ascites is abdominal swelling. Skin turgor is the skin's elasticity and is assessed by gently pinching the skin over the sternum or collarbone. Skin temperature is assessed through palpation.

The nurse is evaluating care provided to a client recovering from hip replacement surgery. Which piece of documentation in the medical record indicates that the client has achieved the expected outcome for pain management? A) The client states pain is a 6 on a numeric pain scale of 0 to 10 prior to evening care. B) The client is crying and requesting pain medication prior to morning care. C) The client is using a PCA pump around the clock and rates pain as a 2 on a numeric pain scale of 0 to 10. D) The client refuses pain medication prior to physical therapy. Pain is rated as a 7 on a numeric pain scale of 0 to 10.

C) Expected outcomes for pain management following hip replacement surgery are to minimize pain to a client rating of 3 or lower via medication administration, including use of patient-controlled analgesia (PCA) as appropriate. Completely eliminating pain is an unrealistic goal. Thus, only the client who is using the PCA pump and has a pain rating of 2 on a 0-to-10 scale has achieved an expected outcome for pain management.

The nurse is caring for a 13-year-old female with a BMI of 30.4. When taking the child's vital signs, the nurse documents a blood pressure of 121/83. How would this blood pressure be categorized for this client? A) Normal blood pressure B) Prehypertension C) Hypertension D) Hypotension

C) For a 13-year-old female, the systolic blood pressure should be between 96-103 mmHg and the diastolic blood pressure should be between 58-61 mmHg. Prehypertension is defined as having a blood pressure between the 90th and 95th percentile for the child's age, height, and sex. Hypertension is defined as having blood pressure above the 95th percentile or a blood pressure higher than 120/80.

What nursing intervention is appropriate for a client with dry and cracked feet? A) Provide slippers for the client to wear at all times B) Soak the client's feet in water several times daily C) Apply lotion to the client's feet after bathing D) Massage the client's feet daily

C) For clients with dry and cracked feet, the nurse should apply lotion to the client's feet after the client's bathing time. Providing slippers and massaging the client's feet will not heal dry and cracked feet. Soaking the client's feet in water without any added moisturizers may make the client's condition worse.

What is the relationship between a full opioid agonist and the ceiling effect? A) A full opioid agonist produces few withdrawal symptoms when the drug's effects plateau and the client begins easing off the drug. B) A client may use a full opioid agonist as much or as little as necessary to control chronic pain with no ill effects. C) Side effects may limit a full opioid agonist's use but not a plateau in the beneficial effects it produces. D) At some point, a full opioid agonist's side effects cease to increase in potency, but the pain-relieving effect continues to increase.

C) Full opioid agonists do not have a ceiling effect. Therefore, full opioid agonists can be given in increasing doses until pain is relieved or side effects become intolerable.

A hospital has created a culture of safety by providing organizational support for safety initiatives and by training and encouraging healthcare employees in the area of safety. What other step is needed to promote safety for everyone in the healthcare environment? A) Keep a mindset for quality of safe practice B) Post signs related to safety on the walls C) Engage clients in their own safety D) Be a safety advocate for others

C) Healthcare facilities should use a three-pronged approach to quality and safety for everyone, including organizational support for keeping safety a priority, encouraging employees to consistently choose to follow health safety rules and standards, and actively engaging clients in every aspect of their care, including safety. Keeping a mindset for quality of safe practice and posting signs related to safety relates to the organizational support for safety. Being a safety advocate for others is related to employees maintaining safety standards.

A client with a history of insomnia is scheduled for a polysomnogram that requires an overnight stay in a sleep laboratory. The test will not include audio and video equipment. It will monitor the client's blood oxygen levels, heart rate, breathing, and eye and leg movements, and it will use an electroencephalogram to monitor brain waves. What disorder is least likely to be identified in this test? A) Periodic limb movement disorder B) Sleep apnea C) Sleep talking D) Restless leg syndrome

C) Heart rate, breathing, and blood oxygen levels as well as audio monitoring can detect snoring and breathing changes that suggest sleep apnea. The monitoring of leg movements detects periodic limb movement disorder and restless leg syndrome. Audio and video equipment can also detect parasomnias such as sleep talking, but because such equipment is not being used in this case, the study is least likely to identify this disorder.

A client who has been diagnosed with terminal pancreatic cancer states to the nurse, "I do not want any further treatment, but I am afraid my doctor will insist that I continue chemotherapy." Which role is the nurse performing when informing the healthcare provider of the client's choice to stop treatment? A) Change agent B) Case manager C) Advocate D) Teacher

C) Here, the nurse is acting as client advocate, because this role involves representing the client's needs and wishes to other health professionals. As change agents, nurses assist clients to make modifications in their behavior. Nurse case managers work with the multidisciplinary healthcare team to measure the effectiveness of the case management plan and monitor outcomes. As teachers, nurses help clients learn about their health and about healthcare procedures used to restore or maintain their health.

A hip fracture that occurs in the trochanter region would be classified as a(n) A) intracapsular fracture. B) intercapsular fracture. C) extracapsular fracture. D) subcapsular fracture.

C) Hip fractures are broadly classified as either intracapsular or extracapsular. Intracapsular hip fractures occur at the head or neck of the femur within the capsule of the hip joint. Extracapsular hip fractures occur within the trochanter region, which is between the neck and diaphysis of the femur. Extracapsular fractures can be further divided into intertrochanteric or subtrochanteric. The terms intercapsular and subcapsular are not used to describe fractures of the hip.

The warmth and redness that accompany inflammation result from which of the following steps in the inflammatory process? A) Exudate production B) Cellular regeneration C) Hyperemia D) Margination of leukocytes

C) Immediately after injury or infection, the damaged tissues release histamines, kinins, and prostaglandins. These substances serve as chemical mediators to dilate blood vessels, causing more blood to flow to the injured area. This increase in blood supply is called hyperemia and is responsible for the characteristic signs of redness and heat that accompany inflammation. Margination refers to the process by which leukocytes aggregate along the inner surface of blood vessels in an injured area. Exudate production occurs later in the inflammatory process and involves the release of fluid, dead cells, and cellular products from the injured area. Cellular regeneration occurs during the last stage of the inflammatory process and does not cause redness or warmth.

What does the nurse anticipate finding in a client with impetigo? A) An infection in the hair follicles B) Loss of skin color in blotches or sections C) An itchy rash with clusters of fluid-filled vesicles D) A fungal infection in the skinfolds

C) Impetigo is a superficial skin infection common on the face, arms, and legs of children that presents as an itchy rash with clusters of fluid-filled vesicles that rupture easily. Ruptured vesicles develop a honey-colored crust over the lesions. Folliculitis is an infection of hair follicles. Vitiligo is a loss of skin color in blotches or sections that occur when the cells that produce melanin die or stop functioning. Candidiasis is a fungal infection commonly known as thrush and found in skinfolds.

During which phase of the fracture healing process is woven bone replaced by lamellar bone? A) Reactive phase B) Reparative phase C) Remodeling phase D) Inflammatory phase

C) In the reactive or inflammatory phase of fracture healing, a hematoma forms around the injury. Inflammatory cells then enter the wound and degrade debris and bacteria in the area. Next, in the reparative phase, fibroblasts, osteoblasts, and chondroblasts begin to secrete collagen to form fibrocartilage, which develops into a soft callus that joins the fractured bone. Once the soft callus is formed, it is replaced by woven bone through endochondral ossification, which forms a hard callus. Finally, during the remodeling phase, woven bone is replaced by highly organized lamellar bone.

The nurse is caring for a client who has come to an urgent care clinic due to an arm infection. The client reports being bitten by a raccoon on a recent camping trip. Based on this data, which treatment option does the nurse anticipate for this client? A) Injection of rabies immunoglobulin only B) Administration of rabies vaccine only C) Both injection of rabies immunoglobulin and administration of rabies vaccine D) Neither injection of rabies immunoglobulin nor administration of rabies vaccine

C) In this case, exposure to rabies may have already occurred, so the client requires immediate protection (passive immunity) from the virus. Such protection is provided by the injection of rabies immunoglobulin produced in another living host. However, passive immunity does not confer lasting immunity, so the client will also require rabies vaccine to start the process of antibody development (active immunity).

Which intervention would best improve diet adherence of an older male immigrant recently diagnosed with gastroesophageal reflux disorder (GERD)? A) Scheduling low-fat meal deliveries to the home B) Providing printed diet information in his native language C) Interviewing the client to assess his current diet D) Giving a list of foods to avoid to the client's wife

C) Interviewing the client to assess his current diet will provide information on which to base a collaborative nutrition plan. Scheduling low-fat meal delivery to the home, giving printed diet information in his native language, and giving a list of foods to avoid to the client's wife are not appropriate until the nurse has information on which to base an individualized culturally sensitive teaching plan

The nurse is caring for an older school-age client who is sleeping when the menu choices for dinner are brought to the room. Which intervention should the nurse use to meet the dietary needs of this client? A) Wake the child to choose a meal for dinner. B) Order chicken nuggets because most children like this meal. C) Ask the dietary worker to come back later. D) Ask the parents to bring dinner from home for the client.

C) Involving children in their own care increases cooperation and decreases anxiety. The nurse would ask the dietary worker to come back later to increase the child's involvement in his own care and to avoid disturbing the client or choosing a meal the client won't eat. If the parents are present, the nurse might ask them if they are comfortable making choices for the child, but asking them to bring food in is inappropriate.

Which of the following procedures would be most appropriate to repair a finger joint that is affected by severe osteoarthritis (OA)? A) Osteotomy B) Joint resurfacing C) Joint fusion D) Internal fixation

C) Joint fusion is used to permanently fuse two or more bones together at a joint using pins, plates, screws, and rods. It is often recommended for badly damaged smaller joints, such as the spine, wrist, ankle, finger, or toe. Osteotomy is usually performed on the knee and hip and entails surgical removal of a wedge of bone above or below the joint to realign the joint and shift the weight away from the damaged portion of the joint. Joint resurfacing, which involves removing a small amount of bone at the articulating surface of the joint and fitting a metal replacement over the end of the bone, is often performed for hip and shoulder joints. Internal fixation is used to fix fractures, not to address osteoarthritis.

A nurse is teaching the parents of a client who was recently diagnosed with osteoarthritis (OA) about their child's condition. Which statement by the parents indicates the need for further instruction? A) "Our daughter's OA is likely related to a joint injury she sustained last year." B) "Most kids with OA usually have only one or two affected joints." C) "Because our daughter developed OA as a child, she is more likely to become disabled as a result of this condition." D) "Our daughter may outgrow her OA as she ages."

C) Juvenile OA is usually secondary to a congenital abnormality, genetic condition, or joint injury. It typically occurs only in the one or two joints affected by the abnormality or injury. Children with OA are less likely to become disabled and may outgrow the condition as they age. Thus, the parents' statement about an increased likelihood of disability indicates the need for further instruction.

Development of leukopenia suggests that an individual A) is immunocompetent. B) is experiencing an infection somewhere in the body. C) may have suppressed bone marrow activity. D) has an abnormally high number of circulating leukocytes

C) Leukopenia involves a decrease in the number of circulating leukocytes in the body. Because leukocytes are produced in the bone marrow, the presence of leukopenia suggests that an individual may have suppressed bone marrow activity. Low leukocyte levels disrupt the body's ability to adequately respond to infection and may result in immunodeficiency, not immunocompetence. In individuals with proper immune system function, the presence of infection should cause an elevation in circulating leukocytes (leukocytosis) rather than leukopenia.

The nurse is caring for an older adult client. The client tells the nurse that he is constipated. What is the nurse's initial action? A) Encourage the client to increase fluid intake and activity. B) Assess the client's intake of fiber and fluids. C) Determine what the client means by constipation. D) Obtain an order for a laxative and an enema from the physician.

C) Many older adults believe that a daily bowel movement is important for health, which leads to an increased incidence of perceived constipation in older adults. The nurse should first carefully evaluate the client's concern and question the person as to what is meant by constipation. Determining the client's normal frequency of bowel movement, consistency of stool, and effort in passing stool is important before deciding to act. The other suggestions—achieving adequate fluid intake, exercising, including fiber in the diet, and using a laxative (and possibly an enema)—may be appropriate once the nurse has adequately assessed the client's concern of constipation.

The nurse for a urology clinic is planning an in-service about urinary infections for a group of novice nurses. Which statement should the nurse include in the presentation? A) Men are most likely to experience descending urinary tract infections. B) Straight catheterization is the only way to evaluate for the presence of a urinary tract infection. C) Women require a shorter course of antibiotic therapy to manage a urinary tract infection than men. D) The rate of urinary tract infections is similar between men and women

C) Men will usually be prescribed a longer course of therapy to manage a urinary tract infection. They have a longer urethra and typically experience more complicated urinary tract infections than women. Descending infections are less common than ascending infections of the urinary tract. A clean-catch midstream specimen is a reliable means to obtain a urine specimen to check for the presence of infection. Women have a higher rate of urinary tract infection than men.

What is the definition of the basal metabolic rate? A) The amount of energy stored in fat each day B) The speed of triglyceride breakdown C) The cost in kilocalories of being alive D) The speed at which glucose is converted to energy

C) More than 70% of the energy expended each day goes to maintaining the basal metabolic rate (BMR)-essentially, the "cost" (in kilocalories) of being alive. It is not a measure of triglyceride breakdown, storage of energy in fat, or the conversion of glucose to energy.

A 15-year-old female client diagnosed with juvenile primary fibromyalgia syndrome asks the nurse whether the fibromyalgia can be cured so that she won't have to deal with it as an adult. Which of the following is the best response the nurse could make to this question? A) The fibromyalgia likely can be cured with proper sleep hygiene, exercise, and pharmacologic management. B) Fibromyalgia can be difficult to treat effectively, but the symptoms are unlikely to persist into adulthood. C) It is likely that the fibromyalgia will persist into adulthood, and there is no cure, but the symptoms can be treated and the condition is not life-threatening. D) The client will suffer from persistent fibromyalgia for the remainder of her life unless a cure can be found.

C) More than 80% of children who have JPFS will have fibromyalgia that persists into adulthood. It is not certain that this client's fibromyalgia will persist into adulthood, but it is likely. There is no cure for fibromyalgia, but it can be treated symptomatically and it is not progressive or life-threatening.

The nurse is caring for a 6-month-old infant with pyloric stenosis. Which of the following statements regarding this client's digestive system is false? A) The client has voluntary control over swallowing. B) Enzymes from the client's pancreas are sufficient to aid in digestion. C) The client has a complete set of primary teeth. D) The client's tongue is larger than an adult's in comparison to the nasal and oral passages.

C) Most children have a complete set of primary teeth by 3-6 years of age. At 6 months, the first teeth begin to erupt. An infant develops voluntary control over swallowing by 6 weeks. Enzymes are sufficient to aid in digestion by 4-6 months, so an infant at 6 months should have sufficient enzymes for digestion. The infant's tongue is larger than that of a child or adult in comparison to the nasal and oral passages.

A nurse overhears two colleagues disparaging a client on a public elevator. Which action by the nurse is most appropriate? A) Immediately ask the nurses to stop talking about clients in public. B) Report the nurses' behavior to the unit manager. C) Wait and speak to the nurses about their behavior in a private place. D) Report the nurses' behavior to the hospital's risk manager.

C) Nursing integrity ensures that patients' rights are respected in the healthcare setting. If the first nurse were to confront the two colleagues in the elevator, this conversation could be overheard by others. Thus, the first nurse should wait and speak to the other nurses privately about the breach of confidentiality. The nurse could report the incident to the unit manager, but it would be better for the nurse to personally confront the two colleagues in a professional way. If this isn't possible, then notifying the nurse manager would be the next step. The risk manager is only involved in situations in which there is an injury to a staff member or client.

The nurse is caring for a client who failed to meet the outcome of healing of a stage 2 pressure injury over the coccyx. Which should the nurse identify as a likely contributing factor? A) The rubber doughnut pressure relief device was not delivered by central supply. B) The client's serum albumin increased over the last month. C) A right side-back-left side-back turning schedule was used. D) Nurses did not document disinfection of the wound with alcohol at each dressing change.

C) Of the options listed, the only one that would result in poor healing is the right side-back-left side-back turning schedule. This schedule places the client on the back 50% of the time, which is where the ulcer is located. There are six possible body positions when preventing or treating a pressure ulcer, and these positions should be used equally. The nurse should be careful to minimize pressure on an already-formed pressure ulcer. A rubber doughnut-style device should not be used, so the fact that it was not delivered did not contribute to failure to meet the outcome. An increase in serum albumin is a good finding and would increase, not decrease, wound healing. Alcohol interrupts healing, so it is good that nurses did not use alcohol to disinfect the wound.

Which oral antiviral drug that is commonly prescribed for influenza A and B works by preventing the release of newly formed virus? A) Rimantadine (Flumadine) B) Zanamivir (Relenza) C) Oseltamivir (Tamiflu) D) Amantadine (Symmetrel)

C) Oseltamivir (Tamiflu) is an oral antiviral drug that is prescribed for influenza A and B. It works by preventing the release of newly formed virus. Zanamivir (Relenza) has a similar mechanism of action, but it is given via inhalation. Rimantadine (Flumadine) and amantadine (Symmetrel) are primarily used for prophylaxis of influenza. They are not recommended by the Centers for Disease Control and Prevention (CDC) for treatment of active influenza.

The nurse is providing teaching to the client recently diagnosed with osteoarthritis. Which statement by the nurse is correct? A) "Osteoarthritis is most commonly seen in thin, small-built female clients." B) "Osteoarthritis is a result of joint inflammation." C) "Osteoarthritis occurs due to erosion of cartilage in the joints." D) "Osteoarthritis is a metabolic bone disease."

C) Osteoarthritis is characterized by progressive erosion of the cartilage within joints. It is not a metabolic bone disease; examples of such diseases include osteoporosis, osteomalacia, and Paget disease. Thin, small-built female clients are at increased risk for osteoporosis, not osteoarthritis. In fact, osteoarthritis is more commonly associated with obesity than with slight build. Finally, joint inflammation is a characteristic of rheumatoid arthritis, not osteoarthritis.

The cells that produce the matrix for bone formation are known as A) osteoclasts. B) sarcomeres. C) osteoblasts. D) epiphyseal plates.

C) Osteoblasts are the cells that produce the matrix for bone formation, whereas osteoclasts are cells that break down bone tissue. Sarcomeres are filaments made of actin or myosin that are found within muscle. Epiphyseal plates are areas of cartilage located between the epiphysis and diaphysis of a child's long bones.

A client who has been undergoing treatment for chronic back pain has been considering various over-the-counter nonopioids to manage the pain. The nurse has assessed the client's needs and discussed the use of available methods with the client. Which client statement indicates the need for further instruction? A) "Nonopioid pain medications can have serious side effects I need to consider and watch for carefully." B) "I should not take a higher than recommended dose because the beneficial effect isn't likely to be higher with a higher dose." C) "I may use these medications for as long as I think they are necessary." D) "I may use both opioid and nonopioid medications together, especially to relieve severe pain."

C) Over-the-counter (OTC) nonopioids are associated with severe side effects, especially when taken long term. NSAIDs can produce gastrointestinal (GI) toxicity and prolong bleeding times, and acetaminophen can produce liver and kidney toxicity. Nonopioids have a ceiling effect, so taking a higher dose will not produce a greater analgesic effect. While nonopioids are rarely effective alone for severe pain, they may produce a synergistic effect to relieve pain when combined with an opioid.

The client is experiencing urinary urgency and frequency. Which medication should the nurse anticipate may be prescribed by the healthcare provider? A) Furosemide B) Bumetanide C) Oxybutynin D) Bethanechol chloride

C) Oxybutynin is an anticholinergic that reduces urgency and frequency by blocking muscarinic receptors in the detrusor muscle of the bladder, thereby inhibiting contractions and increasing storage capacity. The nurse would anticipate an order for oxybutynin. Furosemide is a diuretic and works in a specific place within the nephron to increase fluid excretion and prevent fluid reabsorption. Bumetanide is a diuretic and works in a specific place within the nephron to increase fluid excretion and prevent fluid reabsorption. Bethanechol chloride is a cholinergic agent that stimulates bladder contraction and facilitates voiding.

A client with terminal cancer is undergoing surgery to partially remove a tumor that is pressing on a nerve and causing pain. This is classified as what type of surgery? A) Reconstructive B) Diagnostic C) Palliative D) Emergency

C) Palliative surgery may be performed to alleviate pain or symptoms associated with a disease, and so this client's surgery, because it involves partially removing a tumor causing severe pain, is most illustrative of palliative surgery. Reconstructive surgery is to restore lost or reduced appearance or function. A diagnostic procedure would be conducted to determine or confirm a diagnosis. Emergency surgery is to save life or limb

A novice nurse is planning care for an older adult client with a wound infection and systemic blood infection. The nurse completes the plan of care and decides to complete which action to enhance the skill of critical thinking? A) Discuss the plan with the physician. B) Request that the client review the plan. C) Request a review of the plan with the nurse's preceptor. D) Place the plan on the client's chart.

C) Part of the critical thinking process is inquiry, or searching for knowledge or facts. One way the novice nurse can implement inquiry is to discuss the care plan with a more experienced nurse to determine if additional or better interventions need to be added to the care plan. The nurse would seek the opinion of another nurse, not the physician or the client, as the goal is to enhance the nurse's ability to use the nursing process. The nurse could put the plan on the chart, but the new nurse would learn more about critical thinking by requesting assistance from a more experienced nurse.

Why does breastfeeding confer some degree of passive immunity to an infant? A) The infant receives maternal antibodies via breastmilk, and these antibodies stimulate the infant's immune system to begin producing antibodies of its own. B) Consumption of breastmilk introduces certain antigens into the infant's body, thereby stimulating the infant's immune system to begin producing antibodies to these antigens. C) The infant receives maternal antibodies via breastmilk, and these antibodies provide the infant with immediate protection against specific antigens. D) Consumption of breastmilk introduces certain antigens into the infant's body, thereby stimulating the infant's immune system to begin producing antigens of its own.

C) Passive immunity occurs when individuals receive antibodies from another person rather than by producing them through their own immune system. This type of immunity is immediate, and it can be acquired through the passing of antibodies between mother and newborn via breastmilk.

The nurse is providing care for a client who is experiencing subjective symptoms of carpal tunnel syndrome. Which test should the nurse anticipate being performed by a provider during the physical assessment of this client? A) Bulge test B) Ballottement test C) Phalen test D) McMurray test

C) Phalen test is a special assessment to determine whether the client is experiencing carpal tunnel syndrome. With this test, the wrists are held in acute flexion for 60 seconds. Numbness, tingling, or pain may indicate carpal tunnel syndrome. All of the other tests listed here are used to assess the knee

Which of the following is a pharmacologic therapy for acute pain? A) Antidepressants B) Muscle relaxants C) Opioid analgesics D) Stimulants

C) Pharmacologic pain management for acute pain involves opioid analgesics, nonopioid analgesics, or nonsteroidal anti-inflammatory drugs (NSAIDs). It does not involve antidepressants, stimulants, or muscle relaxants.

What most determines the effectiveness of pharmacologic therapy for fatigue? A) The patient's attitude toward taking medication B) The length of time the patient has been fatigued C) The appropriateness to the patient's condition D) The patient's commitment to complementary therapies

C) Pharmacologic therapy depends on the cause of fatigue. To be effective, the pharmacologic therapy must be tailored to the client's specific condition. The length of the patient's condition isn't a factor, and neither is the patient's commitment to complementary therapies. The patient certainly must be willing to take medication, but if the medication isn't appropriate in the first place, the patient's willingness doesn't matter.

A pediatric client is diagnosed with gastroesophageal reflux disorder (GERD). The nurse is observing a return demonstration of the caregiver preparing and feeding the infant formula. Which observation demonstrates correct procedure for preventing GERD symptoms? A) Burping the infant after 4 ounces of formula are taken B) Thinning the formula with water prior to feeding C) Positioning the infant upright for a minimum of 30 minutes D) Warming the formula prior to feeding

C) Positioning the infant upright for 30 minutes after a feeding can reduce GERD symptoms. Infants with GERD should be burped after every 1-2 ounces of formula are taken. Prethickened formulas can also reduce GERD symptoms. Warming the formula does not impact GERD symptoms.

The nurse is teaching a client scheduled for Roux-en-Y gastric bypass surgery about potential postsurgical complications and how to reduce them. Which client statement best indicates that teaching has been effective? A) "I need to eat at least one meal a day that is high in simple carbohydrates." B) "Complications of this surgery are likely to be limited to mild gastrointestinal issues for several days." C) "I need to be alert for the indications of infection or malnutrition." D) "I will not continue my exercise program following this surgery."

C) Possible postoperative complications for a procedure such as a gastric bypass include anastomosis leak with peritonitis, abdominal wall hernia, gallstones, wound infections, deep venous thrombosis, nutritional deficiencies, and gastrointestinal (GI) symptoms. If the client recognizes a need to be alert to the signs of infection and malnutrition, this shows awareness of some of the complications that might be expected. Mild GI issues are not the only significant complication of this surgery. The client likely should continue the exercise program. Eating meals high in simple carbohydrates can bring on dumping syndrome, a complication in which stomach contents move rapidly through the small intestine, drawing fluid into the intestine by osmosis

The nurse is teaching the family of a school-age client diagnosed with inflammatory bowel disease regarding the administration of prednisone at home. At which time should the nurse instruct the parents to provide this medication to the client? A) 1 hour before meals B) At bedtime C) With meals D) Between meals

C) Prednisone, a corticosteroid, can cause gastric irritation. It should be administered with meals to reduce this irritation. It should not be given on an empty stomach.

Which of the following situations would be considered an example of insubordination? A) A staff nurse informs the unit supervisor that she can't complete an assigned care task because it is outside her scope of practice. B) A staff nurse informs the unit supervisor that he cannot complete an assigned care task because it requires a piece of equipment that is not currently available. C) A staff nurse informs the unit supervisor that she won't complete an assigned care task because it's beneath her level and more appropriate for unlicensed assistive personnel. D) A staff nurse informs the unit supervisor that he cannot complete an assigned care task because another nurse has already performed the task.

C) Professional nurses should refuse work assignments only when they are not qualified or not prepared to perform the assignment—as would be the case when a task is outside the nurse's scope of practice or a needed piece of equipment is unavailable. Refusal to perform a task would also be appropriate if completing the task would result in duplication of services. In all such cases, the nurse should discuss the situation immediately with the supervisor. Otherwise, the nurse should be ready to complete all assigned tasks, because refusal to do so may be regarded as insubordination and grounds for dismissal. Here, the nurse who refuses a task simply because she feels it is "beneath her level" would be engaging in insubordinate behavior.

The most common type of upper urinary tract infection that results from bacteria ascending to the kidney from the lower respiratory tract is A) urethritis. B) prostatitis. C) pyelonephritis. D) cystitis.

C) Pyelonephritis is inflammation of the renal pelvis and parenchyma. This is the most common upper urinary tract infection. Urethritis (inflammation of the urethra), cystitis, inflammation of the urinary bladder), and prostatitis (inflammation of the prostate gland) are all lower urinary tract infections.

The nurse is providing care for several clients on a medical-surgical unit. The nurse anticipates that the client with which condition may require surgery? A) Hepatitis B) Pancreatitis C) Pyloric stenosis D) Malabsorption disorder

C) Pyloric stenosis ultimately requires surgery to split the pyloric muscle to allow passage of food and fluid. Hepatitis, pancreatitis, and malabsorption disorders are medically treated. D) Pyloric stenosis ultimately requires surgery to split the pyloric muscle to allow passage of food and fluid. Hepatitis, pancreatitis, and malabsorption disorders are medically treated.

A nurse educator is conducting a continuing education in-service for the nurses in a pediatric intensive care unit. Why is it so important for the professional nurse to attend these in-services? A) Most states require it to maintain licensure. B) It is a good way to receive overtime pay. C) Research and new technology demand that nurses stay current. D) New diseases are discovered every day.

C) Research and technology are constantly changing and improving client care. Professional nurses are accountable for staying abreast of new information by attending continuing education courses. Receiving overtime pay is not a reason to attend continuing education courses. Although new diseases are discovered regularly, nurses wouldn't necessarily need ongoing training on the diseases themselves, but rather on new ways of treating diseases. Finally, although many states do have continuing education requirements for nurses, new technology and research are the most important reasons for staying current. In fact, these changes are the reasons why continuing education is required by most states.

A hospital is preparing for the American Nurses' Credentialing Center's magnet hospital designation process. Nurse representatives on the Magnet Council consider several Professional Practice Models (PPMs) as their approach to nursing care. After selecting a PPM model, members of the Council plan a series of nursing grand rounds. These focus on the six Cs of caring in nursing: compassion, competence, confidence, conscience, commitment, and comportment. What particular nursing theory, philosophical approach, or framework of caring have the nurses decided to adopt? A) Watson's Theory of Human Care B) Leininger's Theory of Culture Care Diversity and Universality C) Roach's Theory of Caring as the Human Mode of Being D) Boykin and Schoenhofer's Nursing as Caring Theory

C) Roach's Theory of Caring as the Human Mode of Being includes the six Cs of caring in nursing. The latter requires nurses to be knowledgeable about codes of ethics and moral behavior. The other three approaches to caring in nursing focus on other major tenets related to caring. Watson's Theory of Human Care emphasizes the role of authentic caring relationships in healing. Leininger's Theory of Culture Care Diversity emphasizes actions that preserve, maintain, and accommodate the cultures of diverse clients, and Boykin and Schoenhofer's Nursing as Caring Theory describes caring as an essential aspect of nursing and a process rather than a mere goal.

The nurse observes flakes of greasy white dandruff in a client's hair. The nurse should correctly identify this as which type of secondary lesion? A) Nodule B) Macule C) Scales D) Crusts

C) Scales are flakes of greasy, keratinized skin tissue that vary in color from white, to gray, to silver. An example of this type of skin lesion is dandruff. Macules and nodules are primary skin lesions. A crust is an area of dry blood, serum, or pus left on the skin surface when vesicles or pustules break.

The nurse is assessing a child who was last seen in the clinic 2 years earlier. Which of the following should the nurse anticipate as having remained relatively stable over time? A) The child's behavior B) The child's physical characteristics C) The child's temperament D) The child's home environment

C) Temperament refers to innate characteristics that do not change over time. Each individual brings these characteristics to the events of daily life, and they set the stage for the interactive dynamics of growth and development. Physical characteristics include eye color and height; although some physical characteristics remain constant as a child grows, others do not. A person's temperament shapes his or her behaviors; however, regardless of temperament, a child's behaviors should be expected to change over time as a normal part of development. Finally, a child's home environment may change at any time due to any number of factors.

Which of the following statements on the part of the nurse is an example of the communication barrier known as testing? A) "Most people have little to no pain after this type of procedure." B) "Tell me when and why you started smoking marijuana." C) "Do you think you're the only client on the unit right now?" D) "How are you still in pain after receiving both doses of medication?"

C) Testing involves asking questions that make the patient admit to something. These responses permit the patient only limited answers and often meet the nurse's need rather than the patient's. Of the options listed here, "Do you think you're the only client on the unit right now?" best meets this definition. Telling the client that most people have little or no pain after a procedure is an example of stereotyping. Asking about why a client started using marijuana may be an example of probing. Questioning how the client is still in pain after medication administration is an example of challenging, or making clients prove their statements or point of view.

The nurse is caring for a client admitted to the hospital with lower extremity edema and shortness of breath. Which electrocardiogram finding indicates the client is at risk for an alteration in perfusion? A) P wave smooth and round B) Absent U wave C) PR interval 0.30 seconds D) ST segment isoelectric

C) The PR interval is normally 0.12-0.20 seconds. Intervals greater than 0.20 seconds indicate a delay in conduction from the SA node to the ventricles. A P wave should be smooth and round. The U wave is not normally seen. The ST segment should be isoelectric

The structure of the respiratory system that serves as the site of gas exchange is the A) macrophage. B) bronchi. C) alveoli. D) bronchiole.

C) The alveoli comprise the terminal structures of the lower respiratory system. Alveoli serve as the sites of gas exchange, specifically, carbon dioxide and oxygen. Bronchi and bronchioles are larger structures in the respiratory system that serve as tracts for airflow. Macrophages are immune cells that keep the alveoli region free of microbes.

A client diagnosed with asthma has a respiratory rate of 28 at rest with audible wheezes upon inspiration. Based on this data, which nursing diagnosis is the most appropriate? A) Ineffective Airway Clearance B) Impaired Tissue Perfusion C) Ineffective Breathing Pattern D) Activity Intolerance

C) The client is experiencing an increased respiratory rate and is wheezing, which is an ineffective breathing pattern. Not enough information is provided to determine whether the client has ineffective airway clearance, activity intolerance, or impaired tissue perfusion

The nurse is caring for a client who is having difficulty understanding the dressing changes that need to be completed in the home as part of postdischarge wound care. The client asks the nurse to demonstrate the procedure again and allow the client's spouse to perform the procedure while the nurse watches. What is the most likely outcome of this assertive request by the client? A) A slightly increased chance that the wound will become infected due to exposure during dressing changes B) Less compassionate care for the client due to the nurse's irritation by the request C) A greater likelihood that the wound will heal appropriately D) A guarantee that the spouse will change the dressings correctly

C) The client used assertive communication to ensure that the dressing changes would be performed correctly, which will likely result in appropriate healing of the client's wound. No information is provided about the nurse's response to the request, and even if the nurse is irritated, these feelings should not affect the quality of care. There is no guarantee that the client's spouse will always perform the task correctly, because humans make mistakes. Infection of a wound that is dressed correctly is not the likely result of this request.

The nurse is assigned two clients. One client needs postoperative teaching in preparation for discharge, and the other client with pneumonia has a PaCO2 of 85. Why does the nurse decide to see the client with pneumonia first? A) The nurse can delegate postoperative teaching to unlicensed assistive personnel (UAP). B) The client with pneumonia needs more care than the client needing postoperative teaching. C) The client with pneumonia may be experiencing respiratory distress. D) The room of the client with pneumonia is closer than that of the client needing postoperative teaching.

C) The client with a PaCO2 of 85 could be in serious trouble. The nurse would decide to assess that client first in order to prevent dire consequences for the client. The client with pneumonia probably needs more care than the client preparing for discharge, but the reason for the decision is based on a potentially critical need by the client with pneumonia. The nurse cannot delegate discharge teaching to a UAP; even if delegation were permitted, the nurse would see the client with a high PaCO2 as being the greater priority. Placement of the client's room can be a decision that is made when considering time management issues; however, the physiologic needs of the clients are the first consideration of the nurse.

The nurse is planning care for a client experiencing dyspnea and a subsequent activity intolerance. Which action by the nurse is the most appropriate? A) Encourage strenuous activity. B) Consult a dietitian for low-calorie meals. C) Space periods of activity with periods of rest. D) Encourage dependence with activities of daily living.

C) The client with shortness of breath will experience activity intolerance due to a lack of oxygen and fatigue. It will often be appropriate to space periods of activity with periods of rest. Clients with respiratory disorders often need an increase, not a decrease, in calories to maintain body functions. The client will be weak, so the nurse should not encourage strenuous activity. The nurse would want the client to be as independent as possible and would not encourage dependence with activities of daily living.

The nurse is assessing a 12-year-old male client. The client is within the normal range for height, weight, and body mass index (BMI) for his age. The client plans to play contact sports at school this year. He lives with his mother and attends after-school events when she is working late. What education should the nurse identify as a priority for this client to promote safety? A) The importance of learning how to feel secure when he is at home alone B) The importance of maintaining a normal weight and participating in physical activity C) The importance of using safety equipment when playing contact sports D) The importance of good hygiene practices and healthy diet

C) The client's biggest safety risk is a risk of injury from contact sports. The nurse should encourage the client to use proper safety equipment to avoid injury. Promoting a sense of security is important for latchkey children, but this client does not appear to be home alone for extended periods based on participation in school, sports, and after-school activities. The client already has a normal weight and participates in physical activity, so education related to these topics is not as important as sports safety. There is no evidence that this client has poor hygiene or an unhealthy diet.

During an assessment, an adolescent reports: "I get up at 6 a.m., I attend early-morning band classes three times each week, I play sports for 2 hours each day after school, and homework takes me 3 hours each night. I always feel tired." Based on these data, which question will the nurse ask while continuing the client's history? A) "Do you think you are involved in too many activities?" B) "Do you consume foods high in iron such as red meat and green, leafy vegetables?" C) "How many hours of sleep do you get each night?" D) "Have you considered talking with your teachers about decreasing your homework?"

C) The data in this scenario reveal very little time for sleep; therefore, the history should focus on sleep patterns, not diet. Asking if the client thinks the number of activities or homework should be reduced does not directly address the number of hours of sleep the client is getting.

The nurse is conducting a physical assessment of a middle-aged client upon admission to the cardiovascular care unit for cardiopulmonary problems. Which position is likely to be contraindicated for this client? A) Sitting with back unsupported B) Supine (horizontal recumbent) C) Dorsal recumbent (back-lying) D) Sims (side-lying)

C) The dorsal recumbent position (back-lying with knees flexed and hips externally rotated) is usually contraindicated for clients with cardiopulmonary problems because of the increased physiologic stress that this position places on the body. The other positions listed here—sitting (with back supported or unsupported), supine (horizontal recumbent), and side-lying (Sims)—are typically better tolerated by clients with cardiopulmonary illness.

Blood pressure is influenced by all except which factor? A) Pumping action of the heart B) Peripheral vascular resistance C) Heart rate D) Blood volume

C) The factors that determine blood pressure include the pumping action of the heart, peripheral vascular resistance, and blood volume and viscosity. Heart rate by itself does not determine blood pressure.

The first day after surgery to repair a fractured hip sustained from a fall, an older adult client refuses to ambulate but states, "I will consider it tomorrow." In this situation, which is the priority action by the nurse? A) Coordinate personnel to assist with ambulation B) Document the client's refusal C) Assess why the client is refusing to ambulate D) Notify the healthcare provider

C) The first thing the nurse should do is assess why the client is refusing to ambulate. The client might be fearful of falling, given that a prior fall resulted in a fractured hip. Following this assessment, the nurse could plan interventions that would facilitate ambulation, such as controlling pain and reducing the fear of falling. It is premature to notify the healthcare provider. The nurse should not force the client to get out of bed. Documenting the client's refusal is appropriate, but after determining the reason for the refusal

The nurse is caring for a client with a history of chronic urinary tract infections. The nurse is planning care for this client based on the priority nursing diagnosis of urinary retention related to scarring. Based on this data, which treatment does the nurse anticipate from the healthcare provider? A) Antibiotic therapy B) An anticholinergic medication C) Intermittent straight catheterization D) Removal of bladder stones

C) The healthcare provider may order straight catheterization so the client can be taught to self-catheterize and manage the urinary retention at home. Antibiotic therapy is not indicated, as the client does not have an infection now. Anticholinergic medications can cause urinary retention. Bladder stones are not the problem; scarring is

The nurse is assessing an older adult client who experienced a myocardial infarction (MI) a few months ago. The client states, "I don't feel like doing much. I feel okay physically, but I just don't want to be around anyone." Based on this data, which tool should the nurse use to further assess the client? A) Get-Up-and-Go Test B) Barthel Index of Activities of Daily Living C) Geriatric Depression Scale D) Short Portable Mental Status tool

C) The most appropriate tool for the nurse to use in further assessing the client is the Geriatric Depression Scale. This instrument is designed to assess low mood and diminished self-esteem, as well as loss of interest in activities that the client normally enjoys. The nurse should recognize the need to assess for depression in light of the client's statement, as well as the client's recent MI. (Depression is not uncommon following a major illness like MI.) The other instruments listed here measure other factors that are relevant to the functioning of older adult clients, but they do not appear to be indicated at this time given the historical data presented.

An older adult African American client with a history of celiac disease presents with abdominal cramps, pain, and diarrhea. The client denies the use of alcohol, but states, "My favorite foods are steak, cheese, and ice cream." Based on this data, which condition does the nurse suspect? A) Acute pancreatitis B) Appendicitis C) Lactase deficiency D) Food poisoning

C) The most common risk factor for pancreatitis is alcohol abuse. Appendicitis usually involves loss of appetite and nausea and/or vomiting soon after abdominal pain begins. Lactose intolerance is more common in Native Americans, Asians, Hispanics, and African Americans and in those with a history of celiac disease. Food poisoning generally causes some nausea and vomiting

Which action by the novice nurse demonstrates commitment to a new job on a busy cardiac care unit? A) Joining the American Nurses Association (ANA) B) Questioning the preceptor during all procedures C) Arriving at every shift on time D) Exhibiting clinical competence

C) The new nurse can demonstrate commitment by showing up for all shifts in a timely fashion. It should not be necessary for the new nurse to question every procedure, given that the new nurse has some prior clinical experience. Joining the ANA is a commitment, but it is not relevant to this question. Clinical competence develops over time, and the new nurse is not likely to exhibit complete competence yet; thus, showing up on time is a better predictor of the nurse's commitment.

A novice nurse is caring for an older adult client with dementia. The nurse leaves for a break and forgets to put the call light within the client's reach. When checking on the nurse's clients, a colleague discovers the nurse's negligence. Which conclusion by the colleague is appropriate? A) The novice nurse is appropriately taking care of self. B) The novice nurse's workload is too difficult. C) The novice nurse is demonstrating inappropriate safety measures for the client. D) The novice nurse is demonstrating appropriate comfort measures for the client.

C) The novice nurse is demonstrating inappropriate safety measures by not leaving the call light within the client's reach. There is no evidence that the nurse's workload is too difficult, and the scenario does not provide any information about comfort measures. It is appropriate for nurses to take breaks; however, client safety should be the nurse's first commitment.

When auscultating the lungs of a client experiencing dyspnea, the nurse hears a low-pitched sound that is continuous throughout inspiration. What does this lung sound indicate to the nurse? A) Narrow bronchi B) Narrow trachea passages C) Blocked large airway passages D) Inflamed pleural surfaces

C) The nurse auscultated rhonchi, which are low-pitched sounds that are continuous throughout inspiration. Rhonchi suggests blockage of large airway passages, which may be cleared with coughing. Stridor is the sound created by narrow tracheal passages. A low-pitched grating sound is created by inflamed pleural surfaces. Wheezing is created by narrow bronchi

Which nursing intervention related to perfusion can be performed independently? A) Administration of drug regimens B) Insertion of device to measure central venous pressure (CVP) C) Teaching relaxation techniques D) Thoracentesis

C) The nurse can teach relaxation techniques as an independent intervention to provide psychosocial support to the client. The nurse must administer drug regimens only under the order of a physician or nurse practitioner. Although nurses can monitor central venous pressure, they are not responsible for inserting the device to measure CVP. A physician or nurse practitioner usually performs a thoracentesis.

An experienced nurse has accepted a new position in the mental health unit after working in the medical-surgical floor for the past 4 years. What training would be beneficial for the nurse to refresh before starting her new position? A) How to properly use respirators B) How to prevent needlestick injuries C) How to manage aggressive behaviors D) How to safely lift and move clients

C) The nurse is exposed to a higher risk of injuries from assaults by clients or their families in a mental health unit compared to a medical-surgical unit. Therefore, the nurse may benefit from receiving training related to managing aggressive behaviors. Because of her experience on the medical-surgical unit, the nurse is already likely familiar with how to safely lift and move clients and how to prevent needlestick injuries. Knowing how to use a respirator is more important for an infectious disease unit, not a mental health unit.

An older adult client with poor nutritional intake is demonstrating signs of poor wound healing. Which intervention best addresses the client's nutritional needs? A) Assist with deep-breathing exercises. B) Medicate for pain prior to dressing changes. C) Request a dietary consult. D) Encourage ambulation.

C) The nurse should consult with a dietitian to identify ways to improve the client's intake to support wound healing. Deep-breathing exercises and ambulation may or may not help the client at this time. Medicating for pain prior to dressing changes is not going to help with wound healing.

The nurse is caring for a client with a self-reported latex allergy. Which strategy can the nurse use to ensure the safety of this client? A) Wear hypoallergenic gloves B) Wear gloves with powder C) Wash hands after taking gloves off D) Keep beta adrenergic agonists on hand

C) The nurse should wear latex-free gloves that are hypoallergenic and powderless. Not all hypoallergenic gloves are latex-free. Powder from the gloves can absorb the latex and be transferred to clients through touch or through the air. Therefore, it is important to wash hands after removing gloves, especially gloves with powder. Beta adrenergic agonists are used for the treatment of asthma, which may develop with chronic latex exposure in a sensitive individual, but it will not affect the early symptoms of latex allergy

The nurse is providing care for a client who is about to be discharged. The nurse is discussing the discharge orders with the client's primary healthcare provider. Which statement by the nurse is an appropriate example of using assertive communication? A) "Can we talk about this client prior to discharge?" B) "That new medication you prescribed for the client is ineffective." C) "I am worried about the client's blood pressure. It remains high even with the new medication." D) "Excuse me, Doctor, I think you need to do something about the client's blood pressure."

C) The nurse who expresses concern because the client's blood pressure remains high even with new medication is being assertive, clear, and concise. Stating that the new medication is ineffective could be interpreted as an inflammatory remark by the provider. Asking whether the provider can talk about a client does not give the provider enough information. Saying the provider must do something about the client's blood pressure may cause the provider to become defensive.

The nurse is caring for an older adult client with decreased energy who needs to get up to prevent the development of pressure ulcers. The client is unable to ambulate and wants to be alone. What should the nurse do? A) Notify the healthcare provider of the client's noncompliance. B) Leave the client alone until ready to get out of bed. C) Gain knowledge about the client from family to gain compliance. D) Proceed to get help to get the client out of bed.

C) The nurse would use knowledge and creativity to think critically about getting the client to cooperate with the medical regimen. For example, the nurse would ask the family to become involved in order to gain compliance. It is not an option to leave the client alone and not address some method of ensuring intact skin. The compliance of the client rests with the nurse, not the physician. It would not be a good idea to force the client against the client's will; the nurse would use critical thinking to find another way to meet the goal.

A nurse has recently joined an orthopedic unit in the United States that specializes in perioperative care of clients undergoing knee or hip replacement. Which nursing plan of care is this nurse likely to use most often? A) Column plan B) Concept map C) Standardized plan D) Clinical pathway

C) The orthopedic unit has a high volume of clients who are undergoing common surgical procedures. Therefore, this unit would likely use standardized plans that specify nursing care for groups of clients with common needs. Column plans and concept maps are frequently used by novice nurses when more details are needed or to stimulate critical thinking. Clinical pathways are also used for clients with common, predictable health outcomes, but they are more commonly used in Canada, Australia, and the United Kingdom.

A client with acute abdominal pain is scheduled for an appendectomy in 3 hours. While waiting for the surgery, the client reports that the pain has subsided. In this scenario, what is the priority action by the nurse? A) Determine when the client can be medicated for pain. B) Contact the surgery department. C) Contact the healthcare provider. D) Notify the nursing supervisor.

C) The pain relief being experienced by the client is consistent with rupture of the appendix. In the case of suspected rupture, the healthcare provider should be notified immediately, because if the appendix has ruptured, the client's risk for peritonitis will increase. The next time the client can be medicated for pain is not relevant in this situation. Notification of the surgery department and the nursing supervisor should not be completed before contacting the healthcare provider.

The nurse is assigned to a postpartum client who had an anesthetic block during labor and delivery. When providing care for this client, which does the nurse anticipate? A) Nocturnal enuresis B) Risk for hyperkalemia C) Residual urine D) Glycosuria

C) The postpartum woman is at risk for overdistention, incomplete bladder emptying, and buildup of residual urine (urine that remains in the bladder after voiding). Glycosuria is expected for a client during pregnancy, not during the postpartum period. Nocturnal enuresis and risk for hyperkalemia are anticipated for older adult clients

Which of the following best characterizes the sociocultural context of holistic human experience? A) Balance of physical processes B) Connection to a higher power C) Connection to others D) Equilibrium with external circumstances

C) The sociocultural context of holistic human experience involves connection with others in society. Connection to a higher power is involved in the psychospiritual context. Homeostatic balance is involved in the physical context. Equilibrium with external circumstances is involved in the environmental context.

The nurse is planning a teaching seminar for a group of young adult clients who are at risk for obesity. Which statement by the nurse best addresses their needs? A) There are drugs that are good to use to reduce weight. B) Obesity often leads to low self-esteem and depression. C) Proper diet and exercise programs can not only prevent obesity but also potentially improve the ability to think and the positivity of self-perception. D) Maternal obesity often leads to menstrual irregularities and higher incidences of infertility.

C) The young adults who are at risk for obesity need education about changing lifestyles and the importance of preventing obesity as opposed to treating it. Education should include tips on eating healthy and exercising, which can lead to other benefits than preventing obesity. Drugs are used to manage obesity, not prevent it. Information about maternal obesity is not necessarily relevant to all individuals and does not address health promotion. Although obesity is a risk factor for low self-esteem and depression, emphasizing these negative effects does not empower clients to address their risk factors.

Three weeks after receiving a donor liver, a client begins to experience fever, tachycardia, right upper quadrant pain, and increased accumulation of fluid in the abdomen. The transplanted liver also becomes dangerously enlarged. In this scenario, the client is likely experiencing which of the following conditions? A) Hyperacute rejection B) Chronic rejection C) Acute rejection D) Delayed rejection

C) There are three types of transplant rejection: hyperacute, acute, and chronic. Hyperacute rejection occurs within minutes or hours of transplantation and is characterized by organ swelling, clot formation, and hemorrhage. Acute rejection occurs in the weeks following transplantation and is marked by pain, swelling, and enlargement of the transplanted organ. Chronic rejection occurs months after transplantation and involves slow, insidious organ failure

Providing wound care, referring clients to post-trauma psychological counseling, and assisting clients with physical and occupational therapy are all activities associated with which area of nursing competence? A) Health promotion B) Illness prevention C) Health restoration D) Holistic care and support

C) These activities all fall under the umbrella of health restoration, the area of nursing competency that involves efforts to return clients to their optimal state of physical, cognitive, psychological, and spiritual health following illness or injury. Health promotion is the area of nursing competency that involves enabling individuals and communities to increase their control over the determinants of well-being, thereby improving their overall health. Illness prevention is the area of nursing competency that involves helping individuals maintain optimal health by preventing disease and injury. Although holistic care and support are important aspects of nursing, they are not considered a core competency in the same sense as health promotion, illness prevention, health restoration, and caring for the dying.

The nurse is reviewing discharge instructions with the mother of a toddler who was hospitalized for constipation. Which statement made by the toddler's mother indicates the need for further education? A) "I should recognize that when my child walks stiffly on his tiptoes, this could indicate withholding." B) "Rocking and crossing the legs could be a sign of withholding." C) "I need to make sure my child eats a low-fiber diet." D) "Soiling could be a sign of withholding because of involuntary overflow."

C) This child requires a diet that is high in fiber. This statement indicates the need for further instruction. All of the other statements made by the toddler's mother indicate appropriate understanding of the information presented regarding constipation.

A nurse is caring for a client who weighs 209 pounds and is 1.67 meters tall. The client eats a high-protein diet and lifts weights to increase muscle mass. The client presents with complications such as sleep apnea, which is often caused by obesity. Which statement regarding this client is true? A) A body mass index calculation should provide an accurate measure of the client's amount of fat. B) A body mass index calculation is the best possible means of measuring this client's fat if combined with the client's waist-to-hip ratio. C) This client's body mass index calculation might indicate a false positive for obesity. D) The client should be given a bioelectrical impedance test to most accurately measure the client's fat.

C) This client's weight might be at least partially from body building efforts, and so a body mass index calculation, which only uses the parameters of weight and height, might not accurately indicate whether this client is actually obese. Even if combined with the client's waist-to-hip ratio, a BMI calculation is not the best possible means of measuring a client's fat, and neither is a bioelectrical impedance test. Underwater weighing is considered the most accurate way to determine body fat.

The nurse is auscultating heart sounds for a pregnant client in the third trimester of pregnancy. The client wants to know why her doctor told her she had an extra heart sound at the last visit. Which response by the nurse is appropriate? A) "You will need to have an echocardiogram to determine the reason for the extra sound." B) "You are likely experiencing heart failure due to the extra fluid that accumulates during this time in pregnancy." C) "You have what is known as a ventricular gallop, and it can be a normal finding during this trimester of pregnancy." D) "You have what is known as atrial gallop, and this is cause for concern."

C) Two other heart sounds may be present in some healthy individuals. The third heart sound (S3) may be heard in children, in young adults, or in pregnant females during the third trimester. It is heard after S2 and is termed a ventricular gallop. When the atrioventricular (AV) valves open, blood flow into the ventricles may cause vibrations. These vibrations create the S3 sound during diastole. There is no need for an echocardiogram. While the S3 sound can be associated with heart failure, this is not the case during pregnancy. S4, also known as an atrial gallop, can also be present in health individuals.

The healthcare provider prescribes an indwelling urinary catheter for a client with urinary retention. Which intervention, along with strict aseptic technique, will decrease the risk of infection for this procedure? A) Irrigating the catheter with sterile saline on a daily basis B) Instructing the client to void around the catheter C) Using an anesthetic lubricating gel during insertion D) Inflating the balloon while the catheter is in the urethra

C) Unless contraindicated, the additional step of using an anesthetic lubricating gel promotes comfort and protects fragile urethral tissues from trauma, and therefore reduces risk for a catheter-associated UTI. Irrigating the catheter should not be done because it can introduce infection by allowing bacteria to enter the closed urinary drainage system. Although voiding around the catheter will decrease bladder spasms, it will not help reduce infection. The balloon is not inflated until the catheter is in the bladder, in order to prevent trauma to the urethra and therefore decrease the risk of infection.

The nurse is analyzing the intake and output record for a client being treated for dehydration. The client weighs 176 lb and had a 24-hour intake of 2000 mL and urine output of 1200 mL. Based on this data, which conclusion by the nurse is the most appropriate? A) Treatment needs to include a diuretic. B) Treatment has not been effective. C) Treatment is effective and should continue. D) Treatment has been effective and should end.

C) Urinary output is normally equivalent to the amount of fluids ingested; the usual range is 1500-2000 mL in 24 hours, or 40-80 mL in 1 hour (0.5 mL/kg per hour). Clients whose intake substantially exceeds output are at risk for fluid volume excess; however, this client is dehydrated, so the extra fluid intake is being used to improve body fluid balance. The client's output is 50 mL/hr, which is within the normal range. A diuretic is not needed because the client is being treated for dehydration. Treatment has been effective; however, it should continue until the intake and output are more balanced. Ending treatment now could further jeopardize this client's fluid balance.

A nurse is conducting a skin assessment of a patient. Upon palpating skin temperature, the nurse notes the skin is warm and red. This is an abnormal sign that may be indicative of A) decreased hydration. B) decreased blood flow to the skin. C) inflammation and elevated body temperature. D) hypothyroidism.

C) Warm, red skin indicates inflammation and elevated body temperature. Decreased skin temperature is indicative of decreased blood flow to the skin. Excessively dry skin is indicative of hypothyroidism. Poor skin turgor is indicative of decreased hydration.

The nurse is conducting a gait and posture assessment for a client who is experiencing mobility issues. Which action by the nurse is appropriate during this assessment? A) Assessing the client's muscle mass and strength B) Measuring the length and circumference of the client's extremities C) Inspecting the client's spine for curvature D) Palpating the client for tenderness and pain

C) When assessing a client's gait and posture, the nurse should be sure to inspect the client's spine for curvature. Assessing muscle mass and strength, measuring the length and circumference of the extremities, and palpating for tenderness and pain are part of the physical assessment performed by the nurse for clients who are experiencing mobility issues

The nurse is starting preoperative teaching when the client receives a phone call. When the call ends and the nurse resumes teaching, the client is visibly upset and begins to cry. Which therapeutic initial response by the nurse is appropriate? A) "You can deal with whatever is upsetting you once we have finished." B) "It's very important to focus on this teaching so that you will recover quickly after surgery." C) "I can see that phone call has upset you. Let's talk about why you are upset before we move on with teaching." D) "What can you do to solve the problem?"

C) When clients are anxious and upset, they have difficulty focusing, especially on new information, and their anxiety becomes a barrier to effective communication. The nurse needs to prioritize the client's presenting need to express concerns and then engage in teaching at a later time, when the client is able to comprehend. Telling the client to focus will not relieve the anxiety of the imminent issue. Because the nurse is unaware of the issue and is not knowledgeable of the client's situation, it is false reassurance to tell the client that the matter can be handled after the teaching is completed. This response also belittles the client's independence and self-efficacy, as does recommending problem solving.

The nurse is providing care to a client who ignores the urge to defecate when at work. The client states, "I don't like to have a bowel movement anywhere but at home." Which response by the nurse is the most appropriate? A) "This is a common practice, and it will strengthen the reflex later." B) "You will get the urge later, so you should not worry about it." C) "If you continue to ignore the urge to defecate, it can lead to problems." D) "It is better to suppress the urge than to suffer embarrassment at work."

C) When the normal defecation reflexes are inhibited, these conditioned reflexes tend to be progressively weakened. When the urge to defecate is ignored, water continues to be reabsorbed, making the feces hard and difficult to expel. Ignoring the urge repeatedly will eventually cause the urge to be lost. Embarrassment, while unwarranted, is preferable to losing the urge to defecate. Ignoring the urge will not strengthen the reflex later; it will weaken it.

The nurse is reviewing discharge instructions with a client who is newly diagnosed with asthma. Which client statement indicates a need for further teaching? A) "I need to rinse my mouth after every use of my inhaler." B) "I need to take my Singulair at least 1 hour before I eat." C) "When inhaling two different medications, I should use the bronchodilator last." D) "Because I am on theophylline, I will need to have therapeutic blood levels drawn."

C) When using two different medications taken by inhalation, the bronchodilator should always be used first. This helps open the airways to enhance the effectiveness of the second medication. The other statements are accurate and require no further education.

Which action by a nurse would require immediate intervention by another healthcare team member? A) Recapping a needle using the scoop method. B) Recapping a needle with the use of a hemostat. C) Recapping a needle while holding the cap. D) Disposing of a needle in the sharps container.

C) When working with used or contaminated sharps, nurses will employ extra precautions so as to avoid unnecessary injuries. Needles that have been used will be disposed of in an appropriate sharps container. If sharps do need to be recapped, this should be done with the use of another device (such as a hemostat), or with the scoop method. Nurses should never hold the cap in one hand while trying to guide the tip of the needle into the cap with the other-this method substantially increases the risk of a sharp-related injury.

A home care nurse is caring for a client who is recovering from recent surgical debridement that produced large amounts of exudate. Three days after the debridement, the client's surgical wound was closed with staples that are aiding in healing. Given this information, which of the following terms should the nurse use when documenting this client's care? A) Primary intention healing B) Secondary intention healing C) Tertiary intention healing D) Quaternary intention healing

C) Wounds that are left open for 3-5 days to allow edema or infection to resolve or to permit exudate to drain and then are closed with sutures, staples, or adhesive skin closures undergo tertiary intention healing. Primary intention healing occurs where tissue surfaces have been approximated (closed) and there is minimal or no tissue loss. A wound that is extensive and involves considerable tissue loss and in which the edges cannot or should not be approximated heals by secondary intention healing. Quaternary intention healing does not exist.

The nurse is assessing the nutritional status of an older client. Which finding is most likely to suggest xerostomia? A) The client refuses food because it is difficult to chew with missing teeth. B) The client frequently becomes dehydrated due to failure to remember to drink water. C) The client has a chronically dry mouth despite adequate intake of fluids. D) The client does not enjoy foods due to diminished taste.

C) Xerostomia is decreased salivation due to decreased function of salivary glands. It may decrease the taste of food, impair chewing, and lead to avoidance of certain foods. The client with a dry mouth most clearly demonstrates xerostomia. Eating patterns may be altered in clients who are missing teeth, have impaired cognition and become dehydrated, or have diminished taste buds, but these are unrelated to xerostomia.

The nurse is caring for a client with a history of latex allergies. The client develops audible wheezing, pruritus, urticaria, and signs of angioedema. Which of the following is the priority intervention for this client? A) Teach the client regarding use of a kit that contains treatment for allergic reactions. B) Administer diphenhydramine (Benadryl) by mouth every 4 hours per the healthcare provider's orders. C) Administer epinephrine 1:1000 by subcutaneous injection per the healthcare provider's orders. D) Collect a detailed history from the client regarding the history of latex allergies.

C)For reactions with wheezing, pruritus, urticaria, and angioedema, a subcutaneous injection of 0.3-0.5 mL of 1:1000 epinephrine is generally sufficient. The nurse should give the epinephrine first due to the nature and severity of symptoms. Diphenhydramine may also be given, but it would likely be administered by injection rather than mouth due to the need for rapid drug onset. Although providing client teaching and collecting a detailed history are also important, the nurse does not have time to do these things until the client's immediate and potentially dangerous physical symptoms are addressed.

The nurse is prioritizing care activities that are to be completed for a group of clients. From highest to lowest priority, list the order in which the nurse should complete the listed activities. A) Measure blood pressure before administering antihypertensive medication. B) Request dietary consult for gluten-free diet. C) Remove an intravenous access device infusing chemotherapy. D) Change a dressing on an arm wound. E) Call a family member to bring in shoes. F) Ambulate to the bathroom using a walker.

C, A, D, F, B, E The highest-priority action would be to remove an intravenous access device infusing chemotherapy because this could lead to significant tissue damage. Measuring blood pressure before administering antihypertensive medication is the next priority because this could be a timed intervention. Changing a dressing on an arm wound can occur anytime throughout the shift. Ambulating using a walker to the bathroom can be delegated to unlicensed assistive personnel. Requesting a dietary consult for gluten-free diet can also be done at any time and does not have a specific degree of urgency. Contacting family to bring in shoes can be completed at the end of the shift or delayed to the next shift.

The nurse is providing care to a client diagnosed with chronic obstructive pulmonary disease (COPD). Which clinical manifestations are the direct result of altered perfusion? Select all that apply. A) Bounding pulse B) Pink nail beds C) Cyanosis D) Confusion E) Wheezing

C, D A client who is diagnosed with COPD may have alterations in both oxygenation and perfusion. Clinical manifestations associated with a decrease in perfusion include cyanosis and confusion. A weak pulse and blue nail beds would also indicate poor perfusion. Wheezing is an abnormal breath sound that is the result of excess mucus in the airways.

While receiving report from the operating room, the nurse learns that a client's surgical wound after gallbladder removal is classified as III. What does this classification tell the nurse? Select all that apply. A) The alimentary tract was not entered. B) The wound is necrotic and infected. C) Gallbladder contents spilled into the surgical site. D) A break in sterility occurred during the surgery. E) The alimentary, respiratory, genital, or urinary tract was entered.

C, D An incision is classified as III—contaminated if gross spillage from the GI tract occurred. This classification is also identified if a major break in sterile technique occurred. An incision is classified as I—clean if the alimentary, respiratory, genital, and urinary tract are not entered. An incision is classified as IV—dirty, infected if the wound is necrotic and infected. An incision is classified as II—lean contaminated if there are no signs of infection but the alimentary, respiratory, genital, or urinary tracts were entered.

The nurse makes a home visit to a client recovering from influenza. Which client statements indicate that desired outcomes have been met? Select all that apply. A) "I'm eating healthy foods now." B) "I went back to work." C) "I haven't had chills since I left the hospital." D) "I slept the whole night without coughing." E) "I was able to take a walk today."

C, D Desired outcomes for a client recovering from the flu include absence of symptoms of acute infection (such as fever and chills), resolution of respiratory symptoms, and resumption of normal sleep-rest patterns. The facts that the client has returned to work, is able to walk, and is eating a healthy diet do not indicate that the client's flu has resolved.

The nurse reviewing lab results on one of her adult clients notices the​ client's serum sodium level is 150​ mg/dL. Based on this​ data, which interventions should the nurse plan for this​ client? Select all that apply. A. Elevate the head of the bed. B. Monitor heart rate and rhythm. C. Instruct on a​ low-sodium diet. D.Administer diuretics as prescribed. E. Administer potassium supplement as prescribed.

C, D For an elevated sodium​ level, intake of this electrolyte will need to be restricted by adherence to a​ low-sodium diet. Diuretics may also be​ prescribed, as certain diuretics block sodium absorption. Diuretics will also remove any excess fluid being held in the body because of the extra sodium. Monitoring of heart rate and rhythm would be more appropriate if the client had a potassium imbalance. Elevating the head of the bed would be appropriate if the client were demonstrating signs of fluid volume overload

Which statements are correct regarding the various layers of the heart? Select all that apply. A) The endocardium covers the entire heart and great vessels. B) The endocardium is the muscular layer of the heart that contracts during each heartbeat. C) The outermost layer of the heart is the epicardium. D) The myocardium consists of myofibril cells. E) The myocardium has four layers.

C, D The heart wall consists of three layers of tissue: the epicardium, the myocardium, and the endocardium. The epicardium covers the entire heart and great vessels, and then folds over to form the parietal layer lining the pericardium and adheres to the heart surface. The myocardium, the middle layer of the heart wall, consists of specialized cardiac muscle cells (myofibrils). The endocardium, which is the innermost layer, is a thin membrane composed of three layers. The myocardium is the muscular layer of the heart that contracts during each heartbeat. The outermost layer of the heart is the epicardium.

The nurse is performing developmental assessments on several children in a pediatric clinic setting. The nurse should recognize that which child is exhibiting a delay in meeting the expected developmental milestones? Select all that apply. A) A 2-year-old who is unable to cut with scissors B) A 2-year-old who cannot recite her phone number C) A 3-year-old who is unable to speak in sentences D) A 5-year-old who is unable to button his shirt E) A 6-year-old who is unable to sit still for a short story

C, D, E A 5-year-old should be able to button his shirt. A 6-year-old should be able to sit still for a short story; this is a task that children between 3 and 5 years old are typically able to do. A child who cannot cut with scissors by kindergarten age is considered developmentally delayed, but a 2-year-old is not expected to be able to do this. A 2-year-old is not expected to be able to recite a phone number. A 3-year-old is usually able to speak in sentences.

When providing care to a client with a wound, which evidence-based interventions should the nurse anticipate carrying out? Select all that apply. A) Keeping the wound dry B) Only covering the wound if a scab forms C) Ensuring that the wound remains moist D) Keeping the wound covered E) Teaching the client that proper wound management can reduce scarring

C, D, E Individuals with open wounds are more susceptible to contracting a skin infection, such as cellulitis. Many people believe that wounds should be kept dry and should not be covered until a scab forms. However, this slows wound healing and leaves the wound exposed to potential pathogens if the scab comes off. Wound covering helps maintain a moist environment, decreasing the chance of infection. Proper wound moisture management also reduces pain and improves the cosmetic outcome.

The nurse in a rheumatology clinic is managing care for clients who receive nonsteroidal anti-inflammatory drugs (NSAIDs) for the treatment of their disease processes. Which of the following are the primary laboratory tests the nurse should assess prior to initiation of NSAID therapy? Select all that apply. A) Serum amylase B) Electrolytes C) Creatinine clearance D) Complete blood count (CBC) E) Liver function tests

C, D, E It is important to assess the client's creatinine clearance to determine kidney function prior to initiation of NSAID therapy. It is also important to assess the client's liver function tests and complete blood count (CBC) prior to beginning NSAID therapy. There is no need to assess the client's electrolytes or serum amylase, because neither of these levels are affected by NSAIDs

The nurse decides to take a few days of personal time to invest in self-nurturing activities. Which activities indicate the nurse is taking steps to care for self? Select all that apply. A) Sleeping 4 hours each night B) Eating one meal a day C) Being active in church D) Participating in regular exercise E) Participating in daily meditation

C, D, E Some examples of activities that can help the nurse care for self include a balanced diet, regular exercise, adequate rest and sleep, recreational activities, and meditation and prayer. Eating only once a day and sleeping for 4 hours each night are not activities that help the nurse care for self.

The nurse is caring for a client who has experienced a sports-related injury to the knee. During the morning assessment, which signs of inflammation should the nurse anticipate? Select all that apply. A) Pitting edema B) Pallor C) Swelling D) Warmth E) Pain

C, D, E Swelling, warmth, and pain are all signs of inflammation. Pallor is not a sign of inflammation; redness is. Pitting edema is not a sign of inflammation.

The nurse is planning care for an older adult client diagnosed with age-related macular degeneration (AMD) that is nonexudative. Which therapies should the nurse anticipate for this client? Select all that apply. A) Laser surgery B) Eye patches C) Antioxidants D) Eyedrops E) Zinc

C, E High-dose antioxidants and zinc are the treatments of choice for early-to-intermediate dry AMD. Laser surgery is used to treat wet macular degeneration. Eyedrops and eye patches may be used after laser surgery but are not part of the initial treatment for the disorder.

The nurse is providing care to a client admitted after experiencing an acute asthma attack. Which assessment findings should the nurse identify as signs that the client has progressed to respiratory failure? Select all that apply. A) Retractions and fatigue B) Tachycardia and tachypnea C) Inaudible breath sounds D) Diffuse wheezing and the use of accessory muscles when inhaling E) Reduced wheezing and an ineffective cough

C, E Inaudible breath sounds, reduced wheezing, and ineffective cough indicate that the client is progressing to respiratory failure, and immediate interventions are necessary. During an asthma attack, tachycardia and tachypnea are common. They are early symptoms of the disease process and can be addressed without urgency. Diffuse wheezing, the use of accessory muscles when inhaling, retractions, and fatigue indicate a progression of the severity of the symptoms and may require nursing intervention, but they do not indicate respiratory failure.

The nurse is concerned that a client is at risk for pressure injuries. Which assessment data supports the nurse's concern? Select all that apply. A) Age 54 B) Body temperature within normal limits C) Low serum albumin level D) Continence of urine and stool E) Prescribed bedrest

C, E Risk factors for pressure injury development include immobility and inadequate nutrition. The client who is prescribed bedrest is at risk for immobility, and a low serum albumin level is evidence of inadequate nutrition. Continence of urine and stool would reduce the risk of pressure injury development. Although advanced age increases the risk of pressure injuries, this client is only 54 years old. Finally, normal body temperature does not increase the client's risk for pressure injury development.

Which statements regarding upper body obesity are accurate? Select all that apply. A) Upper body obesity is also called peripheral obesity. B) Upper body obesity is when the waist-to-hip ratio in men is greater than 0.8 in men or greater than 1 in women. C) Upper body obesity is associated with a greater risk of hypertension. D) Young women tend to have more intra-abdominal fat than men. E) Postmenopausal women tend to have upper body obesity

C, E Upper body obesity (also called central obesity) is identified by a waist-to-hip ratio of greater than 1 in men or 0.8 in women. Upper body obesity is associated with a greater risk of complications such as hypertension, abnormal blood lipid levels, heart disease, stroke, and elevated insulin levels. Men tend to have more intra-abdominal fat than women, although women develop a central fat distribution pattern after menopause.

The nurse is prioritizing care for a client based on nursing diagnoses. If following Maslow's hierarchy of needs, list the order in which the nurse should provide care to the client. A) Fatigue B) Anxiety C) Alteration in Perfusion D) Self-Care Deficit E) Deficient Knowledge F) Diarrhea

C, F, D, B, A, E High-priority nursing diagnoses should be addressed first. This means that Alteration in Perfusion would be the first priority. Medium-priority diagnoses should be addressed next, in order of the impact on physiologic processes. Diarrhea would be a priority over Self-Care Deficit; however, Self-Care Deficit would be a priority over Anxiety. Low-priority diagnoses would be addressed last, again in order of physiologic impact. Fatigue would be a priority over Deficient Knowledge.

The nurse is caring for a client with third spacing. Which information should the nurse use to explain this health problem to the​ client's family? A. "Fluid leaves the body through increased​ urination." B. "Fluid moves into the space in the body​ cells." C. "Fluid in the blood vessels is unavailable for the body to​ use." D. "Fluid moves into the fatty tissue under the​ skin."

C. ​Rationale: In third​ spacing, fluid moves from the vascular space into an area where it is not available to support normal physiological functioning. The fluid may move into the peritoneal space or​ pleura, where it is trapped. The unavailable fluid in third spacing may be located in the bowel or peritoneal cavity. The fluid loss that can be attributed to third spacing may be difficult to detect because the​ client's weight may remain stable and intake and output records may not indicate a fluid loss. Fluid does not leave the body or enter the intracellular space or subcutaneous tissue.

The nurse is assessing the urinalysis of a client with fluid volume deficit. On which component of the urinalysis should the nurse focus to determine the​ client's fluid​ balance? A. Nitrites B. Glucose C. Specific gravity D. Leukocyte esterase

C. ​Rationale: Specific gravity measures the concentration of urine. Glucose found in the urine is indicative of diabetes mellitus. Nitrites in the urine indicate a possible bacterial infection. Leukocyte esterase also can be indicative of a possible bacterial infection.

The school nurse notes that a​ school-age child is experiencing mild heat exhaustion after playing outside during recess. Which recommendation should the nurse make to help prevent future occurrences of​ heat-related illness? A. Teach children to drink water only before recess B. Engage children to drink water when they feel thirsty C. Move afternoon recess to cooler morning hour D. Provide a time for children to rest after recess

C. ​Rationale: To prevent​ heat-related illness, it would be best to move recess from the hottest part of the day to a cooler part of the day. Children should be encouraged to take frequent water breaks and drink before they begin to feel​ thirsty, not just when they feel thirsty or only before recess. Children should also be encouraged to take frequent rest breaks during​ recess, not just afterward.

______ is a process whereby fluid and solutes move together across a membrane from an area of higher pressure to one of lower pressure. A. Diffusion B. Osmosis C. Filtration D. Active transport

C. Filtration is a process whereby fluid and solutes move together across a membrane from one compartment to another. The movement is from an area of higher pressure to one of lower pressure. Osmosis is the movement of water across cell​ membranes, from the less concentrated solution to the more concentrated solution. Diffusion is the continual intermingling of molecules in​ liquids, gases, or solids brought about by the random movement of the molecules. Active transport is the process by which substances move across cell membranes from a less concentrated solution to a more concentrated one.

Which of the following lab values is indicative of​ hypokalemia? A. Serum potassium of 5.45​ mEq/L B. Serum sodium of 155​ mEq/L C. Serum potassium of 3.25​ mEq/L D. Serum sodium of 125​ mEq/L

C. Normal potassium levels are 3.6-5.2

The nurse is preparing to administer 20 mEq of potassium chloride to a client who has been vomiting. Which information about the purpose of this medication should the nurse explain to the​ client? A. It controls and regulates water balance in the body. B. It is vital in regulating muscle contraction and relaxation. C. It is needed to maintain​ skeletal, cardiac, and neuromuscular activity. D. It is used to synthesize protein and DNA within the​ body's cells.

C. Potassium is the major cation in intracellular​ fluids, with only a small amount found in plasma and interstitial fluid. Potassium is a vital electrolyte for​ skeletal, cardiac, and smooth muscle activity. Calcium is vital in regulating muscle contraction and relaxation. Sodium controls and regulates water balance in the body. Magnesium is used in the cells to synthesize protein and DNA.

The healthcare provider prescribes calcium gluconate for a client. For which electrolyte imbalance should the nurse assess this​ client? A. Hypochloremia B. Hypernatremia C. Hypermagnesemia D. Hyponatremia

C. Rationale: Calcium gluconate is used to treat hypermagnesemia. Hypernatremia is treated with fluid replacement. Hypochloremia is treated by increasing dietary salt and adding chloride to the IV fluid. Hyponatremia is treated by increasing dietary sodium and administering​ sodium-containing IV fluids.

The nurse is preparing material on fluid compartments in the body. Which fluids should the nurse identify as the components of extracellular​ fluid? A. Intravascular, interstitial, and intracellular fluids B. Transcellular, intracellular, and extracellular fluids C. Intravascular, interstitial, and transcellular fluids D. Intracellular, interstitial, and intravascular fluids

C. ​Rationale: Body fluids found outside of the cell include​ intravascular, interstitial, and transcellular fluids.​ Conversely, intracellular fluids are found inside the cell

The nurse is determining a​ client's fluid balance. Which method should the nurse use to identify this​ client's fluid volume excess or​ deficit? A. Blood pressure B. Skin turgor C. Daily weight D. Intake and output

C. ​Rationale: Daily weight is the best indicator of fluid volume excess or deficit. Skin​ turgor, blood​ pressure, and intake and output are assessments that would be included in the care of a client with fluid​ imbalances, but daily weight is the best indicator of changes in fluid status.

The nurse is assessing a client with fluid volume deficit. Which finding should the nurse identify that supports fluid volume​ deficit? A. Wheezes opon auscultation B. Edema C. Increased hematocrit D. Weight gain

C. ​Rationale: Increased hematocrit is a finding consistent with fluid volume deficit. Edema and weight gain are consistent with fluid volume overload. Wheezes upon auscultation of the lungs is not related to fluid imbalances.

The nurse reviews a list of clients waiting to be seen in a community health clinic. Which client should the nurse identify as experiencing the most common cause of an electrolyte imbalance in​ adolescents? A. A​ 14-year-old male who is losing water through increased insensible water loss B. A​ 16-year-old female participating in heavy exercise to lose weight for a school dance C. A​ 17-year-old female with diarrhea after gastroenteritis D. A​ 16-year-old male who is not drinking enough at wrestling practice

C. ​Rationale: The most common reason for electrolyte imbalances and FVD in children and adolescents is diarrhea or gastroenteritis. Heavy​ exercise, insensible​ loss, and not drinking enough are also potential causes of electrolyte imbalance and​ FVD, but they are not the prominent reasons in adolescents.

The nurse is caring for an adult female client who is admitted to the hospital with a possible hip fracture. Following the admission assessment, the nurse determines that the client is obese. The client's BMI is 33.2 and her waist circumference is 90 cm. How should the nurse classify her obesity and associated disease risk? A) Class I, high B) Class II, very high C) Class III, extremely high D) Class I, very high

D) A BMI of 33.2 would be in the range for Class I obesity (30.0-34.9), which is the same for men and women. However, although a 90 cm waist circumference for a man who is Class I obese would indicate high associated disease risk, for woman a 90 cm waist circumference is above the 88 cm threshold for women, and therefore her associated disease risk is very high. Her BMI would have to be higher to be classified as Class II or Class III.

A client presents with acute constipation for the second time in two months. The physician orders a diagnostic barium enema. Based on the testing order, the nurse understands that the client's condition is likely associated with: A) rectal muscle contractions. B) completeness of bowel elimination. C) the efficiency with which the food moves through the gastrointestinal tract. D) the structure of the bowel or the presence of tumors or diverticula

D) A barium enema is used to identify bowel structure, tumors, or diverticula; thus, the nurse understands that one of these is likely a causative factor in the client's condition. A defecography is used to assess rectal muscle contractions. An anorectal manometry is used to assess the completeness of bowel elimination. A colorectal transit study is used to determine how efficiently food moves through the gastrointestinal tract.

A nurse is caring for a client with congestive heart failure. The healthcare provider prescribes propranolol (Inderal) for the client. Which instruction should the nurse include when administering a beta-adrenergic like propranolol (Inderal) to the client? A) "This medication must be taken on an empty stomach." B) "You will need to discontinue the medication when your symptoms subside." C) "This medication causes constipation. You should take a laxative every day." D) "It is important to notify the healthcare provider if you experience urinary retention."

D) A beta-adrenergic blocker such as propranolol can cause urinary retention; therefore, it would be important for the client to notify the healthcare provider if this occurs. Clients should always check with their healthcare provider before stopping any medication, because there could be some major complications. Constipation has been reported from clients taking propranolol, but a laxative should not be taken every day, as one can become dependent. This medicine should be taken with food, not on an empty stomach, in order to enhance absorption

The nurse is caring for a client who is being discharged following abdominal surgery with an incision. Which instruction is most important for the nurse to teach this client regarding wound healing? A) "Thoroughly irrigate the wound with hydrogen peroxide once a day." B) "Apply a lubricating lotion to the edges of the wound twice a day." C) "Add more fruits and vegetables to your diet." D) "Notify the healthcare provider if you notice swelling, warmth, or tenderness at the wound site."

D) A client being discharged with a surgical wound has to be instructed on the detection of infection, as the skin is the first line of defense. Signs such as edema, heat, and tenderness would indicate a local infection. Increasing fruits and vegetables would increase vitamin C, which helps with wound healing, but more protein would be the best choice. Irrigating with hydrogen peroxide would break down good granulating tissue, so this also would not increase healing. Applying lubricating lotion to the edges of a wound would impede the healing process.

The nurse is reviewing discharge instructions for a client diagnosed with urinary incontinence resulting from a urinary tract infection. Which statement made by the client indicates the need for further education? A) "I should drink plenty of water to prevent damage to my kidneys while I am on the antibiotics for the infection." B) "Drinking cranberry juice will decrease the risk for developing urinary tract infections." C) "I will contact the healthcare provider prior to taking over-the-counter medications while on my antibiotic." D) "I will continue to hold my urine while in public so that I do not get another infection."

D) A client who is diagnosed with urinary incontinence secondary to a urinary tract infection will require specific education. The client who states that he or she will hold their urine while in public to decrease the risk of another infection requires more education. Urinary retention is a contributing factor to urinary tract infections. The other statements are appropriate and indicate appropriate understanding of the information presented.

Friends of a client hospitalized with asthma would like to bring the client a gift. Which gift should the nurse recommend for this client? A) A basket of flowers B) A stuffed animal C) Fruit and candy D) A book

D) A client with asthma must not be exposed to items that can exacerbate their disease process. Specific allergens, chemicals, and foods must be avoided. Flowers, food, and items that may contain dust, such as a stuffed animal, should be avoided. Objects void of irritants, such as a book, would be an appropriate gift.

The nurse is reviewing the lab values for a client being cared for on the unit. The client's phosphorus level is currently 2.0 mg/dL. Based on this data, which nursing intervention is most appropriate for the nurse to implement? A) Enforce contact precautions. B) Encourage consumption of a high-calorie carbohydrate diet. C) Strain all urine. D) Encourage consumption of milk and yogurt.

D) A phosphorus level of 2.0 is low, so the client will need additional dietary phosphorus. Providing phosphorus-rich foods such as milk and yogurt is a good way to provide that additional phosphorus. There is no indication of the need to place this client on contact precautions, to increase the client's carbohydrate calorie intake, or to strain all urine.

The nurse assigned to the newborn nursery is conducting shift assessments. While assessing one newborn, the nurse notes the respiratory rate is 52 breaths per minute. Which action by the nurse is appropriate? A) Notify the healthcare provider of this assessment finding. B) Obtain an arterial blood gas for further respiratory assessment. C) Begin monitoring the respiratory rate every 5 minutes. D) Continue to monitor the newborn per facility policy.

D) A respiratory rate of 52 breaths per minute is a normal finding in a newborn. Respiratory rates are highest and most variable in newborns. The respiratory rate of a neonate or newborn is 30-60 breaths per minute. Therefore, this client only needs monitoring. No other actions are necessary.

A nurse conducted a class on fall prevention for a group of older adult clients in the community. Which observation during a client home visit indicates that teaching on fall prevention was effective? A) All meat is placed in the freezer. B) The locks were changed on the doors. C) Scatter rugs are placed in the kitchen. D) A shower seat was placed in the shower.

D) A shower seat in the shower can prevent falls. The client who installed the seat has understood the nurse's teaching. Changing the locks may promote safety if there have been frequent break-ins, but there is no evidence of that. Scatter rugs in any area of the home are a safety hazard. Placing meat in the freezer does not help prevent falls.

The nurse is conducting a psychoeducation group with male and female clients. The nurse observes what appear to be differences in moral perspectives between the men and women. Which observation is most consistent with Gilligan's theory of moral development? A) The men are focused on human-made rules governing morality. B) The women have difficulty looking at moral issues objectively. C) The men believe that morality is tied to relationships and caring. D) The women believe that it is most important not to inflict harm.

D) According to Gilligan, men tend to consider what is right to be what is just, whereas for women what is right is taking responsibility for others as a self-chosen decision. The ethic of justice, or fairness, is based on the idea of equality: Everyone should receive the same treatment. This is the development path usually followed by men and widely accepted by moral theorists. By contrast, the ethic of care is based on the premise of nonviolence: No one should be harmed. This is the path typically followed by women.

A community nurse is planning an educational program supports the developmental tasks of adults ages 50 to 60. According to Gould's theory, which task should be emphasized? A) Engaging in self-reflection B) Viewing personality as set C) Adjusting to decreasing physical capacities D) Undergoing a period of transformation

D) According to Gould, the seventh stage of adult development is experienced by individuals ages 50-60 and is a period of transformation. Gould's theory states that personalities are set at ages 43-50, in stage 6. Adjusting to decreasing physical capacities is a part of Peck's theory, and self-reflection occurs in stage 5 of Gould's theory, during the ages 35-43.

After accidentally providing food to a client who is NPO for surgery, the nurse reports the error to the healthcare provider and follows hospital policy for managing the error. Which characteristic is demonstrated by the actions of this nurse? A) Social justice B) Human dignity C) Reliability D) Accountability

D) Accountability involves accepting responsibility for the consequences of one's actions, as the nurse did in this scenario by informing the provider and following hospital policy for managing the error. Reliability implies that a nurse is dependable. Social justice involves upholding moral, legal, and humanistic principles. Human dignity involves respect for the worth and uniqueness of individuals and populations.

What characterizes individuals with achromatopsia? A) They perceive only the secondary colors of purple, orange, and green. B) They perceive only the primary colors of red, blue, and yellow. C) They perceive some colors as indistinguishable from other colors. D) They perceive only shades of gray and no colors.

D) Achromatopsia is a rare form of color blindness in which the individual cannot distinguish any color at all and sees only shades of gray. The most common variant of color blindness is the inability to distinguish between red and green. Less common is the inability to distinguish between blue and yellow. Many people with the blue-yellow variant also have problems distinguishing between green and red.

The nurse is providing teaching on the recommended hearing tests for older adults. Which information should be included in this teaching? A) Schedule an annual hearing test until the age of 50 and then have a test every 6 months. B) Annual screenings are recommended for adults with diabetes. C) For individuals without comorbidities, hearing exams should be repeated every 1-3 years for ages 55-64, and every 1-2 years for ages 65 and above. D) Have a hearing test every 10 years until age 50 and then every 3 years.

D) Adults should be screened for hearing at least every 10 years until the age of 50 and then every 3 years. A biannual or annual hearing test is not necessary for this age group. Annual screenings for vision are recommended for adults with diabetes, and for individuals without comorbidities, eye exams should be repeated every 1-3 years for ages 55-64, and every 1- years for ages 65 and above.

A client with congestive heart failure (CHF) is having difficulty breathing. Before leaving the room, the nurse ensures the client has an overbed table to lean on when awake if needed to ease breathing. Which technique did the nurse use to make this decision? A) Delegating a task B) Priority setting C) Conflict resolution D) Critical thinking

D) After assessing the client, the nurse sets goals for and with the client. To arrive at the goal, the nurse uses critical thinking to make the decision to provide the client with optimum ability to breathe. Delegating involves giving the task to another team member. There is no conflict in this decision. Conflict resolution usually involves a compromise that affects two sides that are in disagreement. Priority setting involves deciding which task to perform first.

Which risk factor for hypertension is modifiable? A) Age B) Ethnicity C) Family history D) Tobacco use

D) Age, race/ethnicity, family history, and genetic factors are all nonmodifiable risk factors for hypertension. Hypertension has many modifiable risk factors, including tobacco use, high sodium intake, obesity, excess alcohol consumption, and low activity level.

In planning care for a 32-year-old pregnant client with worsening GERD in her second trimester, which teaching would be inappropriate? A) Eat smaller and more frequent meals. B) Avoid foods that are known to be triggers for GERD. C) Proton pump inhibitors (PPIs) are safe to use, with the exception of omeprazole. D) The use of antacids containing sodium bicarbonate is recommended

D) Antacids that contain sodium bicarbonate can lead to fluid retention, and pregnant women should avoid using them. Antacids that contain calcium carbonate, which do not cause fluid retention, may be an appropriate alternative. For clients with mild-to-moderate GERD, lifestyle modifications may be enough to control symptoms. Eating small, frequent meals and avoiding foods that trigger symptoms may help. The majority of PPIs are considered safe for use by pregnant women; the exception is omeprazole. The FDA has concerns about fetal safety associated with maternal omeprazole use.

What is characteristic of assessment in perioperative care? A) It is primarily a component of preoperative care. B) It is used most often during the intraoperative process. C) It primarily is involved in evaluating clients during postoperative care. D) It is involved in every part of the perioperative process

D) Assessment is the most significant concept during the perioperative process and encompasses most of the other concepts. In addition to monitoring the client's vital signs and taking into account the spiritual, cultural, and emotional aspects of a client's care, assessment may include a nurse's communication with the healthcare and surgical team. Assessing what other team members gather from a nurse's communication is vital to a client's safety.

Clients with osteoarthritis (OA) can reduce their risk of further joint damage by doing which of the following? A) Applying topical analgesic creams as prescribed B) Avoiding movement of affected joints C) Taking acetaminophen or nonsteroidal anti-inflammatory drugs (NSAIDs) before joint pain becomes severe D) Receiving cortisone injections in affected joints no more than three times per year

D) Because frequent use of corticosteroids can cause joint damage, clients should receive cortisone injections in affected weight-bearing joints no more than three or four times per year. Avoiding movement of affected joints does not reduce the risk of joint damage; rather, it worsens the effects of OA. Applying topical analgesics and taking acetaminophen and NSAIDs reduces the pain of OA but does not reduce the risk of further joint damage.

The nurse is caring for a pediatric client recovering from surgery for a perforated appendix. Which nursing diagnosis should the nurse use to guide this client's care during the immediate postoperative period? A) Risk for Chronic Pain B) Risk for Impaired Perfusion C) Risk for Deficient Fluid Volume D) Risk for Infection

D) Because the client is recovering from an appendectomy, the client will most likely have acute pain. There is no evidence to suggest the client is at risk for impaired perfusion or deficient fluid volume. Because the appendix ruptured before surgery, the client is at risk for infection, specifically peritonitis.

The nurse is caring for a new older adult client who speaks a foreign language and who does not speak English. Which action by the nurse is appropriate when conducting the health history portion of the assessment? A) Speaking in a loud tone when addressing the client B) Providing the client with educational materials that are written in English C) Asking the client's adult son to translate during the assessment D) Having a medical translator available during the health history

D) Because the client speaks a foreign language, the nurse will need a medical translator to be available during the health history portion of the assessment. Although the nurse may need to increase the volume of speech because of age-related changes in the client's hearing, this is not an appropriate action until the nurse determines that the client is hard of hearing. Educational material should be provided to the client in the client's native language. Asking a family member to translate during an assessment can violate the client's privacy.

A client recovering from knee surgery is being prepared to ambulate for the first time. Prior to getting the client up, what should the nurse do? A) Ask the client about readiness to walk. B) Call for a wheelchair to start the process. C) Conduct a breathing assessment. D) Evaluate the client's level of pain.

D) Before implementing activities, the nurse considers the impact on the client. In this case, ambulation is likely to be painful, so the nurse evaluates the need for pain medication prior to the intervention. Most clients are not going to want to get up after surgery, and this intervention is not an option for the client. The nurse explains the need and makes the client as comfortable as possible. Assessing the client's breathing is not relevant to the activity. A wheelchair is not appropriate when getting the client ready to ambulate.

A nurse recommends a gluten-free diet to a client recently diagnosed with celiac disease. The client says that she wants to become pregnant and asks how a gluten-free diet might aid in that. What is not a benefit for this client of a gluten-free diet? A) A gluten-free diet may lower the risk of infertility. B) A gluten-free diet may lower the risk of miscarriage. C) A gluten-free diet may increase the probability that the child's birth weight will be healthy. D) A gluten-free diet may improve fetal bone development.

D) Calcium is critical for fetal bone development, and pregnant women should consume around 1000 mg of calcium per day, but this is a separate issue from maintaining a gluten-free diet. Studies suggest that negative outcomes in mothers and their children are often the result of undiagnosed celiac disease. Adherence to a strict gluten-free diet after diagnosis may improve outcomes for clients with celiac-related infertility, miscarriage, and stillbirth. Birth weights of children born to mothers with celiac disease who follow a gluten-free diet may also be higher than those of mothers who do not.

What is an example of chronic pain? A) Pain that precedes injury B) Pain that follows injury and ends when healing is complete C) Pain that is felt during injury and immediately after D) Pain that outlasts the healing process

D) Chronic pain is pain that lasts beyond the expected time of healing, usually for at least 6 months; it does not always have a known cause. Pain can range from mild to severe, and autonomic responses decrease over time as the body adapts to the persistent pain impulses. Chronic pain does not precede injury, nor does it subside immediately after injury, and it may not be related to an injury. It does not end when healing is complete.

The nurse is caring for a client who is admitted with cellulitis of the foot. Which assessment findings support this diagnosis? A) Blood urea nitrogen (BUN) and creatinine B) Breath sounds C) Blood cultures D) Redness, pain, and edema at the site

D) Classic signs of cellulitis are swelling, pain, redness, and edema at the site. BUN, creatinine, and blood cultures are ordered by the physician and are not nursing assessments. Breath sounds are not indicated at present, as the infection is in the foot.

A hospital has had higher than average reports of client handling and movement injuries. What could the nurse advocate for that could most help reduce the number of client handling injuries? A) Keep the clients restricted to bed B) Encourage clients to lose weight C) Hire more nurses D) Purchase lifting devices

D) Client handling and movement injuries are one of the leading causes of work-related injuries for nurses. Resources such as lifting devices, training, and lift teams can potentially reduce the number of occupational injuries of healthcare workers. Keeping the clients stationary would increase complications and slow the healing process. Encouraging clients to lose weight would not reduce injuries. Unless the hospital is understaffed, hiring more nurses would not help reduce injuries. Instead, nurses need to form lift teams to help reduce injuries.

A client asks the nurse if the staff members make many mistakes because there are so many posters and signs about safety on the walls. Which response by the nurse is best? A) "The nurses here are safe. The posters are directed at certain members of the healthcare team who have been making more mistakes than usual." B) "You don't need to worry about posters on the wall. Our primary concern is getting you well." C) "We never make mistakes here. We want the public to know that we have client safety goals here." D) "There is a potential for errors in all healthcare settings. The posters remind the staff and the clients of the need to work together to prevent them."

D) Client safety initiatives address collaborative efforts by staff and clients to promote safety in healthcare settings. These initiatives require collaboration by all members of the team, including clients. Mistakes can occur in all healthcare settings; behaviors, not goals, help to prevent them.

Which client should the nurse anticipate will have the greatest psychosocial needs? A) A client under standard precautions B) A client taking antibiotics C) A client under droplet precautions D) A client in isolation

D) Clients requiring isolation precautions can develop psychosocial problems related to their separation from other people, including sensory deprivation and decreased self-esteem. The nurse will need to provide additional care for these clients to promote their psychosocial health. Clients taking antibiotics or under standard or droplet precautions may have psychosocial needs as well, but they will not be as severe as those of the client in isolation.

The nurse is interviewing a client who is experiencing constipation. During the interview, the client states, "I don't understand what is going on. I feel the urge to go to the bathroom but, once I am in there and I begin pushing with my abdominal muscles, nothing happens." Which of the following represents the nurse's best response to the client? A) "Try taking an over-the-counter medication containing bismuth salts, such as Kaopectate or Pepto-Bismol. If your symptoms don't subside in 2 days, come back to the office." B) "Stop taking all medications until you have reestablished a normal elimination routine. Medication usage often leads to constipation." C) "Make sure that you are taking proper care of the skin in the anal area. Skin breakdown can result in hesitancy when defecating." D) "Try to avoid straining with the abdominal muscles during defecation. Doing so may actually close the anal sphincter, preventing feces from passing through."

D) Clients should be instructed to use caution when straining the abdominal muscles during defecation, because it may close the anal sphincter rather than allowing feces to pass through. Bismuth salts are antidiarrheals-not laxatives-and would likely compound the client's problem. Certain medications can lead to constipation in some clients, but it is not appropriate for the nurse to encourage the discontinuation of all medications. Skin care is a concern for clients who are experiencing diarrhea or fecal incontinence, and is not typically an issue for patients with constipation.

Fecal impaction is a mass or collection of hardened feces in the folds of the rectum or colon as a result of prolonged retention and accumulation of fecal material. Which clinical manifestation is common in cases of fecal impaction? A) No passage of stool or fecal material of any kind B) Passage of soft, formed stools C) Passage of lumpy stools that are hard and dry D) Passage of liquid, foul-smelling fecal material in the absence of formed stool

D) Clients with a fecal impaction pass liquid, foul-smelling fecal material in the absence of formed stool; this is the liquid portion of the feces that seeps around the impacted mass. Passing no stool or fecal material is indicative of an obstruction. Soft, formed stools are generally considered normal. Lumpy stools that are hard and dry are indicative of constipation

The nurse identifies the nursing diagnosis of Chronic Pain as being appropriate for a client with fibromyalgia. Which manifestation did the client most likely report that caused the nurse to select this diagnosis? A) Pain from eyestrain B) Pain from a severe skin rash C) Acute chest pain D) Tender points in the knees

D) Clients with fibromyalgia typically complain of multiple tender points generally including the neck, spine, and knees. Acute chest pain, pain from a rash, and muscle strain of the eye are not reported symptoms.

The nurse is providing care to a client who is experiencing urinary incontinence. Which independent nursing intervention is the most appropriate for this client? A) Encouraging increased fluid intake B) Providing catheter care C) Instructing on self-catheterization D) Teaching hygiene care

D) Clients with urinary incontinence must be taught hygiene care—sometimes called incontinence care—to protect against tissue breakdown. Encouraging increased fluid intake is appropriate for a client who is dehydrated. Instructing on self-catheterization and providing catheter care is appropriate for a client who is diagnosed with urinary retention

A nurse is caring for a pediatric client who is receiving an infusion of intravenous antibiotic at the ambulatory clinic. Which clinical manifestation indicates that the client is experiencing a type I hypersensitivity reaction? A) Erythema B) Fever C) Joint pain D) Hypotension

D) Clinical manifestations associated with a type I hypersensitivity reaction include hypotension, wheezing, gastrointestinal or uterine spasm, stridor, and urticaria. Erythema and fever are associated with type IV hypersensitivity reactions. Fever and joint pain are associated with type III hypersensitivity reactions

The nurse is providing teaching to a female client about dietary modifications to promote weight loss. Which statement by the nurse is accurate? A) "Your diet should consist of 1200-1600 calories per day, with calorie consumption increasing toward the end of the day." B) "Your diet should consist of 750-1000 calories per day, with just one big meal and then intermittent snacking." C) "Your diet should simply cut 500 calories per day from your normal intake, with a special attention to eliminating all fats from your diet." D) "Your diet should consist of 1000-1200 calories per day and be low in fat, high in fiber, and include a variety of foods."

D) Collaboration with a nutritionist helps clients to identify healthy foods that appeal to them and that can make up a diet plan to create a daily 500- to 1000-kcal deficit. Ideally, the recommended diet should be low in kilocalories and fat, contain adequate nutrients and minerals, and be high in dietary fiber. The client should eat regular meals with small servings. A gradual, slow weight loss of no more than 1-2 lb/week is recommended. For most individuals, this means a diet of 1000-1200 kcal/day for most women and 1200-1600 kcal/day for men. Fewer than 1200 kcal each day may lead to loss of lean tissue and nutritional deficiencies.

The nurse is preparing to assess comfort for several clients. If the nurse, in addition to assessing the client's physical experience of pain, assesses whether the client has a present and reliable personal support network, then the nurse is assessing which context of holistic human experience during this process? A) Transcendence B) Environmental C) Psychospiritual D) Sociocultural

D) Comfort is the experience of having needs for relief and ease met in four contexts: physical, psychospiritual, social, and environmental. Sociocultural comfort is related to family and social relationships, which a personal support network would exemplify. Transcendence is not a context of holistic human experience.

A preschool-age client's IV has infiltrated and must be restarted immediately for medication administration. There is no time for placing local anesthetic cream on the skin to decrease the pain associated with the procedure. Which complementary therapy would be most helpful when placing the IV for this pediatric client? A) Moderate sedation B) Restraint using a "mummy wrap" C) Anesthesia D) Distraction using bubbles

D) Complementary therapies—especially guided imagery, relaxation techniques, and distraction—can reduce the anxiety associated with the anticipation of the procedure. Playing games such as blowing bubbles would provide distraction for this pediatric client and be a valid nursing intervention. All the other choices are not considered complementary therapies and are inappropriate for the situation.

A client presents with an alteration in mobility. Which finding would suggest damage to the muscle? A) Increased PTH levels B) Decreased PTH levels C) Decreased CK levels D) Increased CK levels

D) Creatine kinase (CK) is used to detect muscle damage, muscle inflammation, rhabdomyolysis, polymyositis, and muscular dystrophy. Thus, increased CK levels are suggestive of increased muscle inflammation. Parathyroid hormone (PTH) levels are not linked to muscle inflammation but rather to osteoporosis, kidney disease, parathyroid gland tumors, lack of calcium, and vitamin D disorders.

An adult female client develops signs and symptoms of appendicitis during the night. The client is brought to the emergency department by her family. Which nursing intervention is the most culturally sensitive for this client? A) Ask the healthcare provider who should assess the client. B) Ask for a female healthcare provider to assess the client. C) Ask for a male healthcare provider to assess the client. D) Explain the assessment procedure and ask the client and family their preference

D) Culturally competent care means collaborating with the client to determine his or her preferences for medical care. The nurse should not assume that a client will want a male or female doctor. Asking the healthcare provider to decide does not include the client's wishes.

Which of the following clients would be the most appropriate candidate for autolytic debridement? A) A 47-year-old client with a stage 2 pressure injury B) A 68-year-old client with a suspected deep tissue injury C) A 71-year-old client with a stage 1 pressure injury D) A 59-year-old client with a stage 3 pressure injury

D) Debridement, regardless of type, is typically reserved for pressure injuries with full-thickness tissue loss. This includes stage 3 pressure injuries, stage 4 pressure injuries, and (in some cases) unstageable pressure injuries. Thus, only the client with a stage 3 injury would be an appropriate candidate.

The charge nurse is observing a newly licensed nurse catheterize an older adult client admitted with an enlarged prostate. Which action by the newly licensed nurse requires intervention from the charge nurse? A) The newly licensed nurse injects 10 mL of 2% lidocaine gel into the client's urethra. B) The newly licensed nurse inserts a 16 French coudé-tipped catheter. C) The newly licensed nurse uses sterile technique to place the catheter. D) The newly licensed nurse clamps the catheter after draining 800 mL

D) Draining 800 mL before clamping might cause a vasovagal response, so the charge nurse would need to intervene. Using 2% lidocaine gel 10 mL injected into the male urethra reduces discomfort during catheterization and the risk of catheter-associated infection, and it promotes pelvic muscle relaxation. A coudé-tipped catheter is passed more easily in the older man with an enlarged prostate. Sterile technique should always be used when inserting catheters.

A postoperative client prescribed pain medication every 4 to 6 hours is requesting medication every 6 hours. At 4 hours the client's pain level is 8 on a rating scale of 1 to 10. The nurse decides to give the pain medication now. What does this nurse's action exemplify? A) Meeting a client goal B) Time management skills C) Prioritizing the client's care D) Responding to a change in the client's condition

D) Each client has a plan of care, but it is the nurse who constantly evaluates the client for changes that the nurse responds to, if needed. Prioritizing involves choosing tasks in order of importance. Time management is completing the assigned tasks in the given time frame by organizing and using efficiency. The goal has not been met if the client is experiencing pain.

The nurse is caring for an older adult client who visits the clinic semiannually to help maintain quality of life. When providing caring interactions to this client, what intervention should the nurse avoid? A) Self-care B) Assessment for mental health problems C) Referral to a geriatrician D) Use of elderspeak

D) Elderspeak is a simplified speech characterized by shorter sentences and words. This type of speech shows a negative attitude toward older adults, especially adults who are generally healthy and are only seeking care to enhance quality of life. Assessments of older adults should include mental health problems, and older adults may be referred to a geriatrician as needed. Self-care is related to the nurse caring for one's self, not the nurse caring for a client.

Which of the following electrolytes would be classified as a cation? A) Chloride B) Bicarbonate C) Phosphate D) Potassium

D) Electrolytes are ions, or charged particles, that are capable of conducting electricity when dispersed in water. Electrolytes may be positively or negatively charged. Positively charged electrolytes are called cations and include sodium, potassium, calcium, and magnesium. Negatively charged electrolytes are called anions and include chloride, bicarbonate, phosphate, and sulfate.

During a care conference, the nursing student differentiates between the different theories of caring when discussing client care. Which type of knowledge is the student demonstrating? A) Aesthetic knowing B) Ethical knowing C) Personal knowing D) Empirical knowing

D) Empirical knowledge is systematic and helps to describe, explain, and predict phenomena. This student is exhibiting empirical knowing as the student is able to analyze the different theories of caring. Aesthetic knowing is the art of nursing and is expressed in creativity and style in meeting the needs of the client. Personal knowing is concerned with knowing, encountering, and actualizing the concrete, individual self. Ethical knowing focuses on matters of obligation or what ought to be done, and goes beyond simply following the ethical codes of the discipline.

The nurse is providing care to a pregnant client who has type 2 diabetes mellitus. The client has asked about how the medications she is taking will affect her fetus. How should the nurse respond? A) "The medications you are taking will not adversely affect your fetus. You should continue taking them as you did before your pregnancy." B) "The medications you are taking have a risk of causing fetal defects. You should stop taking your medications while you are pregnant." C) "The medications you are taking will not work as well when you are pregnant, so you should increase the dose of your medications." D) "If you have any concerns about how your medication will affect your fetus, you should talk to your primary care physician."

D) Encouraging the client to change medication dosages, stop taking medications, or continue with the present treatment plan after a major change in health status is outside the nursing scope of practice. If the client has concerns about medications, she should talk to her primary care physician or other provider. Depending on the medication and the client's health status, the provider may recommend increasing, decreasing, or stopping treatment during pregnancy, or the client may continue the present treatment plan. However, the nurse can reinforce any teaching provided by the physician.

A nurse who works in an extended care facility is planning a staff teaching session regarding the care of older adult clients with appendicitis. Which of the following points would be most appropriate for the nurse to include in the lesson plan? A) "Almost all older clients with appendicitis present with a moderate to high fever." B) "In older adults, the pain associated with appendicitis tends to fall closer to the midline than to McBurney point." C) "Older adults with appendicitis rarely present with confusion, although they may be agitated due to severe abdominal pain." D) "About half of older adult clients with appendicitis do not demonstrate rebound or involuntary guarding."

D) Fewer than 30% of older adults who have appendicitis present with classic symptoms. Almost half of older patients are afebrile, half demonstrate no rebound or involuntary guarding, and one fourth have no lower right quadrant tenderness or pain. Instead, older adults are likely to present with confusion. When abdominal pain is present in older adults, it is located near McBurney point, not along the midline.

Fibromyalgia is a disorder that involves which of the following? A) Muscle strength B) Respiratory control C) Heat regulation D) Pain Processing

D) Fibromyalgia is a disorder of pain processing, not a disorder of muscle strength, respiratory control, or heat regulation. Pain associated with fibromyalgia results from central amplification of pain signals, including spontaneous nerve activity, enlarged receptive fields, and abnormal levels of neurotransmitters.

A nurse is teaching a client about a dressing change that should be done three times per day. The client is from a culture that is "present oriented." Based on this data, at which times should the nurse tell the client to perform the dressing changes? A) At whatever times the client selects, as long as they are 8 hours apart B) At 9 a.m., 3 p.m., and 9 p.m. C) At whatever times the client selects, as long as the dressing is changed three times each day D) After breakfast, lunch, and dinner

D) For clients who are "present oriented," it is important to avoid fixed schedules. The nurse can offer a time range for activities and treatments, such as in the morning or after breakfast, and in the evening or before going to bed. Relating the dressing changes to regular daily activities would be a good approach for a client who is not focused on times of the day, such as 9 a.m., 3 p.m., and 9 p.m. It is not necessary for the dressing changes to be exactly 8 hours apart. Leaving it up to the client to change the dressing at any time as long as it is changed three times a day does not allow for any regularity in the dressing changes.

Bariatric surgery is an option for which category of clients? A) Overweight B) Pregnant women C) Adolescents D) Morbidly obese

D) For people with morbid obesity, bariatric or lap band surgery is an option. Bariatric surgery may be considered in adolescents in severe cases of obesity resistant to previous weight loss attempts, but not simply adolescents as a group regardless of their obesity. Bariatric surgery would not be considered for clients who are only overweight or for pregnant women.

A client with clinical depression asks the nurse for suggestions on how to improve the quality of sleep. The client often drinks a glass of wine before bedtime to help sleep. The client falls asleep quickly but then wakes up an hour or so later and often feels anxious. A period of wakefulness follows, often lasting several hours. Which of the following planning goals would be least appropriate for this client? A) Avoid the use of alcohol late in the evening. B) Reduce or remove environmental distractions from the bedroom. C) Maintain a consistent bedtime. D) Report decreased snoring.

D) For this client, avoiding the use of alcohol late in the evening would definitely be a worthwhile goal, as well as ensuring an environment and a routine conducive to sleep. However, the client's history does not indicate that snoring or apnea is a problem, so a goal related to decreasing snoring is probably least appropriate for this client unless there is reason to suspect this as a problem.

A pregnant client tested positive for group B streptococcus during her 36-week checkup. For which intervention should the nurse prepare the client in order to prevent transmission of infection to the neonate? A) Not breastfeeding the neonate during the first week after birth B) Administration of antibiotics to the neonate after birth C) Delivery by cesarean section D) Administration of antibiotics to the client during labor

D) Group B streptococcus (GBS) can be transmitted to the newborn during delivery. Administration of antibiotics during labor and delivery can prevent this transmission, so the nurse should prepare the mother for this intervention. GBS is not transmitted to the neonate through breastfeeding, antibiotics are not given to the neonate after birth, and a positive GBS test does not require cesarean delivery.

A nurse is planning an in-service on preventing infection for the staff nurses on a hospital's medical-surgical unit. Which of the following should be the priority teaching point for this in-service? A) Raising the temperature in each client's room B) Assessing vital signs once daily C) Wearing a mask for client care D) Performing hand hygiene

D) Hand hygiene is always the first and best way to stop the spread of microorganisms, which cause infections. Raising the temperature in a client's room would contribute to the growth of microorganisms. Assessing vital signs is important but should be done more frequently than once daily. Wearing a mask for all clients is not practical and is unnecessary unless a microorganism is airborne and the client is in isolation.

While reviewing safety precautions with the staff in a long-term care facility, which step should the nurse emphasize that helps to promote a safe environment for the clients? A) Keep clutter out of the hallway and inside the client's room. B) Provide dim lighting. C) Turn off alarms to reduce noise. D) Have the client wear shoes with rubber skid-resistant soles.

D) Having the client wear shoes with rubber skid-resistant soles is the most appropriate intervention to decrease the risk of client falls, which will promote a safe environment. Dim lighting will increase the risk of client falls. Both the hallways and the clients' rooms should be clutter free. Noise should be kept to a minimum, but turning off alarms would endanger clients.

Which of the following statements best describes the role of gluten in celiac disease? A) It acts as a toxin damaging the villi of the small intestine. B) It acts as an immunological body to destroy pathogens. C) It acts as a virus to cause disease. D) It acts as a foreign substance that provokes an immune response.

D) In celiac disease, gluten is a foreign substance that provokes an immune response. The immune response prompts an inflammatory response in the small bowel, which in turn leads to a loss of villi and microvilli; the immune response causes this to happen, not gluten. Gluten doesn't act as an antibody as part of the immunological response of the body; it provokes that response. It doesn't act as a viral pathogen to cause disease but as an antigen

Which of the following lab results suggests that a client with gallbladder disease is experiencing obstructed bile flow in the biliary duct system? A) Decreased WBC count B) Elevated WBC count C) Decreased direct bilirubin D) Elevated direct bilirubin

D) In clients with gallbladder disease, elevated direct bilirubin may indicate obstructed bile flow in the biliary duct system. Although clients with gallbladder disease often have an elevated WBC count, this result is suggestive of infection and inflammation rather than obstructed bile flow

Which of the following procedures used in the treatment of osteoarthritis (OA) involves removing a small amount of bone at the articulating surface of the joint and fitting a metal replacement over the end of the bone? A) Osteotomy B) Arthroplasty C) Arthroscopy D) Joint resurfacing

D) In joint resurfacing, a small amount of bone is removed at the articulating surface of the joint and a metal replacement is fitted over the end of the bone. Osteotomy involves surgical removal of a wedge of bone above or below the joint to realign the joint and shift the weight away from the damaged portion of the joint. Arthroscopy entails insertion of a small fiber optic light source, magnifying lens, and camera into the joint to visualize the joint structures. Arthroplasty is total joint replacement, in which a surgeon removes the damaged joint surfaces and replaces them with plastic, metal, or ceramic prostheses.

A client has a laceration that was closed with tissue adhesive. By what process will this wound heal? A) Tertiary intention B) Secondary intention C) Delayed primary intention D) Primary intention

D) In primary intention wound healing, the edges of the wounds are approximated and held together with sutures, bandages, or tissue adhesive. Scarring is minimal with these wounds. Secondary intention healing involves wounds that cannot be approximated and that must "heal in." These wounds are at higher risk for infection, take longer to heal, and are more prone to scarring. With tertiary intention healing, also called delayed primary intention healing, wounds are left open for 3 to 5 days to allow edema or infection to resolve before being closed by sutures, staples, or adhesive skin closures.

What is a lifestyle risk factor for fatigue? A) Chemotherapy B) Stress C) Surgery D) Inactivity

D) Inactivity is a lifestyle risk factor for fatigue. Stress is a mood disorder that is a risk factor for fatigue. Surgery and chemotherapy are medical procedures that are risk factors for fatigue.

A resident in an assisted-living facility is restless most nights and sits in the lounge area reading. When questioned, the resident reports suffering from insomnia. What should the nurse expect as a likely outcome if the resident continues with this pattern of sleep? A) Sleep paralysis B) Onset of cardiac dysfunction C) Onset of new underdiagnosed health problems D) The client's activities during the day may be hindered by these episodes.

D) Insomnia is defined as an inability to fall asleep or stay asleep on most nights for over a month. The individual experiencing insomnia is at risk for daytime drowsiness and may experience cognitive deficits, fatigue, and irritability, all of which can hinder the client's activities during the day. Sleep paralysis is a clinical manifestation of narcolepsy, not insomnia. There is no evidence that insomnia leads to cardiac problems. There is no evidence that the resident client is demonstrating underlying problems.

In which of the following situations would the nurse's actions most likely be classified as intimidation? A) When addressing a client, the nurse states, "I can't overemphasize how important it is that you stick with the treatment plan." B) When addressing a coworker, the nurse states, "Your behavior is a violation of hospital policy. If you keep it up, I'll be forced to notify your supervisor." C) When addressing a client, the nurse states, "It's essential that you take this medication at the same time each day. If you don't, your symptoms will likely intensify." D) When addressing a coworker, the nurse states, "You'd best think twice about doing that again, or you might be sorry."

D) Intimidation is bullying, threatening, or forcing someone who is physically or emotionally weaker to do something (or refrain from doing something) in order to avoid retribution. Telling a coworker that he or she will "be sorry" for carrying out a particular action is a clear example of intimidation. A more suitable, less intimidating approach would be to tell the coworker why the behavior is unacceptable and ask him or her to stop; if the coworker refuses to do so, approaching the supervisor would be an appropriate next step. Telling a client that you can't overstate the importance of treatment is not intimidation because it does not involve the threat of retribution. Explaining that failure to take a medication as prescribed may result in a worsening of symptoms is not intimidation; here, rather than threatening retribution, the nurse is providing education about the chosen pharmacological therapy.

A nurse is assessing the hospital environment in order to decrease the risk for client falls. Which intervention should the nurse implement to decrease the risk of client falls? A) Encourage the client to wear diapers. B) Read label directions. C) Lower side rails on client beds. D) Clean the environment of clutter.

D) Keeping the environment tidy and free of clutter will go a long way in preventing falls. Lowering side rails on client beds would increase the risk of falls. Reading label directions will prevent the wrong use of substances given to the client but would not directly prevent falls. Encouraging the client to wear diapers would increase functional decline, and it is not an appropriate strategy to help reduce falls.

The nurse is planning care for a newly admitted bedbound older adult client. Which nursing diagnosis would be most appropriate for this client? A) Risk of Bowel Incontinence B) Disturbed Body Image C) Risk for Diarrhea D) Risk for Constipation

D) Lack of activity, like being bedbound, is a major contributor to constipation. Lack of movement slows bowel movements. Lack of sphincter control, not bedrest, contributes to bowel incontinence. Diarrhea would come from a GI upset triggered by diseases, medication, or diet. Disturbed Body Image would affect a client who has undergone a bowel diversion.

The nurse is admitting a pediatric client to the hospital with a ventriculoperitoneal (VP) shunt malfunction. When gathering the history, the nurse learns that the client received the shunt at birth after a meningocele repair. Based on this data, which product should be avoided when providing care to this client? A) Synthetic rubber gloves B) Polyethylene gloves C) Non-powdered nitrile gloves D) Latex gloves

D) Meningocele is a form of spina bifida, and clients with a history of spina bifida are at increased risk for latex allergy. Thus, it is important for the nurse and other healthcare providers to use latex alternative products on this client, such as synthetic rubber gloves, polyethylene gloves, and non-powdered nitrile gloves.

For non-elderly adult clients who fracture a hip, why is internal fixation or casting of the fracture generally preferred over hip replacement? A) Internal fixation or casting is preferred because it does not disturb the client's epiphyseal plate. B) Internal fixation or casting is preferred because of the lower risk of deep vein thrombosis. C) Internal fixation or casting is preferred because of the shorter recovery time. D) Internal fixation or casting is preferred because of the limited longevity of hip prostheses.

D) Non-elderly adults are likely to live beyond the decade or so anticipated lifespan of a replacement hip. Internal fixation and casting are the preferred treatment methods for these clients because hip replacement may eventually necessitate revision surgery, which carries a greater level of risk than the initial hip replacement surgery. Protection of the epiphyseal plate is not a concern in adult clients, because they no longer have epiphyseal plates. Internal fixation, casting, and hip replacement all carry a similar risk of deep vein thrombosis, and none of these methods offers a definitive benefit in terms of recovery time.

The nurse is reviewing the results of laboratory tests conducted on a client admitted with an alteration in respiratory function. Which laboratory finding would be most significant for this client? A) Hemoglobin level 14 g/dL B) Oxygen saturation 96% C) Serum sodium 140 mg/dL D) Blood pH 7.32

D) Normal blood pH is 7.35-7.45. A decreased pH indicates that the client is experiencing acidosis, which indicates an alteration in oxygenation. The serum sodium does not impact the oxygen capacity of the body. The hemoglobin level affects the amount of oxygen that can be carried in the blood; however, the value is within normal limits. Oxygen saturation of 96% is within normal limits.

A nurse is overseeing a group of students who are completing a clinical rotation on a medical-surgical unit. The students are providing direct client care with the assistance of the nurse. The nurse who is overseeing the students is functioning in which capacity? A) Clinical nurse specialist B) Nurse practitioner C) Nurse entrepreneur D) Nurse educator

D) Nurse educators are responsible for classroom and often clinical teaching—as is happening in this scenario. A clinical nurse specialist has an advanced degree or expertise and is considered to be an expert in a specialized area of practice. He or she provides direct client care, educates others, consults, conducts research, and manages care. A nurse practitioner has an advanced education, is a graduate of a nurse practitioner program, and usually deals with nonemergency acute or chronic illness and provides primary ambulatory care. A nurse entrepreneur usually has an advanced degree, manages a health-related business, and may be involved in education, consultation, or research.

What is one of the primary reasons that it is important for nurses to prioritize care? A) Nurses need to plan how to accomplish all activities within one shift. B) Nurses can accomplish more if they perform the easiest or fastest interventions first. C) Nurses should perform interventions related to client preferences early in the shift. D) Nurses only have a limited amount of time to perform nursing interventions.

D) Nurses only have a limited amount of time to perform nursing interventions. By prioritizing care, nurses can ensure that high-priority interventions are completed first, followed by medium-priority and then low-priority interventions as time allows. It will likely not be possible for a nurse to plan how to accomplish all activities within one shift. Nurses often accomplish less and are more stressed if they perform the easiest or fastest interventions first. Nurses should consider client preferences for all interventions regardless of the time the intervention is completed.

An older adult client experiences a hip fracture. Prior to the injury, the client had an active lifestyle. Based on this information, which surgical procedure should the nurse anticipate? A) Total hip replacement B) Open reduction and external fixation C) Arthroplasty D) Open reduction and internal fixation

D) Open reduction and internal fixation is the preferred surgical procedure to repair a fractured hip for older adult clients who are active and will be able to use crutches with partial weight bearing following surgery. A total hip replacement, also called arthroplasty, is generally performed only when severe arthritis or an underlying bone condition is present, which does not appear to be the case given this client's activity level prior to the injury. Open reduction and external fixation is not a surgical option for a fractured hip.

Which of the following statements describes all pain? A) Pain is the result of tissue damage. B) Pain's effects are primarily physiological, not mental or emotional. C) Pain can be localized to a particular area of the body. D) Pain's effects can be verbalized.

D) Pain can be described by the client, and so all pain can be verbalized. It may be the result of tissue damage or a warning of the potential for damage. Pain is both a physical and an emotional experience. Pain may not be localized in its effects

The nurse is working in a urology clinic and is providing care for a client with stress urinary incontinence. The nurse has chosen the diagnosis of Stress Urinary Incontinence related to sphincter incompetence. Which is the desired outcome for a client with this diagnosis? A) The client will stop the flow of urine when voiding. B) The client will improve her incontinence within 1 month. C) The client will empty her bladder every time she voids. D) The client will perform 4-5 squeezes (Kegel exercises) for 10-15 seconds.

D) Performing 4-5 squeezes for 10-15 seconds is the goal to start with when teaching a client Kegel exercises, which are used for stress and urge incontinence. Emptying the bladder completely every time she voids would not be realistic in the beginning. This will take time. Improved continence takes 3-6 months, so 1 month is not a realistic goal. Clients are not instructed to stop the flow of urine when voiding, because this could lead to retention.

The nurse is providing care to a client with asthma. When developing the client's plan of care, which intervention would be most appropriate to promote effective gas exchange? A) Provide adequate rest periods B) Reduce excessive stimuli C) Assist with activities of daily living D) Place in Fowler position

D) Placing in Fowler position facilitates breathing and lung expansion, promoting airway clearance. Providing adequate rest periods prevents fatigue and reduces oxygen demands. Reducing excessive stimuli promotes rest. Assisting with activities of daily living conserves energy and reduces oxygen demands

The nurse is caring for an adolescent with a chronic illness who suddenly becomes noncompliant with the medication regimen. Which intervention should the nurse choose to help improve medication compliance for this client? A) Give the client a computer-animated game that presents information on the management of the condition. B) Recommend to the client's parents that certain privileges should be taken away, such as cell phone use and texting, if compliance fails to improve. C) Arrange for the physician to discuss the risks related to noncompliance with medications to the client. D) Set up a meeting with some older teens with the same condition who have been managing their disease effectively.

D) Providing the adolescent with positive role models from his or her peer group is the intervention most likely to improve compliance. Interest in games might begin to wane at this age. Adult opinions, even from a physician, could be viewed negatively and challenged. Threatening punishment could further incite rebellion.

Which of the following findings suggests that a wound is infected with pyogenic bacteria? A) Sanguineous exudate B) Serous exudate C) Serosanguineous exudate D) Purulent exudate

D) Purulent exudate is more commonly called pus, and it is created by microorganisms known as pyogenic bacteria. In contrast, sanguineous exudate consists of large amounts of red blood cells; serous exudate is clear or straw colored and has few cells; and serosanguineous exudate consists of both clear and blood-tinged drainage

A client's spouse reports the presence of a reddened area on the client's coccyx and wants to massage the area. Which response by the nurse is appropriate? A) "I will need to obtain an order from the healthcare provider to perform a massage." B) "Massaging the area twice daily will help restore circulation and should be incorporated into the plan of care." C) "I will record these findings in the medical record." D) "Massage may actually cause more harm to a potentially compromised area of skin."

D) Redness may indicate the presence of a stage 1 pressure injury. Evidence suggests that massage over bony prominences like the coccyx can cause or worsen deep tissue trauma in patients at risk for a pressure injury. Massage should thus be restricted when problems are noted. Even when appropriate and therapeutic for a client, massages do not require a healthcare provider's order.

The nurse is caring for a client who sustained multiple injuries in an automobile accident. As a part of secondary prevention for this client, which does the nurse include in the plan of care? A) Promote wellness. B) Detect early disease. C) Restore the client to previous functioning. D) Prevent the progression of more symptoms.

D) Rehabilitation is tertiary prevention and is aimed at restoring the client to the previous level of functioning. Prevention of the progression of symptoms and early detection of disease are secondary preventions. Promoting wellness is considered primary prevention.

A nurse is conducting a training session with new staff members at a nursing home. One of the staff members asks why the facility's older adult clients are at elevated risk for pressure injuries. Which response is best? A) "As people age, their epidermis becomes more elastic. This increased elasticity makes older adults' skin more susceptible to damage." B) "As compared to younger clients, older adults have higher average body temperatures, and excess body heat is a risk factor for pressure injuries." C) "Due to increased oil production, the skin of older adults tends to be moister than that of younger clients. Increased moistness increases the risk for impaired skin integrity." D) "Age-related changes in the veins and arteries put older adults at risk for diminished blood flow, which can contribute to impaired skin integrity."

D) Several factors put older adults at increased risk for pressure injuries; these include loss of lean body mass; generalized thinning of the epidermis; decreased strength and reduced elasticity of the skin; and diminished venous and arterial flow due to aging vascular walls. Increased dryness due to a decrease in the amount of oil produced by the sebaceous glands also increases the risk for impaired skin integrity in older adults. Although excess body heat is a risk factor for pressure injuries, older adults tend to have lower average body temperatures than younger clients.

Necrosis of respiratory epithelial cells and shedding of serous and ciliated cells of the respiratory tract produce which common symptom of influenza? A) Malaise B) Coryza C) Cough D) Rhinorrhea

D) Shedding of serous and ciliated cells of the respiratory tract leads to rhinorrhea, or a runny nose. Serous cells are replaced more rapidly than ciliated cells, leading to continued cough and coryza. Malaise is a general symptom of influenza and is not directly related to the shedding of cells from the respiratory tract.

A middle-age adult client states to the nurse, "I do not want to have brown spots on my skin like my parents did as they got older." Which instruction by the nurse is appropriate? A) Spend at least 15 minutes each day in the sun. B) Increase the intake of calcium. C) Increase the intake of dietary fat. D) Avoid the sun or use a sunscreen to reduce skin damage

D) Small areas of hyperpigmentation, or liver spots, occur as an age-related skin change because of hyperplasia of melanocytes in sun-exposed areas. The nurse should instruct the client to avoid the sun or use a sunscreen to reduce skin damage. The nurse should not instruct the client to spend at least 15 minutes each day in the sun. The intake of dietary fat or calcium will not affect the development of liver spots.

An older adult client is talking with the nurse about sleep problems. Which fact regarding sleep should the nurse teach this client? A) All elderly individuals experience disrupted sleep and depression. B) The need for sleep decreases with age. C) Sleep problems signal the onset of other developing medical conditions. D) The elderly do not experience as much deep sleep as a younger person

D) Starting at age 20, there is a reduction in stages 3 and 4 NREM sleep and in REM sleep, which is the deepest sleep. This reduction in deep sleep progresses with aging. Sleep problems may be from a sleep disorder as the primary condition and not another developing medical condition . Generally, the amount of sleep needed is about the same for the youth, middle-aged, and older adult. Not all elderly people experience depression and sleep disturbances.

The nurse is collecting data about a client's current health status. Which statement would assist in gathering subjective data about the client? A) "Your eyelid is red and swollen." B) "Your skin appears to be dry and irritated." C) "I see that you have bruises on your legs." D) "Tell me why you have difficulty sleeping."

D) Subjective data is information that the client gives the nurse regarding feelings or happenings. Asking the client to explain why they are having difficulty sleeping will encourage the client to give a subjective response. Noting that the client has bruises on the skin, a red and swollen eyelid, or dry and irritated skin are objective observations by the nurse and do not require a response from the client.

Data that are detectable by an observer or can be measured or tested against an accepted standard are known as A) subjective data or symptoms. B) objective data or symptoms. C) subjective data or signs. D) objective data or signs.

D) Subjective data, also referred to as symptoms, are feelings or perceptions that can be described or verified only by the client. Itching, pain, and feelings of anxiety are examples of subjective data. Objective data, also referred to as signs, are detectable by an observer or can be measured or tested against an accepted standard. Examples of objective data include discoloration of the skin and blood pressure readings

One of the roles of the community health nurse is to educate individuals about health promotion and wellness. Which activity would the nurse dismiss as irrelevant to health promotion and wellness? A) Holding classes for teenagers regarding prevention of sexually transmitted infections B) Teaching a class about smoking cessation C) Initiating infant care classes for new parents D) Implementing an exercise class for clients who have had a heart attack

D) Teaching clients about recovery activities, such as exercises that accelerate recovery after a heart attack, would fall under the category of health restoration, not health promotion. All of the other activities listed here promote health and wellness by teaching activities and behaviors that enhance clients' quality of life and maximize their personal potential, including their physical fitness and emotional health.

A client with primary hypertension is prescribed terazosin (Hytrin) to treat this condition. What is the mechanism of action of this drug? A) Prevents conversion of angiotensin I to angiotensin II B) Prevents beta-receptor stimulation in the heart C) Inhibits the flow of calcium ions across the cell membrane of vascular tissue and cardiac cells D) Blocks alpha-receptors in the vascular smooth muscle

D) Terazosin (Hytrin), an alpha-adrenergic blocker, acts by blocking alpha-receptors in the vascular smooth muscle. ACE inhibitor medications prevent conversion of angiotensin I to angiotensin II. Beta-adrenergic blockers prevent beta-receptor stimulation in the heart. Calcium channel blockers inhibit the flow of calcium ions across the cell membrane of vascular tissue and cardiac cells.

The nurse is considering nutritional support for a client experiencing severe side effects of chemotherapy. Which independent and collaborative interventions will best limit the adverse digestive and nutritional effects of chemotherapy? A) Encourage client to drink 350 mL of clear liquids within 1 hour prior to meals. B) Position the client flat during intermittent enteral nutrition feedings. C) Verify that enteral nutrition and total parenteral nutrition (TPN) are never used concurrently. D) Teach the client relaxation techniques in addition to offering IV ondansetron

D) The American Society of Clinical Oncologists suggests that individuals with cancer who are experiencing nausea and vomiting try distraction, relaxation, positive imagery, and acupuncture. The NCCIH also suggests relaxation techniques for those with chemotherapy-induced nausea and vomiting. Ondansetron is a serotonin receptor agonist used commonly as an antiemetic. Clients should be positioned with the HOB elevated 30 degrees during enteral nutrition feedings. Enteral nutrition and TPN are used concurrently. Nausea is worsened by drinking fluid within 1 hour of meals.

The nurse is caring for a client who is experiencing anaphylactic shock following the administration of a medication. Based on this data, which position is the most appropriate for the nurse to place the client? A) Trendelenburg position B) Flat, with legs slightly elevated C) Supine position D) High-Fowler position

D) The Trendelenburg position elevates the foot of the bed and is no longer recommended for the treatment of shock, as it causes the abdominal organs to press against the diaphragm, which impedes respirations and decreases coronary artery filling. Lying flat is not recommended. A person in a supine position may not be able to maintain an open airway. Instead, placing the client in Fowler or high-Fowler position allows optimal lung expansion and ease of breathing.

The nurse is assigned to a 4-month-old infant with vomiting and diarrhea who is brought to the pediatric clinic. The infant's vital signs are temperature: 37°C, apical HR: 130, R: 40/min. The abdominal assessment reveals a soft, concave abdomen, 10 gurgles auscultated in 1 minute in all four quadrants, and tympani to percussion. Which collaborative care action does the nurse anticipate? A) Check the surgical call schedule and reserve an operating suite. B) Place the infant NPO for a barium swallow. C) Prepare a milk-based infant formula to replace fluids. D) Complete a thorough digestion assessment interview with the mother

D) The assessment data for this pediatric client indicates a nonemergent alteration in digestion that requires additional interview information from the mother. Nothing in the assessment indicates a surgical emergency. A barium swallow is not indicated for diarrhea. Milk-based formulas would be avoided until symptoms subside.

The nurse in an urgent care center assesses a 40-year-old adult client who presents with a fever of 101.2°F and complaints of painful urination. What should the nurse ask to elicit further data that indicate cystitis? A) "Do you have any symptoms of menopause?" B) "How long have you had a fever, and have you had chills with this?" C) "Do you have any upper abdominal pain or cramping?" D) "What color is your urine?"

D) The classic symptoms of cystitis include dysuria or painful urination, urinary frequency and urgency, and bloody urine or hematuria. A 40-year-old client who presents with a fever and painful urination is not experiencing symptoms of menopause. Asking about fever and chills is not specific to suspected cystitis and does not elicit information related to urinary symptoms. Pain from cystitis is typically suprapubic, not upper abdominal.

An adult client is admitted to the hospital with a diagnosis of kidney stones. The healthcare provider prescribes IV fluids, x-rays, blood work, and a Foley catheter for the client. The nurse is caring for the client 3 days after admission and documents morning vital signs of 101°F, heart rate 92, respirations 25, and blood pressure 120/80. The urinary output has decreased, and the urine is cloudy and dark amber. Based on this data, which conclusion by the nurse is the most appropriate? A) The client has passed the kidney stones. B) The client is in acute renal failure. C) The client has developed a respiratory infection. D) The client has a probable urinary tract infection.

D) The client has developed a urinary tract infection that was not present upon admission. This is a healthcare-associated infection (HAI) and is likely due to poor technique when placing the Foley catheter. There is no evidence of a respiratory infections, renal failure, or the passage of kidney stones.

An 84-year-old client with poor skin turgor has slipped down in the hospital bed. Which action by the nurse is appropriate to safely reposition this client to prevent further skin breakdown? A) Using the bed sheet to slide the client up in bed B) Placing the bed in reverse Trendelenburg position C) Using the client's arms to pull the client up in bed D) Lifting the client, using the client's legs and arms for assistance

D) The client is of advanced age and has poor skin turgor. Both of these factors put the client at increased risk for alterations in skin integrity, including damage due to shearing forces. To prevent shearing of the client's skin, the nurse should lift the client up in bed, using the client's legs and arms for assistance. Pulling the client up in bed may cause skin shearing. Sliding the client on a bed sheet also has the potential to cause shearing because the skin may adhere to the sheet. Placing the bed in reverse Trendelenburg position will not facilitate appropriate positioning of the client in the bed.

A client reports feeling tired and not refreshed after sleeping. The client also tells the nurse that family members have been making comments about the client's loud snoring at night. What should the nurse suspect as being the cause of this client's fatigue? A) Insomnia B) Depression C) Thyroid disorder D) Sleep apnea

D) The client is snoring at home, which could indicate obstructive sleep apnea. Insomnia is the absence of sleep. There is not enough information to determine if the client is depressed or has a thyroid disorder.

A client with chronic hip pain is diagnosed with osteoarthritis. Which instruction regarding home safety is most appropriate for the nurse to provide to this client? A) Walk up and down the steps at home as much as possible. B) Rest in a recliner. C) Place scatter rugs in high-traffic areas. D) Install grab bars in the bathroom near the commode and in the shower.

D) The client should be encouraged to install grab bars in the bathroom near the commode and in the shower. The client should be instructed not to overuse the affected joints with excessive stair climbing. Scatter rugs are a hazard to mobility and should be avoided. The client should also be instructed to sit in a straight-back chair, avoid slumping, and avoid use of a recliner.

An older adult client with heart failure is experiencing activity intolerance due to dyspnea on exertion. Which nursing intervention is a priority for the client? A) Complete all nursing care at the end of the shift. B) Delegate care for the client to an aide. C) Complete all nursing care in the morning. D) Pace nursing care throughout the shift.

D) The client with little energy reserve is best served by pacing care throughout the shift to allow for rest periods between activities. Completing all care at once can hurt the client's ability to heal by tiring the client too severely. Delegating care does not help the client to conserve energy. Waiting until the end of the shift would have the same effect as doing all care in the morning. The key is not to overtax the client's reserves at any time during the shift.

A client who is hospitalized after a left hip fracture is scheduled for surgery late this afternoon. After receiving report, the nurse evaluates the Buck traction applied by a new physical therapist. Which finding would indicate that the traction is correctly applied? A) A foam boot covers the right lower leg from the knee down. B) Twenty-pound weights are connected to the bottom of a foam boot. C) Weights are supported by a stool at the end of the bed. D) The left knee and hip are in alignment above a foam boot.

D) The correct placement of Buck traction permits the client's left knee and hip to align. Because Buck traction is a type of skin traction, it does not involve heavy weights; usually, 5-pound weights are used. The weights always hang free from a pulley and are never supported by a stool at the end of the bed. Also, a foam boot covers the affected leg—in this case, the left leg, not the right.

The nurse is providing teaching to the family of an older adult client with a urinary tract infection (UTI). Which common early symptom that is likely to occur in older adults should the nurse stress? A) Urinary urgency B) Blood in the urine C) Urinary frequency D) Alteration in cognition

D) The early manifestations of UTI are different for older adults than for younger adults. Older adult clients are often asymptomatic until changes in cognition occur.

What primarily differentiates lack of sleep due to a sleep disorder from lack of sleep due to another developing condition? A) The length of sleep B) The quality of sleep C) The symptoms of lack of sleep D) The cause for the lack of sleep

D) The etiology, or cause, of the lack of sleep will differ depending on whether the condition is because of a sleep disorder or another developing condition. However, the length of sleep, quality of sleep, and symptoms related to the lack of sleep may not differ.

The nurse is teaching a child care class for mothers of young children. What should the nurse teach as being the most common mode of transmission of infectious disease? A) Children who are playing board games B) Children who are sitting together eating meals C) Children who are playing with the same toy D) Children who don't wash their hands after using the bathroom

D) The fecal-oral and respiratory routes are the most common sources of transmission in children. Microorganisms might be left on toys that children share, but this is not the most common mode of transmission of infectious diseases. Playing with board games will not transmit infectious disease. Eating together will not transmit infectious disease. Poor hand hygiene is a common source of transmission.

What organism is most commonly responsible for cellulitis? A) Staphylococcus epidermidis B) Streptococcus pneumoniae C) Streptococcus viridans D) Staphylococcus aureus

D) The most common causative organism of cellulitis is Staphylococcus aureus, followed by group A Streptococcus. The other bacteria produce other types of infections

Which question best helps the nurse establish a common cause of recurrent urinary tract infections (UTIs) in a preadolescent female client? A) "When was your last UTI?" B) "How often do you shower?" C) "Do you have a family history of urinary problems?" D) "In what direction do you wipe after a bowel movement?"

D) The most important teaching to provide females is always to wipe the perianal/genital area from front to back. Escherichia coli are the most common microorganisms responsible for urinary tract infections and can easily be dragged into the urethral orifice by wiping from the anus to the urethra after defecation. Females do have a shorter urethra compared with males and are more susceptible to urinary tract infections for this reason. Personal hygiene practices, such as cleansing after a bowel movement, are a priority teaching instruction to include for this client. Urinary tract infections are not associated with families or genetics.

A nurse receives a shift report and is preparing to care for clients assigned on a medical-surgical unit. Which client should the nurse plan to assess first? A) The client who needs assistance with activities of daily living B) The client who needs help ambulating to the bathroom C) The client with a pain rating of 3/10 D) The client experiencing shortness of breath

D) The nurse begins by assessing the client who is at the greatest risk, who in this case is the client having trouble breathing. The risk for the client with mild pain is not as severe as that for the client with dyspnea. The nurse can delegate the ambulation of a client to a nursing assistive individual. The nurse can also delegate assisting a client who needs help with a bath.

A nurse is caring for a client with a stage 2 pressure injury on the coccyx who is at risk for additional pressure injuries. Which nursing intervention is appropriate when caring for this client? A) Clean the pressure injury as needed. B) Use hydrogen peroxide for chemical debridement of wound bed as needed. C) Maintain the head of the client's bed at 30 degrees. D) Avoid placing the client in the side-lying position.

D) The nurse should avoid placing the client in the side-lying position because this position places increased pressure on the bony prominence of the greater trochanter. Also, the nurse should maintain the head of the bed at the lowest degree of elevation consistent with the client's medical condition and other restrictions. In addition, the nurse should clean the client's pressure injury at every dressing change, not as needed. Hydrogen peroxide should never be used on the wound bed due to the tissue damage it promotes.

The nurse is conducting a class for a group of expectant mothers regarding basic infant care techniques. What goal will allow the nurse to best evaluate the mothers' learning? A) The mothers will be able to set goals for the next class session. B) The mothers will be able to pass a written test on how to bathe a newborn infant. C) The mothers will be able to review the major points of the class. D) The mothers will be able to provide a return demonstration of a bath on a newborn doll.

D) The nursing process identifies needs, plans, goals, and interventions and then evaluates the effectiveness of the interventions. In this case, the evaluation includes having the client demonstrate the ability to care for an infant. The nurse would review major points before the end of the class and before evaluating the effectiveness of the teaching. Passing a written test does not allow the nurse to determine if the participants are able to complete the skills taught in the class. Goals for learning are completed after assessing the knowledge level of the class.

The nurse is evaluating care provided to an older adult client with a history of cholecystitis 5 months ago. Which of the following statements on the part of the client indicates that the client met a goal in the plan of care? A) "I have increased my intake of fat." B) "I have been eating out often." C) "I have been walking 1 mile every day." D) "I have been able to gain 5 pounds on the new diet."

D) The older adult client with cholecystitis is at elevated risk for infection. Thus, a goal would be to stabilize or increase weight through appropriate dietary measures to support the client's immune system and resist infection. Exercise is excellent but does not directly support this goal. Eating out would not be a goal for a client being treated for cholecystitis. The client would want to decrease fat intake.

A nurse was involved in the perioperative care of a preterm infant requiring cardiothoracic surgery. The infant has now been moved to the NICU. How should the nurse change the focus of her assessment in the postoperative phase? A) The nurse should assess the client as preterm. B) The nurse should assess the client's respiratory status. C) The nurse should assess the client's glucose levels. D) The nurse should assess the parents' coping mechanisms.

D) The postoperative infant client will most often be transferred to and cared for in the neonatal intensive care unit (NICU) at the facility at which the infant had the procedure. At this stage, the nurse should focus largely on support for the parents. Assessment of parents' grief, guilt, anxiety, and coping mechanisms is necessary so that the nurse may better assist the infant in receiving necessary care required from the parents. Assessing the client as preterm happens during preoperative care. Assessing the client's respiratory status is given special care during the transition into intraoperative care, and regulating the client's glucose levels happens during intraoperative care. Assessing the client in these areas would not be a change in focus for the nurse.

What are the two components of the sensory process? A) Stimulus and receptor B) Kinesthesia and stereognosis C) Visual and auditory D) Reception and perception

D) The sensory process involves two components: reception and perception. Sensory reception is the process of receiving stimuli or data. Sensory perception involves the conscious organization and translation of the data or stimuli into meaningful information. Stimuli and receptors are aspects of how sensory information is received and perceived. Kinesthesia is awareness of the position and movement of body parts, and stereognosis is the ability to perceive and understand an object through touch. Visual and auditory stimuli are parts of how the body senses the external world.

What is the largest lymphoid organ in the human body? A) Thymus gland B) Bone marrow C) Tonsils D) Spleen

D) The spleen is the largest lymphoid organ in the body. Although the thymus and tonsils are also lymphoid organs, they are not as large as the spleen. Bone marrow is not an organ, but rather a tissue with several lymphoid functions.

Which of the following cells would be classified as granulocytes? A) Helper T cells B) Macrophages C) Natural killer (NK) cells D) Eosinophils

D) There are three types of granulocytes: neutrophils, eosinophils, and basophils. Macrophages are mature monocytes and thus would not be classified as granulocytes. Helper T and NK cells are both lymphocytes, not granulocytes

Which type of commitment involves a feeling of obligation to continue in a profession, usually as a result of having received benefits or having had positive experiences through engagement in the profession? A) Continuance commitment B) Affective commitment C) Reciprocal commitment D) Normative commitment

D) There are three types of professional commitment: affective, normative, and continuance. Affective commitment is an attachment to a profession and includes identification with and involvement in the profession. Normative commitment is a feeling of obligation to continue in a profession that develops as a result of having received benefits or having had positive experiences through engagement in that profession. Continuance commitment is awareness of the costs associated with leaving a profession; it develops when negative consequences of leaving, such as loss of income, are seen as reasons to remain. Page Ref: 2654

The nurse is planning care for a client with a surgical wound. Which goal related to the surgical wound is most appropriate for this client? A) The client will discharge to home as soon as possible. B) The client will resume independent activities of daily living (ADLs). C) The client will increase ambulation. D) The client will regain intact skin.

D) This client has impaired skin integrity because of a surgical wound. An appropriate goal of care would be for the client to experience wound healing to achieve intact skin. For a client who otherwise has good health, the other goals are appropriate, but they are not directly related to the surgical wound. However, for some patients, discharge to home, resuming independent ADLs, and increasing ambulation may not be appropriate goals.

Which statement is true? A) A dehydrated client would be considered to be in a hypotonic state because the client would have a lower concentration of solutes in the body in relation to water. B) A dehydrated client would be considered to be in a hypertonic state because the client would have a lower concentration of solutes in the body in relation to water. C) A dehydrated client would be considered to be in a hypotonic state because the client would have a higher concentration of solutes in the body in relation to water. D) A dehydrated client would be considered to be in a hypertonic state because the client would have a higher concentration of solutes in the body in relation to water.

D) Tonicity represents the balance between the amounts of water on either side of a membrane. Different states of balance can exist relative to the sides of the membrane. A patient may be in a hypotonic condition if there is a lower concentration of solutes to water. The opposite exists in hypertonic states when there is a higher concentration of solutes in relation to water. Dehydrated clients are hypertonic because there is a reduced percentage of water in their bodies and thus a higher concentration of solutes to water.

In what ways do type IV hypersensitivity reactions differ from other types of hypersensitivity reactions? A) Unlike other types of hypersensitivity reactions, type IV reactions are antibody-mediated responses and develop almost immediately. B) Unlike other types of hypersensitivity reactions, type IV reactions are cell-mediated responses and develop almost immediately. C) Unlike other types of hypersensitivity reactions, type IV reactions are antibody-mediated responses and take 24 hours or more to develop. D) Unlike other types of hypersensitivity reactions, type IV reactions are cell-mediated responses and take 24 hours or more to develop.

D) Type IV reactions differ from other hypersensitivity responses in two ways. First, they are cell-mediated immune responses, not antibody-mediated responses, that involve the T cells of the immune system. Second, type IV reactions are delayed rather than immediate, developing 24-48 hours after exposure to an antigen

In the presence of inflammation, a client's erythrocyte sedimentation rate (ESR) A) decreases due to the decreased proportion of fibrinogen in the blood. B) decreases due to the increased proportion of fibrinogen in the blood. C) increases due to the decreased proportion of fibrinogen in the blood. D) increases due to the increased proportion of fibrinogen in the blood.

D) When an inflammatory process is active, the increased proportion of fibrinogen in a client's blood causes the red blood cells to stick to one another and settle faster. This, in turn, results in a higher ESR reading.

The novice nurse is writing his first nursing plan of care. He includes category headings for each phase of the nursing process, includes specific and detailed information related to interventions using complete sentences, considers the client's preferences in the chosen interventions, and incorporates preventive and restorative interventions. He then signs and dates the nursing plan of care. What did the nurse do wrong when creating the plan of care? A) He should not have used category headings for each phase of the nursing process. B) He should not have included preventive measures in the care plan until restorative goals were met. C) He should have included the physician's preferences for care rather than the client's preferences. D) He should have used approved abbreviations and key words rather than complete sentences.

D) When creating a nursing plan of care, the nurse should use category headings related to the nursing process; use approved abbreviations and key words rather than complete sentences; consider the client's preferences when planning care; and incorporate preventive, health maintenance, and restorative interventions. He should also sign and date the nursing plan of care. He should not use complete sentences to communicate ideas and plans unless facility policy dictates otherwise.

The nurse is caring for a 230-lb client who needs to be repositioned every 2 hours. While repositioning the client, the nurse injured a muscle in her back. To prevent the injury and ensure safety for both the nurse and client, what should the nurse have done differently in this situation? A) She should have used proper lifting techniques. B) She should have repositioned the client only if the client requested it. C) She should have questioned the physician about the need to reposition the client. D) She should have asked for help from another nurse.

D) When moving or repositioning clients, especially larger clients, the nurse should always ask for help from another healthcare worker to prevent injury. Although using proper lifting techniques is important, they do not guarantee that injuries will not occur. In addition, there is no evidence that the nurse was not already using proper lifting techniques. The nurse should question physician orders if she is unclear about the reasoning for the order, but this is a standard best practice and would likely not require questioning. The nurse should reposition the client as ordered, not only when the client requests it.

Which of the following situations demands that all perioperative staff cover their bodies with lead shields? A) The surgical team uses a bipolar handpiece to cauterize a client's tissue. B) The surgical team uses a class 3 laser to cut a client's kidney stone. C) The surgical team uses a pneumatic tourniquet to cut off circulation to a client's hand. D) The surgical team uses radiology to take intraoperative photos.

D) When radiology is used, perioperative staff must make sure lead shields cover their bodies, including women's ovaries and men's testicles, because radiation can cause sterility. Staff should also wear a neck shield to protect the thyroid gland. The other procedures do not involve radiation and don't require lead shields.

The nurse is preparing to assess a client when one of the client's family members begins showing symptoms of latex sensitivity. Which action by the nurse is the most appropriate? A) Ask the family member to leave the unit. B) Transfer the client to a department that does not use latex products. C) Wait until Monday to report the problem to the unit supervisor. D) Obtain latex-free products for the client's room.

D) When symptoms of sensitivity to latex occur on exposure, latex-free products should be supplied. Transferring the client to a department that does not use latex products is not realistic because the family member might experience exposure on another unit. (No hospital unit can be completely latex-free.) Waiting until Monday does not solve the problem. Asking the family member to leave would be a violation of the client's rights.

Which action by a student nurse is most consistent with commitment to the nursing profession? A) The student calls in sick for clinicals in order to study for a class exam. B) The student declines to observe a new procedure for giving a necessary bath. C) The student misses class to attend a political rally. D) The student calls in sick for clinicals because of a respiratory infection.

D) Whereas calling in sick for a frivolous reason demonstrates a lack of commitment, calling in sick with a bona fide illness demonstrates protection of clients who are already compromised. Attending a political rally may be important, but for the student, attending class demonstrates greater commitment to the profession of nursing. Studying for a class exam is also important, but not more important than learning clinical skills. Indeed, a student who demonstrates commitment seeks out as many new learning experiences as possible.

Which nursing action is most appropriate when communicating with a client who has a hearing deficit? A) Drawing out the articulation of words with extra emphasis in order for the client to understand B) Using shorter phrases, which tend to be easier to understand than longer ones C) Varying the volume of voice, which is easier to understand than one consistent volume D) Writing ideas or pantomiming as appropriate in order for the client to understand

D) Writing ideas and pantomiming as appropriate are acceptable forms of communication for a client who has a hearing deficit. The nurse should not overarticulate words, use short phrases, or vary the volume of voice because these things make it more difficult to understand for the client with a hearing deficit.

The nurse is conducting a physical assessment of a middle-aged female client during an annual exam. What should the nurse assess that is particularly relevant to this age group? Select all that apply. A) Speech and language B) Body development and growth C) Sleeping patterns D) Ability to carry out activities of daily living (ADLs) E) Body mass index (BMI) measurement

D, E Areas of assessment that are relevant to middle-aged adults include BMI measurement to assess for disease risk and the ability to carry out ADLs. Speech and language, body development and growth, and sleeping patterns are more appropriate to assess in pediatric clients.

The nurse is providing care to a client who is experiencing skin inflammation and pruritus. Which of the following medications does the nurse anticipate will be prescribed for this client? Select all that apply. A) Erythromycin B) Bacitracin C) Gentamycin D) Desoximetasone E) Desonide

D, E Erythromycin is an antibacterial that interferes with bacterial DNA and protein synthesis, causing cell death. Bacitracin and gentamycin are antibiotics that interfere with bacterial replication and synthesis and are used to treat infections. Desoximetasone and desonide are topical corticosteroids that relieve inflammatory and pruritic manifestations of corticosteroid-responsive dermatoses.

The nurse is caring for a client with severe inflammation. Which assessment findings would indicate a systemic reaction to inflammation? Select all that apply. A) Erythema B) Edema C) Pain D) Tachypnea E) Tachycardia

D, E Systemic manifestations of infection include elevated or abnormally low temperature, tachycardia, tachypnea, and leukocytosis. Erythema, warmth, pain, edema, and functional impairment all indicate a local reaction

The nurse is conducting education regarding urinary health at an assisted living facility. When planning topics to include in the session, which are appropriate for the nurse to consider? Select all that apply. A) Full urinary control usually occurs at 4 or 5 years of age. B) Due to neuromuscular immaturity in infancy, voluntary urinary control is absent. C) The kidneys reach maximum size between 35 and 40 years of age. D) Renal blood flow decreases because of vascular changes and a decrease in cardiac output. E) Urinary incontinence may occur because of mobility problems or neurological impairments.

D, E When planning an education session regarding urinary health at an assisted living facility, the nurse would include information that affects the urinary health of the older adult client. Information that is appropriate for the nurse to consider is the decrease in renal blood flow due to vascular changes and that urinary incontinence may occur because of issues with mobility and neurological impairment. While all of the other statements are true regarding urinary health, they are not appropriate for this presentation to older adult clients.

The nurse plans care for a client with fluid volume deficit. Which direction should the nurse provide to nursing assistive personnel about turning and repositioning this​ client? A. Every 30 minutes B. Every 180 minutes C. Every 90 minutes D. Every 120 minutes

D. Rationale: Turning the client every 2 hours​ (120 minutes) and monitoring for evidence of skin breakdown are nursing interventions to prevent alterations in skin integrity. 30 minutes is too often and would be very disruptive to the client. 90 minutes is also too soon. 180 minutes is too long and poses a risk to the​ client's skin.

A client with nausea and vomiting has orthostatic​ hypotension, dry​ skin, flat neck​ veins, and a urine specific gravity of 1.060. Which diagnosis should the nurse use to guide this​ client's care? A. Tissue​ Perfusion: Peripheral, Ineffective​ (NANDA-I ©2014) B. Skin​ Integrity, Impaired C. Gas​ Exchange, Impaired D. Fluid​ Volume: Deficient

D. ​Rationale: The​ client's symptoms and urine specific gravity indicate deficient fluid volume. The other diagnoses are not the priority for the client at this time

The nurse is monitoring the fluid and electrolyte status of a client receiving intravenous colloids. For which imbalance should the nurse assess this​ client? A. Hypernatremia B. Hyperkalemia C. Fluid deficit D. Fluid overload

D. ​Rationale: The client receiving intravenous​ (IV) colloids or any IV fluid is at risk for fluid overload. It​ is, therefore, important to monitor the client for manifestations of fluid overload. Fluid​ deficit, hyperkalemia, and hypernatremia do not typically result when infusing colloids.

A client with dehydration secondary to poor fluid intake has a 1 kg weight loss and voids 20 mL in the last hour. Which action should the nurse take​ first? A. Infuse 100 mL of normal saline per the standing order B. Document these normal findings C. Encourage to increase oral intake of water D. Discuss a fluid challenge with the healthcare provider

D. ​Rationale: The weight loss and low urine output indicate fluid volume deficit. These findings indicate the need for a fluid challenge. A fluid challenge may be performed to evaluate fluid volume when urine output is low and cardiac or renal function is questionable. A fluid challenge helps to prevent fluid volume overload resulting from IV fluid therapy when cardiac or renal function is compromised. These are not normal findings and require intervention. Drinking water or administration of 100 mL of normal saline are not appropriate interventions for this client.

A client has a serum sodium level of 140​ mEq/L, hematocrit level of​ 31%, and generalized edema. Which intervention should the nurse make a priority for this​ client? A. Increase sodium intake in the diet B. Prepare to administer a blood transfusion C. Encourage to drink ginger ale D. Restrict fluid intake

D. ​Rationale: The​ client's laboratory values and symptoms indicate excessive fluid volume. The priority would be to restrict fluid. The other actions are not appropriate for excessive fluid volume.

The nurse is administering a blood transfusion to a client who is hemorrhaging. In which fluid compartment should the nurse identify that the client is experiencing a​ deficit? A. Transcellular Fluid B. Intracellular Fluid C. Interstitial Fluid D. Intravascular Fluid

D. Intravascular fluid ​ Rationale: Blood loss causes a deficit in the intravascular fluid​ compartment, which is a subcompartment of extracellular fluid​ (ECF). Transcellular and interstitial​ fluids, along with​ lymph, make up the other compartments of ECF. Intracellular fluid is the other major fluid compartment in the body.

Which activities should the nurse emphasize when teaching parents about how to foster development in preschool-aged children? A) Providing time for playing sports, such as basketball, to increase gross motor skills B) Helping them develop skills needed in the adult world, such as allowance budgeting C) Allowing "pretend" time, such as dress-up or role-playing activities D) Presenting diversity in culture and practices as part of home-based study

Preschool-aged children should be given activities that focus on make-believe and pretend opportunities. Gross motor activities are also important, but children this age prefer activities such as swinging, riding a tricycle, and throwing a ball to organized sports. School-age children are motivated by activities that provide a sense of worth. They concentrate on mastering skills that will help them function in the adult world. Understanding diversity, role preference, and performance is the task of the adolescent


Conjuntos de estudio relacionados

CH18 - Mastering Chemistry - Alaa Hashim

View Set

AP United States History Midterm Guide

View Set

Hinkle 66 Management of Patients With Neurologic Dysfunction.

View Set

Histotechnician Practice Questions

View Set

Health Insurance Terms / Definitions Unit 1

View Set

Chapter 25 "Asepsis and infection control" TB/coarse point questions

View Set